2ちゃんねる ■掲示板に戻る■ 全部 1- 最新50    

■ このスレッドは過去ログ倉庫に格納されています

分からない問題はここに書いてね434

1 :132人目の素数さん:2017/09/13(水) 09:04:23.05 ID:Xv9heNdt.net
さあ、今日も1日頑張ろう★☆

前スレ
分からない問題はここに書いてね433 [無断転載禁止]©2ch.net
http://rio2016.2ch.net/test/read.cgi/math/1504362539/

2 :132人目の素数さん:2017/09/13(水) 09:47:25.45 ID:i1anpb+k.net
>>1 もうお前に用はない

  ○
 く|)へ
  〉 ヾ○シ
 ̄ ̄7 ヘ/
  /  ノ
  |
 /
`|


3 :132人目の素数さん:2017/09/13(水) 10:31:35.66 ID:QWHnHK+A.net
ここは分からない問題を書くスレです。
お願いごとをするスレでも分からない問題に答えてもらえるスレでも本の気に入らない箇所を罵倒するスレでもありません。

4 :132人目の素数さん:2017/09/13(水) 10:40:42.82 ID:eL+LHDfU.net
分からない問題です
神はいますか?

5 :132人目の素数さん:2017/09/13(水) 11:42:21.64 ID:Kdrcikdz.net
「有」=「全」=「無」=「永遠」=「神」

なのでしょうか?

6 :132人目の素数さん:2017/09/13(水) 12:13:18.24 ID:xSSk7ZhE.net
そうだよ

7 :132人目の素数さん:2017/09/13(水) 13:16:15.53 ID:c08G5Hbx.net
無意味な事書くな

8 :132人目の素数さん:2017/09/13(水) 14:03:52.32 ID:CM7tAgU1.net
三角形ABCの辺AB、CA上に各々点D、EをAD:EF=2:3となるようにとる。二直線DEとBCは点Fで交わるとする
AD:BD=2:3かつAE:CE=3:1のとき、三角形ADEと四角形BCEDの面積の比は?

9 :132人目の素数さん:2017/09/13(水) 14:34:43.52 ID:/lAU5CtW.net
なんかへんなもんだいね

10 :132人目の素数さん:2017/09/13(水) 17:11:21.71 ID:GXEkDNdz.net
>>8
△ADEの面積をSとする。補助線BEを引いて考える。

AD:BD=2:3よりAB=5/2AD
よって
△ABE=5/2△ADE=5/2S

AE:CE=3:1よりAC=4/3AE
よって
△ABC=4/3△ABE
    =4/3・5/2S
    =10/3S

これにより
(四角形BCED)=△ABC-△ADE
         =10/3S-S
         =7/3S

したがって
△ADE:(四角形BCED)=S:7/3S
             =3:7


点Fの存在と「AD:EF=2:3」の与条件に謎の残る問題
続きの問題があるんかな

11 :132人目の素数さん:2017/09/13(水) 18:36:29.84 ID:cYiiB4qx.net
a<b<c<d (a,b,c,dは実数)

から

0<c-b<d-a

を導く方法を教えて下さい

12 :132人目の素数さん:2017/09/13(水) 19:06:54.84 ID:GXEkDNdz.net
>>11
まずb<cより
bを移項して
0<c-b…(1)

次にa<bより
両辺に-1をかけて
-a>-b
両辺にcを足して
c-a>c-b…(2)

またc<dより
両辺からaを引いて
c-a<d-a…(3)

(2)と(3)を合わせて
c-b<c-a<d-a
よって
c-b<d-a…(4)

(1)と(4)を合わせて
0<c-b<d-a

13 :132人目の素数さん:2017/09/13(水) 19:17:41.05 ID:cYiiB4qx.net
>>12
Thx!

14 :132人目の素数さん:2017/09/13(水) 23:16:07.02 ID:J2eQGraM.net
>>11
(d-a)-(c-b) = (d-c)+(b-a) > 0 も簡単
数直線かけば明らかなんだけどね

15 :132人目の素数さん:2017/09/14(木) 10:27:57.87 ID:o9wAZAfc.net
三角関数を級数で定義する方法があるそうですが、高校式の幾何学的な定義と一致するということはどうやって示すのですか?

16 :132人目の素数さん:2017/09/14(木) 10:56:22.61 ID:RiWKHdGC.net
「全」が完全に消滅したらどうなりますか?

17 :132人目の素数さん:2017/09/14(木) 11:29:10.26 ID:RnvZcoOa.net
チェビシェフの第1種多項式が絶対値最大値の最小値
を与えることの証明が分かりません。誰かお願いします。n時の多項式f(x)閉区間-1,1がfn(cosθ)=g(cosnθ) をみたすときn次の多項式一般に対して|f(x)|が絶対値最大値の最小値を与えることを出来るだけ簡単に証明してください。

18 :132人目の素数さん:2017/09/14(木) 13:07:45.60 ID:IkgeO4sm.net
>>17
>絶対値最大値の最小値
あなたこれまったく日本語として意味不明ですよ

19 :132人目の素数さん:2017/09/14(木) 13:14:56.76 ID:m+HLZjme.net
>>18
それはあなたのレベルが低いからですよね?
チェビシェフ多項式がわかれば普通にわかりますよ

20 :132人目の素数さん:2017/09/14(木) 13:20:03.46 ID:Wgh+OeUG.net
すり替えしてやんの

21 :132人目の素数さん:2017/09/14(木) 14:01:20.54 ID:IkgeO4sm.net
>>19
『最高次係数が1になるように規格化したn次のチェビシェフ多項式をTn(x)と書く.
また,Pnによって最高次係数が1であるn次多項式の集合を表す.このとき
  |Tn(x)| = min_{f ∈ Pn} |f(x)|
となる.』

なら私は答えを知っているし,ついでに言えばP.J.Davis "Interpolation and Approximation",p.62 に10行程度の証明が載ってます.
ただ,残念ながら『チェビシェフの第1種多項式が絶対値最大値の最小値』という字面をどう見てもこの問題とは無関係なので
ここには証明を引き写さないことにします.

22 :132人目の素数さん:2017/09/14(木) 14:38:17.29 ID:RnvZcoOa.net
>>21
僕の書き方が悪かっただけなので。あなたの書いたことが僕の言いたかったことです。それを教えて貰えませんか?

23 :132人目の素数さん:2017/09/14(木) 14:54:19.93 ID:WBG7wvqi.net
「僕」の態度も悪いけど

24 :132人目の素数さん:2017/09/14(木) 15:39:17.81 ID:UmLB2r4C.net
マルチポストが親切に教えてもらえると思うなよ

25 :132人目の素数さん:2017/09/14(木) 17:03:11.36 ID:RiWKHdGC.net
ピタゴラスとアインシュタインはどっちの方が頭が良いですか?

26 :132人目の素数さん:2017/09/14(木) 17:09:08.29 ID:SQkiLAMH.net
>>25
神が最強です

27 :132人目の素数さん:2017/09/14(木) 17:13:36.77 ID:RiWKHdGC.net
>>26
神ってのは、有=全=無のことですか?

28 :132人目の素数さん:2017/09/14(木) 17:14:51.87 ID:SQkiLAMH.net
>>27
神は神です

29 :132人目の素数さん:2017/09/14(木) 17:16:03.58 ID:RiWKHdGC.net
>>28
有=全=無=永遠=神

じゃないのでしょうか?

30 :132人目の素数さん:2017/09/14(木) 17:17:44.12 ID:SQkiLAMH.net
>>29
違います

31 :132人目の素数さん:2017/09/14(木) 17:19:37.00 ID:RiWKHdGC.net
>>30
なぜ違うのでしょうか?

32 :132人目の素数さん:2017/09/14(木) 19:16:58.34 ID:IkgeO4sm.net
>>22
いや,私のレベルは低いと言ったまま謝りもしない人に教えることなんてないし
あなたが急いで P. J. Davis "Interpolation and Approximation"を買えば
解決する問題なのでこれ以上言うことはないです.

33 :132人目の素数さん:2017/09/14(木) 20:34:29.05 ID:o9wAZAfc.net
三角関数を級数で定義する方法があるそうですが、高校式の幾何学的な定義と一致するということはどうやって示すのですか?
角の概念とどう結びつけるのですか?

34 :132人目の素数さん:2017/09/14(木) 21:02:13.03 ID:RnvZcoOa.net
>>32
それ僕じゃないんですけど

35 :132人目の素数さん:2017/09/14(木) 22:42:31.16 ID:UmLB2r4C.net
図書館行けばよくね?

36 :132人目の素数さん:2017/09/15(金) 01:14:15.80 ID:LR0trQwh.net
>>34
僕の書き方が悪かっただけなのでw

37 :132人目の素数さん:2017/09/15(金) 08:40:59.46 ID:9GqTdcdM.net
「僕」ちゃん悪くないw

38 :132人目の素数さん:2017/09/15(金) 10:08:34.22 ID:AZ6d+caw.net
>>34

補題1 n(≧0)次多項式fがn+1個以上の零点を持つならばf=0(ゼロ多項式).
証明 n+1個の零点を a0,...,an とするとき因数定理により
f(x) = (x-a0)・...・(x-an)・g(x)
となる多項式gが存在する.次数についての考察からg=0でなければならない.■

n(≧0)に対し,チェビシェフ多項式T_nを次のように定義する:
T_0(x)=1, T_1(x)=x,
T_{n+1}(x)= 2x T_n(x) - T_{n-1}(x) (n≧1).

命題1 T_nはn次多項式であり,その最高次係数は2^{n-1}.■
命題2 T_n(cosθ) = cos(nθ).
略証:帰納法による.余弦についての和積の公式を使う.■

命題1により,T_nの零点は高々n個である.
命題2により,x=cos((2k+1)π/n),[k=0,...,n-1]は
T_nの零点となる.(n個あるのでこれで全部).これらの零点は
(cosの値域から)区間I=[-1,1]に含まれていることに注意する.

同じく,x=cos(kπ/n), [k=0,...,n]のときT_n(x)=(-1)^k
となる.(正負が交互に入れ替わるのが重要).
また,命題2を考慮すれば,これらの点は区間IにおけるT_nの最大絶対値点
であることがわかる.
さて,いま関数fに対して?f?:=sup_{x∈I}|f(x)|と書くことにすると
以上の議論より次が成立する:

命題3 ?T_n? = 1. (n=0,1,2,...) ■

以下において,P_n := {最高次係数が1のn次多項式}.
U_n := T_n / (2^{n-1}) とすると,命題1より
U_n ∈ P_n であることに注意する.また,命題3により
?U_n? = 1/(2^{n-1}).

(続く)

39 :132人目の素数さん:2017/09/15(金) 10:09:24.50 ID:AZ6d+caw.net
>>34
(続き)

次の定理が我々の目標である:
定理1  U_n = argmin_{f ∈ P_n} ?f?.
証明 (方針:?p? < 1/(2^{n-1}) となるp ∈ P_nの存在を
仮定して矛盾を導く.)
仮に,p ∈ P_nであって?p? < 1/(2^{n-1})であるものが存在したとせよ.
このときq:= U_n - p とすると,最高次係数についての仮定からqは高々n-1次
多項式である.区間Iに含まれるn+1個の点 x0 < x1 < ... < xn において
交互に U_n(xk) = ±1/(2^{n-1}) となることと?p? < 1/(2^{n-1})であることから
qは x0,...,xn において交互に正負の値を取る.多項式関数の連続性により
qは区間Iにn個の零点を持つ.qはn-1次多項式なので補題1によりq=0.
従ってp=U_n となるがこれは?p?=1/(2^{n-1})を意味するので仮定と矛盾する.
よってmin_{f ∈ P_n} ?f? = 1/(2^{n-1})となる.■

40 :132人目の素数さん:2017/09/15(金) 10:10:19.34 ID:AZ6d+caw.net
あれ,縦棒二本の記号が?になってますね...読みづらくてごめん...

41 :132人目の素数さん:2017/09/15(金) 10:12:57.05 ID:AZ6d+caw.net
>>34
やりなおしで.


補題1 n(≧0)次多項式fがn+1個以上の零点を持つならばf=0(ゼロ多項式).
証明 n+1個の零点を a0,...,an とするとき因数定理により
f(x) = (x-a0)・...・(x-an)・g(x)
となる多項式gが存在する.次数についての考察からg=0でなければならない.■

n(≧0)に対し,チェビシェフ多項式T_nを次のように定義する:
T_0(x)=1, T_1(x)=x,
T_{n+1}(x)= 2x T_n(x) - T_{n-1}(x) (n≧1).

命題1 T_nはn次多項式であり,その最高次係数は2^{n-1}.■
命題2 T_n(cosθ) = cos(nθ).
略証:帰納法による.余弦についての和積の公式を使う.■

命題1により,T_nの零点は高々n個である.
命題2により,x=cos((2k+1)π/n),[k=0,...,n-1]は
T_nの零点となる.(n個あるのでこれで全部).これらの零点は
(cosの値域から)区間I=[-1,1]に含まれていることに注意する.

同じく,x=cos(kπ/n), [k=0,...,n]のときT_n(x)=(-1)^k
となる.(正負が交互に入れ替わるのが重要).
また,命題2を考慮すれば,これらの点は区間IにおけるT_nの最大絶対値点
であることがわかる.
さて,いま関数fに対して||f||:=sup_{x∈I}|f(x)|と書くことにすると
以上の議論より次が成立する:


命題3 ||T_n|| = 1. (n=0,1,2,...) ■

以下において,P_n := {最高次係数が1のn次多項式}.
U_n := T_n / (2^{n-1}) とすると,命題1より
U_n ∈ P_n であることに注意する.また,命題3により
||U_n|| = 1/(2^{n-1}).

42 :132人目の素数さん:2017/09/15(金) 10:15:42.55 ID:AZ6d+caw.net
>>34
(やりなおしの続き)

次の定理が我々の目標である:
定理1  U_n = argmin_{f ∈ P_n} ||f||.
証明 (方針:||p|| < 1/(2^{n-1}) となるp ∈ P_nの存在を
仮定して矛盾を導く.)
仮に,p ∈ P_nであって||p|| < 1/(2^{n-1})であるものが存在したとせよ.
このときq:= U_n - p とすると,最高次係数についての仮定からqは高々n-1次
多項式である.区間Iに含まれるn+1個の点 x0 < x1 < ... < xn において
交互に U_n(xk) = ±1/(2^{n-1}) となることと||p|| < 1/(2^{n-1})であることから
qは x0,...,xn において交互に正負の値を取る.多項式関数の連続性により
qは区間Iにn個の零点を持つ.qはn-1次多項式なので補題1によりq=0.
従ってp=U_n となるがこれは||p||=1/(2^{n-1})を意味するので仮定と矛盾する.
よってmin_{f ∈ P_n} ||f|| = 1/(2^{n-1})となる.■

43 :132人目の素数さん:2017/09/15(金) 10:54:51.58 ID:h3sg6csr.net
受験数学は全然できなくて無問題
あんなのは所詮公式と解法パターンの丸暗記競争だから
ルービックキューブと一緒でやり方知ってりゃ10秒で解法が組み上がる
大学行ったら数学や物理は勿論、化学だって高校数学なんか全く役に立たないよ
そうはいっても国公立の理系は少なくともセンター数学を受けないと入れない
国立、特に下位駅弁からは同レベルの理系単科私大等と比べて突出した才能が出ない一因でもある
俺も文系からの理系学部進学組みだけど高校で理系だった奴は暗記重視で本質を理解している奴はいなかった印象がある
何でも覚えようとしちゃうのね。理解しようとしないで
今でも私大なら理系学部で入試に数学を課してない所があるはず(理由は前述のとおり)
但し記述式の国語があるから地頭勝負になるけどね
数学や理科といった暗記科目で挽回の効く東大理系前期なんかよりある意味難関

44 :132人目の素数さん:2017/09/15(金) 10:55:15.89 ID:AZ6d+caw.net
>>34
間違えてすみませんでした

>>19
お前が全部悪い

45 :132人目の素数さん:2017/09/15(金) 11:06:29.78 ID:8ceeAROh.net
>>44
ありがとうございました

46 :132人目の素数さん:2017/09/15(金) 11:23:40.43 ID:h3sg6csr.net
>>44
あなたを殺すにはどうすればいいですか?

47 :132人目の素数さん:2017/09/15(金) 12:14:16.53 ID:m88TQhBX.net
数列{αn} は α1=1, αn+1 = 2αn + n(n=1,2,3...) によって定められる
このとき
αn=2α.n-1 + (ア) であるから b = α n+1 -α n とするとき
bn=イ・2(n-1) -1 となる
したがって αn = ウ
http://www.pic-loader.net/2017/56058zwns620pya.jpg

これお願いします・・
頑張ったけど時間切れ

48 :47:2017/09/15(金) 12:51:06.70 ID:m88TQhBX.net
ア n-1
イ 1
ウ 1+3n-1 -n-1

??? 間違いなのは間違いない

49 :132人目の素数さん:2017/09/15(金) 13:00:45.09 ID:X0Hkjyx7.net
だから、まだ書くなっての
普通は日曜日だろ
せっかく受けた試験の信頼性を自分で落としてどうすんの

50 :132人目の素数さん:2017/09/15(金) 13:01:35.95 ID:X0Hkjyx7.net
>>49
と思ったけど、それじゃないのか。
すまんかった。

51 :47:2017/09/15(金) 15:23:37.49 ID:m88TQhBX.net
解けたンゴ

お騒がせしました〜

52 :132人目の素数さん:2017/09/15(金) 17:41:56.37 ID:Pk6Au68T.net
高校生でも意味は分かるけど、解けない漸化式ってありますよね
a(n+1)={a(n)^2/a(n-1)}+1/n とか
こういう漸化式の一般項が、nの多項式になるのか、有利式になるのか、等々を判定する方法はありますか?

53 :132人目の素数さん:2017/09/15(金) 18:29:23.59 ID:qX4VIyCR.net
ベイズ論の基礎
コイントス問題において事前確率を一様に0.5ずつとし、ただ1度の試行で表が出たとき、ベイズ更新によって事後確率はどのように更新されるか

また、続けてもう一度表が出たとき、表の事後確率は0.75になるらしいが、それはどのような計算によって導かれるか

54 :132人目の素数さん:2017/09/15(金) 18:39:02.93 ID:0C2CZXOU.net
Uを有限集合、HをUからN_mへの関数とする。u, v∈U かつu≠vであるような任意のu, vに対して、
|{h∈H | h(u)=h(v)}|/|H|を計算せよ。

55 :132人目の素数さん:2017/09/15(金) 18:39:30.26 ID:0C2CZXOU.net
ただし、N_m={0,1,…,m-1}

56 :132人目の素数さん:2017/09/15(金) 18:40:14.66 ID:0C2CZXOU.net
HをUからN_mへの関数の集合とする

57 :132人目の素数さん:2017/09/15(金) 18:51:30.16 ID:X+5ZkarR.net
>>43
劣等感満載ですね

58 :132人目の素数さん:2017/09/15(金) 19:26:56.69 ID:kh+vJCky.net
>>47 >>48

数列{α_n}は α_1 = 1,α_{n+1} = 2α_n + n(n=1,2,3...)によって定められる。
このとき
α_n = 2 α_{n-1} + (n-1)であるから b_n = α_{n+1} - α_n とするとき
b_n = 3・2^(n-1) -1 となる
したがって α_n = 3・2^(n-1) -n-1,

------------------------------------

b_n = 2 b_{n-1} +1 より
b_n + 1 =(b_{n-1} +1)*2
= ・・・
=(b_1 +1)*2^(n-1)
= 3 2^(n-1),

59 :132人目の素数さん:2017/09/15(金) 21:01:03.49 ID:w28vy4EV.net
"kのマイナスs乗の形の総和でkが1からnマイナス1まで動くとき、s が1の場合、nが5以上の素数なら、和の分子はnの2乗を法としてゼロと合同"

証明できますか?

n=5のとき、
1/1+1/2+1/3+1/4 = 25/12
で成り立っているところまで調べました
この定理の出典や拡張などありましたらお願いします。

60 :132人目の素数さん:2017/09/15(金) 21:40:58.26 ID:X/zjAlm7.net
http://rio2016.2ch.net/test/read.cgi/math/1412425325/
171

61 :132人目の素数さん:2017/09/15(金) 21:45:58.81 ID:w28vy4EV.net
>>60
ありがとうございます。

62 :132人目の素数さん:2017/09/15(金) 23:40:02.20 ID:Qnfvx4kK.net
ラブジュース❤

63 :132人目の素数さん:2017/09/16(土) 00:49:23.31 ID:SrzKiM05.net
>>52
それは解けるけど...
a(n)= -(n+1)

64 :132人目の素数さん:2017/09/16(土) 02:28:57.51 ID:fjHcW2re.net
>>63
え、解けましたかw
すごいですね

では、a(n)がa(k)(ただしk=1,2,…,n-1)の多項式または有理式で表せてかつ解けない漸化式については、
a(n)はnの多項式や有理式であると判定できるものなのでしょうか?

65 :132人目の素数さん:2017/09/16(土) 04:19:13.63 ID:mSmt8o73.net
その手の問題は「解の表示に用いることのできる関数」を指定しないと議論できない
指定したなら後はガロア理論

ただし差分方程式だから一般には体拡大にはならないし、非線形はどこまで進んでるのかは知らん

66 :132人目の素数さん:2017/09/16(土) 11:49:06.21 ID:6p25P2EF.net
>>63
別の初項だったら?

67 :132人目の素数さん:2017/09/16(土) 12:00:30.99 ID:fjHcW2re.net
>>65
ありがとうございます。
一般項がきれいな数列を、漸化式が解けないような形で表示しようと思っていて、ふと思いつきました。
どこまで解決されているか不明なんですね。

68 :132人目の素数さん:2017/09/16(土) 12:06:46.45 ID:fjHcW2re.net
xが実数でも虚数でも、
(-1)^x
=(cosπ+isinπ)^x
=cosπx+isinπx
と定義するのが一般的なのでしょうか(複素関数論では)。

69 :132人目の素数さん:2017/09/16(土) 12:16:22.68 ID:mSmt8o73.net
>>67
その質問ならまた別の話、作られた漸化式が解けるかどうかは個々の知識に依存するだけで理論的には常に可解
どんなに見かけ上の式が変わっても本質は変わらん

もちろん元の「きれいな数列」なるものに特殊関数等が用いられてるならまた更に別の話になるけど

70 :132人目の素数さん:2017/09/16(土) 12:18:14.51 ID:kErXo0Hq.net
>>68
そうです
でもxが複素数ならcosπxも複素函数で考えるので
e^(iπx)
の方が適当でしょう

71 :132人目の素数さん:2017/09/16(土) 12:19:26.88 ID:kErXo0Hq.net
πでなくて(2n+1)πですね

72 :132人目の素数さん:2017/09/16(土) 13:32:48.66 ID:TT0KDfsj.net
a_(n+1) = k(a_n)(1-(a_n))

はい

73 :132人目の素数さん:2017/09/16(土) 13:43:29.52 ID:GxfqWaWr.net
斎藤毅著『微積分』を読んでいます。

ロピタルの定理の条件が以下のように書かれています。

「f(x) と g(x) を開区間 (u, a) で定義された微分可能な関数とし、 (u, a) で
g(x) ≠ 0 とする。さらに、 (u, a) で g'(x) > 0 か (u, a) で g'(x) < 0 とする。
左極限 lim_{x → a-0} f'(x) / g'(x) が収束するとし、その値を c とする」

なぜ、「(u, a) で g'(x) > 0 か (u, a) で g'(x) < 0 とする」と書いてあるのでしょうか?
g'(x) ≠ 0 と書けば済むことではないでしょうか?

74 :132人目の素数さん:2017/09/16(土) 13:53:02.48 ID:GxfqWaWr.net
g'(x) ≠ 0 よりも強い仮定ですよね。

75 :132人目の素数さん:2017/09/16(土) 13:55:36.25 ID:2c/onthj.net
何故簡単な微積分の本ばかり読んでいるのでしょうか?

76 :132人目の素数さん:2017/09/16(土) 14:44:40.40 ID:mSmt8o73.net
その本では導関数に対して(連続でなくとも)中間値の定理が成り立つことは示してるの?

示してないならg'の値が正負に振動する(かもしれない)場合に対する証明は?
示してるなら同値だしどうでもよい

77 :132人目の素数さん:2017/09/16(土) 19:04:59.62 ID:zll+Irdc.net
水1立法メートルを平方メートルに換算したいのですがどういう計算をすればいいのでしょうか?
高さの分を縦横のどちらかにくっつければいいだけですか?

78 :132人目の素数さん:2017/09/16(土) 19:15:26.08 ID:GxfqWaWr.net
>>76

ありがとうございます。

その中間値の定理についてはロピタルの定理以前にはおそらく書いてありません。

同値であるにしても、無駄に読者を混乱に陥れてしまいますね。

やはり、 g'(x) ≠ 0 と書くべきでしたね。

79 :132人目の素数さん:2017/09/16(土) 19:31:38.30 ID:0WmAA42w.net
>>78
混乱するのはアタマが弱いから。

ちゃんとした人は
同値なのかなと考えて
自分で確認する。

80 :132人目の素数さん:2017/09/16(土) 19:37:05.10 ID:iGQ0Whpp.net
数学の厳密性や一般性に慣れ始めてどうでもいいことにケチをつけたくなる時期なのか
それとも元々の性格なのか

81 :132人目の素数さん:2017/09/16(土) 19:54:01.73 ID:mSmt8o73.net
>>78
いや、書いてないならg'≠0だと駄目だろ……と言ったつもりなんですが

82 :132人目の素数さん:2017/09/16(土) 21:09:27.77 ID:JdCJuTuz.net
国語も苦手なんだから仕方がない

83 :132人目の素数さん:2017/09/16(土) 21:12:04.48 ID:6p25P2EF.net
>>77
ネタじゃないのなら無理

3kg の金塊を 秒 に変換するようなもの

84 :132人目の素数さん:2017/09/16(土) 21:12:45.36 ID:6p25P2EF.net
松阪君の松阪君たる所以

85 :132人目の素数さん:2017/09/16(土) 21:22:18.59 ID:ozfKE5fP.net
f(x)は任意の点で無限回微分可能でかつ非線形
f(f(x))=x
f(x)=0
のような関数は存在するのか

86 :132人目の素数さん:2017/09/16(土) 21:22:39.46 ID:ozfKE5fP.net
f(0)=0
申し訳ない

87 :132人目の素数さん:2017/09/16(土) 21:30:42.58 ID:u3kwi8mg.net
申し訳ないの一言で済む話かよ!
どうしてくれんだよテメー

88 :132人目の素数さん:2017/09/16(土) 21:50:57.86 ID:9UzbPaCe.net
f(x)は全単射
全単射だから単調増加または単調減少
単調増加のとき、f(x)≠xならあるyについてf(y)≠y
f(y)>yならf(f(y))>f(y), ここでf(y)>yなのでf(f(y))≠y
f(y)<yのときもf(f(y))≠y
単調減少のときも同じ

もっと綺麗なやり方あるのかな

89 :132人目の素数さん:2017/09/16(土) 22:30:04.15 ID:kErXo0Hq.net
>>88
y=f(x)
f(y)=x
y=xに関し対象
cos(x-y)=2(x+y)

90 :132人目の素数さん:2017/09/16(土) 22:35:55.03 ID:kErXo0Hq.net
cos(x-y)=2(x+y)+1

91 :132人目の素数さん:2017/09/17(日) 00:14:07.50 ID:GiSZJ3T4.net
お湯は何度からお湯なんですか?

92 :132人目の素数さん:2017/09/17(日) 06:33:15.02 ID:SMzO9ZfS.net
微積分の質問を「微分積分 [無断転載禁止]&copy;2ch.net」にしましたが、
このスレはほとんど誰も見ないことに気づきました。

質問内容は
309
1/(x^2+1)の定積分ではx=tanΘとおいて
やるのがふつうですが、これ以外の
方法はありますか。

ですが、マルチと言う人もいるので、あちらで答えて頂けますか?
 

93 :132人目の素数さん:2017/09/17(日) 07:49:35.74 ID:aM377AWr.net
>>92
半分ネタだが書いといた

94 :132人目の素数さん:2017/09/17(日) 10:31:03.39 ID:MQAeCOVi.net
tandθ/2 = 1/2 のとき、cosθ, tanθ, tan2θ を求めよ(西南学院大)
ヒント 答えはすべて分数 tan2θ= -**/* と分子が2桁

答,三角比の表から tanθ/2 = 0.5 -> 約 26.6°
θ = 約 26.6°×2 = 約53.2°
cos約53.2°= 約0.60 3/5
tan約53.2°= 約1.33 4/3
2θ=約106.5 だから tan106.5 = -(3.35/1) = -10/3

これ間違いっぽいのだが俺には精一杯
お願いします

95 :132人目の素数さん:2017/09/17(日) 10:32:04.34 ID:MQAeCOVi.net
× tandθ/2 
〇 tanθ/2

96 :132人目の素数さん:2017/09/17(日) 10:32:42.80 ID:WaFovudP.net
それはさすがに進学を諦めたほうが

97 :132人目の素数さん:2017/09/17(日) 10:43:04.44 ID:ABMj63yA.net
(sinx)^2+(cosx)^2=1
1+(tanx)^2=1/(cosx)^2
tan2x=(2tanx)/(1-(tanx)^2)

98 :132人目の素数さん:2017/09/17(日) 10:50:51.08 ID:ABMj63yA.net
tanx = 4/3
cosx = +3/5,-3/5
sinx = +4/5,-4/5
tan2x = -24/7

99 :94:2017/09/17(日) 11:01:00.89 ID:MQAeCOVi.net
>>98
ありがとうございます

>>96
進学とかではなくゲームです
これに答えないと洞窟から出られない

100 :132人目の素数さん:2017/09/17(日) 13:07:22.47 ID:RbR4aw+H.net
しっかし誰も解けない難しい質問ばっかでつまんねえなぁ。
本当に「実際は解いている連中ばっか」状態になったこと一度もねえじゃんw
もっと簡単な質問してこい、脳みそウンコまみれの底辺層ども。

101 :132人目の素数さん:2017/09/17(日) 14:03:06.68 ID:lrZFgKHP.net
古いコピペで喜んでる馬鹿がいるな

102 :132人目の素数さん:2017/09/17(日) 14:37:35.27 ID:RjsQ6qux.net
劣等感うんこ婆には通用しない

103 :132人目の素数さん:2017/09/17(日) 15:14:15.46 ID:O6kE+mBh.net
劣等感婆って「公理系が〜」の人?
簡単な本にいちゃもんつけてるだけの人は松坂君?
よくわかりません

どっかに数学、物理板に棲息するキチ一覧みたいなのないの?

104 :阿呆の参上:2017/09/17(日) 15:20:30.58 ID:BnHZ89Db.net
ひまだねえ

105 :132人目の素数さん:2017/09/17(日) 15:57:46.14 ID:ifsjo3DJ.net
>>103
劣等感うんこ婆のスレッド

理系思考の残念な点
http://rio2016.2ch.net/test/read.cgi/math/1400124698/

口癖
分からないんですね

松坂君(馬鹿アスペ)の最近の書き込み
ID:GxfqWaWr

106 :132人目の素数さん:2017/09/17(日) 17:10:53.24 ID:gMyUTi3U.net
Kleinberg & Tardosの本に以下のような内容の記述があります。
でも、 n > 1 のとき、 H が universal になることは決してないですよね。
u = v のとき、常に、 h(u) = h(v) なので、問題の確率は 1 ですから。



--------------------------------------------------
U を要素数の非常に多い有限集合とする。

H を U から {0, 1, ..., n-1} へのすべての写像の集合のある部分集合とする。

u, v ∈ U に対して、ランダムに選んだ h ∈ H が h(u) = h(v) を満たす確率はたかだか 1/n であるとき、
H は universal であるという。

107 :132人目の素数さん:2017/09/17(日) 17:30:26.81 ID:gMyUTi3U.net
S を #S ≦ n であるような任意の U の部分集合とする。
u を U の任意の要素とする。
X を ランダムな選択 h ∈ H に対して、値 #{s ∈ S | h(s) = h(u)} をとるようなランダム変数とする。

このとき、

E[X] ≦ 1

である。

証明:

s ∈ S に対し、
h(s) = h(u) であるならば、 1
h(s) ≠ h(u) であるならば、 0
となるようなランダム変数を X_s とする。

仮定により、 H は universal であるから、
E[X_s] = Pr[Xs = 1] ≦ 1/n

X = Σ X_s だから期待値の線形性により、

E[X] = ΣE[X_s] ≦ #S * (1/n) ≦ 1

108 :132人目の素数さん:2017/09/17(日) 17:32:37.60 ID:gMyUTi3U.net
この証明は、

u ∈ S であるとき、破綻しますよね。

109 :132人目の素数さん:2017/09/17(日) 17:37:05.97 ID:gMyUTi3U.net
Kleinbergはネヴァンリンナ賞を受賞した人だそうですが、大丈夫な人なのでしょうか?

110 :132人目の素数さん:2017/09/17(日) 18:50:50.23 ID:yIuhu+xO.net
ID:gMyUTi3Uは松坂君(馬鹿アスペ)

111 :132人目の素数さん:2017/09/17(日) 19:55:51.25 ID:PEDJEw//.net
△ABCにおいて∠A=θ、BC=1と決まっているとき内接円の半径の最大値を求めよ。

112 :132人目の素数さん:2017/09/17(日) 20:16:39.30 ID:0NgLjHN2.net
>>111
で、どうしろというのかね?

113 :132人目の素数さん:2017/09/17(日) 20:23:58.55 ID:RbR4aw+H.net
↑これが数学板の実力です
専門板なのに異常にレベルが低い
せいぜい数学の少しできる高校生レベル

114 :132人目の素数さん:2017/09/17(日) 20:34:39.58 ID:dDuY3xwn.net
でっていう

115 :132人目の素数さん:2017/09/17(日) 20:39:21.96 ID:16iYADYr.net
ID:RbR4aw+Hは劣等感うんこ婆の書き込み

116 :132人目の素数さん:2017/09/17(日) 20:43:29.37 ID:yQpGxA+8.net
>>111
パラメータが足りない

117 :132人目の素数さん:2017/09/17(日) 21:56:12.50 ID:Fq87qWEj.net
高校生です
このa>0、b>0の「0」って三角形のどこの部分を示してるんですかね?
あとa=bのときってなってますが全然同じ長さに見えないんですがどういうことなんでしょ…
https://i.imgur.com/hgRpNBm.jpg

118 :132人目の素数さん:2017/09/17(日) 21:59:33.63 ID:RbR4aw+H.net
>>117
何年か前に発見されたとかいう新しい証明ですね

aやbは長さを表すので、0より大きいのは当たり前です

a=bのときはそれとは異なる図になります
OとQが一致して半分ずつになるわけですね

119 :132人目の素数さん:2017/09/17(日) 22:05:09.18 ID:Fq87qWEj.net
>>118
とてもお詳しいですね
親切にありがとうございました

120 :132人目の素数さん:2017/09/17(日) 22:23:15.97 ID:k/sLYgaV.net
>>116


121 :132人目の素数さん:2017/09/18(月) 00:18:41.63 ID:WmEuPBdl.net
https://page.auctions.yahoo.co.jp/jp/auction/j451479908

新装版が出ていて、かつこの出品物はワンオーナー品ではありまえん。

それにも関わらず、3000円で落札されています。

これはなぜでしょうか?

かなりの難問かと思いますが、よろしくお願いいたします。

122 :132人目の素数さん:2017/09/18(月) 00:20:31.64 ID:WmEuPBdl.net
>>121

それに加えて、送料が510円です。

新品を送料込みで買ってもおつりがもらえます。

これは超難問ですね。

解ける人はいますでしょうか?

123 :132人目の素数さん:2017/09/18(月) 02:14:51.33 ID:0fzPxrGF.net
痛い奴だ

124 :132人目の素数さん:2017/09/18(月) 02:30:41.95 ID:Ap2VerJg.net
>>111
θは定数という認識で構いませんね

内心をPとすると、∠BPC=2θ(中学とかでもやる幾何の角度問題。面倒なので過程は省略)
∠BPCが2θで一定なので、PはBCを弦とする円弧の上を動く。
ここでPからBCに垂線PHを下ろすと、内接円の半径rはPHである。

rが最大になるのは、PがBCから最も離れるとき、すなわちBCの垂直二等分線上に来るときである。
このとき△PBCは二等辺三角形になり、底角が等しいので△ABCも二等辺三角形になる。

この場合について考える。
AB=AC=xとおいて余弦定理よりx=1/(2sin(θ/2))
内接円の半径と面積の関係から
(1/2)AB*ACsinθ=(1/2)r(AB+AC+BC)
(1/(2sin(θ/2)))^2(sinθ)=r(1+(1/sin(θ/2))
〜略〜(2倍角公式で変形)
r=(cos(θ/2))/(2(1+sin(θ/2)))
よって最大値は(cos(θ/2))/(2(1+sin(θ/2)))


これの類題が今年の京大4番で出てます。

125 :132人目の素数さん:2017/09/18(月) 02:59:40.82 ID:hC6SXTUk.net
>>124
>∠BPC=2θ

126 :124:2017/09/18(月) 03:25:37.01 ID:Ap2VerJg.net
>>125
何を思ったか意味不明なミスをしてますね。(θ+π)/2ですかね
そのあとはこの角を使ってないので合ってましたが

127 :132人目の素数さん:2017/09/18(月) 08:39:47.02 ID:k048EUpW.net
0 < a < b
a, b ∈ Z

とする。

表と裏の区別のできるフェアなコインを1つだけ与えられたとき、

確率 a/b で「表」
確率 (b - a)/b で「裏」

と出力するためには、コインをどのように使えばよいか?

128 :132人目の素数さん:2017/09/18(月) 10:08:36.85 ID:8Zx4M/fR.net
複素数平面上の原点Oと、Oと異なる2点A(α)、B(β)について次の問いに答えよ。ただし、|α|=4、|β|=2とする。
(1)点P(z)がz=β+4i をみたすとき、zの偏角θの取りうる範囲を求めよ。
→ (答え)π/3≦θ≦2π/3 はわかりました。
(2)線分APの中点をQ(w)とするとき、点Qの存在範囲を図示し、その領域の面積を求めよ。

(2)について、考え方だけでもいいので教えて下さい

129 :132人目の素数さん:2017/09/18(月) 10:10:24.16 ID:k048EUpW.net
また、必要なコイン投げの回数の平均値を求めよ。

130 :132人目の素数さん:2017/09/18(月) 10:19:26.33 ID:mKdfzFEo.net
>>128
α = 4 cisθ
β = 2 cisφ
とおき,とりあえずθを固定したときの軌跡をまず考える

131 :k:2017/09/18(月) 10:22:59.75 ID:nZTNdNKW.net
∫[t:0→π/2](cos|t-x|)e^sin|t-x|dt (0≦x≦π) の解き方教えてください。

132 :132人目の素数さん:2017/09/18(月) 10:42:55.43 ID:k048EUpW.net
>>128

https://imgur.com/oEGDwSW.jpg

133 :132人目の素数さん:2017/09/18(月) 10:50:04.68 ID:k048EUpW.net
w = (α + β + 4*i) /2

=

[4*(cos(s) + i*sin(s)) + 2*(cos(t) + i*sin(t)) + 4*i] / 2

=

2*(cos(s) + i*sin(s)) + (cos(t) + i*sin(t)) + 2*i

=

[2*cos(s) + cos(t)] + i*[2*sin(s) + sin(t) + 2]


(2*cos(s), 2*sin(s)) + (cos(t), sin(t)) + (0, 2)
0 ≦ s ≦ 2*π
0 ≦ t ≦ 2*π

を図示すればよい。

134 :132人目の素数さん:2017/09/18(月) 11:17:40.05 ID:Irx6kqsx.net
ここに書いてあるZの確率密度関数は正規分布に近い形になるとおもうけど
厳密にはどうなる?
https://detail.chiebukuro.yahoo.co.jp/qa/question_detail/q11174528166

135 :132人目の素数さん:2017/09/18(月) 11:18:45.13 ID:8Zx4M/fR.net
>>130
>>132
>>133
ありがとうございます!
(1)の結果を利用するのかと思って悩んでました・・・。

136 :132人目の素数さん:2017/09/18(月) 11:22:51.18 ID:k048EUpW.net
n 個の事象 A_1, A_2, …, A_n でどの2つの異なる事象も独立であるが、
どの3つ以上の互いに異なる事象も独立ではないようなものを構成する
方法を示せ。

137 :132人目の素数さん:2017/09/18(月) 11:45:50.38 ID:k048EUpW.net
刑務所長は3人の囚人 A, B, C の中からランダムに1人を選んで解放し、残りの2人を処刑する。
所長は看守にその結果を伝えたが、どの囚人にも彼が解放されるかどうか教えてはならぬと言った。

A は看守に「B, C のうち少なくとも1人は処刑される。確実に処刑されるのはどちらか教えてほしい。
あなたがそれを私に教えたとしても、私についての情報を漏らしたことにはならないから問題ない。」
と主張した。看守は納得し、「A に B は確実に処刑されると教えた。」

A は彼か C のうちどちらかは解放されるので、彼が解放される確率は、 1/3 から 1/2 になった
と喜んだ。

A は正しいか?

138 :132人目の素数さん:2017/09/18(月) 11:51:44.36 ID:k048EUpW.net
明らかに 1/3 ですね。

139 :132人目の素数さん:2017/09/18(月) 11:54:26.49 ID:k048EUpW.net
Monty Hallの問題で変更しない場合の確率ですね。

140 :132人目の素数さん:2017/09/18(月) 12:01:17.72 ID:k048EUpW.net
Monty Hallの問題の説明ですが、こういう説明が分かりやすいのではないでしょうか?

変更しなければ、明らかに、当たる確率は 1/3 である。

変更した場合の確率は、 1 - 1/3 = 2/3 である。

141 :132人目の素数さん:2017/09/18(月) 12:01:41.84 ID:k048EUpW.net
Monty Hallの問題の説明ですが、こういう説明が分かりやすいのではないでしょうか?

変更しなければ、明らかに、当たる確率は 1/3 である。

よって、変更した場合の確率は、 1 - 1/3 = 2/3 である。

142 :132人目の素数さん:2017/09/18(月) 12:02:41.47 ID:k048EUpW.net
これがベストな説明ですね。

143 :132人目の素数さん:2017/09/18(月) 12:29:04.93 ID:nZTNdNKW.net
∫[t:0→π/2](cos|t-x|)e^sin|t-x|dt (0≦x≦π) の方針だけでも教えてください

144 :132人目の素数さん:2017/09/18(月) 12:34:53.20 ID:ZbUVRqaj.net
>>143
どうせ質問答えてもらってもなんの返事もせず消えるんだろ

e^(sin(t-x))を微分してみろや

145 :132人目の素数さん:2017/09/18(月) 12:53:03.77 ID:ndtkiHmV.net
究極集合と絶対無限とオムニバースの大きい順を教えてください。
それとも、この三つは名前が違うだけで、中身は同じなのでしょうか?

146 :132人目の素数さん:2017/09/18(月) 12:54:05.96 ID:nZTNdNKW.net
>>144
e^sin(π/2-x)-e^sin(-x) まで出して
これの最小最大出したいんですけど
0≦x≦πなので場合分け必要ですよね?

147 :132人目の素数さん:2017/09/18(月) 14:23:09.32 ID:veZ8kvGP.net
>>140
松阪君〜!
なぜ明らかなのですかぁ?


明らか打なんて雑な説明をするなんて
アナタは大丈夫な一人なんですかぁ?
www

148 :132人目の素数さん:2017/09/18(月) 14:39:01.06 ID:cBM4SCe3.net
直感があてにならないという問の説明で「明らか」なんて感覚的な言葉を出しちゃう時点でモンティホールがどういう問題なのかがわかってない、論点がわかってないことであって説明としては0点

もちろんこんなことを考えずとも>>141は余事象すらわかってないようだから即0点

149 :132人目の素数さん:2017/09/18(月) 15:19:39.69 ID:qWK1GFhr.net
6面にそれぞれ1,1,2,2,9,9と書かれたさいころをn回投げて出た目の合計をXとします
このときnが十分に大きいとXはどのような分布に近づきますか?

150 :132人目の素数さん:2017/09/18(月) 16:27:54.47 ID:ZbUVRqaj.net
>>146
xが0〜π/2にあるなら、
積分区間を、0〜x、x〜π/2、と分ければ絶対値は外れる
もしxがπ/2〜πにあるなら、
即座に絶対値が外れる

あとは先程の微分の結果を使えばOK

151 :132人目の素数さん:2017/09/18(月) 17:01:49.79 ID:nZTNdNKW.net
>>150
できました
ありがとうございます

152 :132人目の素数さん:2017/09/18(月) 20:01:13.21 ID:ndtkiHmV.net
ピタゴラスとアリストテレスってどっちの方が頭いい?

153 :132人目の素数さん:2017/09/18(月) 20:15:24.42 ID:xTdf/qI4.net
>>152
神が頭がいいです

154 :132人目の素数さん:2017/09/18(月) 20:20:41.53 ID:ndtkiHmV.net
>>153
神と全はどっちの方が上ですか?

155 :132人目の素数さん:2017/09/18(月) 20:20:48.65 ID:k048EUpW.net
>>147

三つの箱の一つに当たりがあるので、明らかに当たる確率は 1/3 です。

156 :132人目の素数さん:2017/09/18(月) 20:22:02.14 ID:xTdf/qI4.net
>>154
神の方が上です

157 :132人目の素数さん:2017/09/18(月) 20:23:46.23 ID:k048EUpW.net
>>147

はずれの箱を開けて見せてもらおうがもらうまいが確率は変わりません。

158 :132人目の素数さん:2017/09/18(月) 20:24:36.40 ID:ndtkiHmV.net
>>156
神も全に含まれるわけだから、全の方が上でしょう。

159 :132人目の素数さん:2017/09/18(月) 20:25:06.82 ID:k048EUpW.net
そして、箱を変更しないときに当たる確率が 1/3 である以上、
変更したときの確率は 1 - 1/3 = 2/3 にならざるを得ません。

160 :132人目の素数さん:2017/09/18(月) 20:26:15.82 ID:k048EUpW.net
これがもっともすっきりとしたMonty Hall問題の説明ですね。

161 :132人目の素数さん:2017/09/18(月) 20:35:36.96 ID:xTdf/qI4.net
>>158
神は全の創造主なので上です

162 :132人目の素数さん:2017/09/18(月) 20:36:25.25 ID:ndtkiHmV.net
>>161
その神も当然全に含まれます。
つまり全の方が上です。

163 :132人目の素数さん:2017/09/18(月) 20:37:08.65 ID:h+QTi+CW.net
神さんマークの

164 :132人目の素数さん:2017/09/18(月) 21:02:02.76 ID:5C6o6yuo.net
>>127
n回目のコイン振りで表が出ると 1/2^n のポイントを与えることとする。
何回かコインを振り、ポイントの合計が a/b 以上になることが確定したら「表」
a/b 未満になることが確定したら「裏」と出力すればよい。

>>129
コインの表裏と、a/b の2進数表記10が一致する限り、コインを振り続けることになる。
1*(1/2)+2*(1/2^2)+3*(1/2^3)+...+n*(1/2^n)+...=(2^(n+1)-(n+2))/2^n → 2 (n→∞)

165 :132人目の素数さん:2017/09/18(月) 21:05:54.75 ID:8TT39P7a.net
場合の数について質問させて下さい。
1から6までのカードを、5枚取って並べる方法をこのように考えると間違いな仕組みが分かりません

1. 先ず6枚の中から2枚取る
6C2

2. 更に残りの4枚の中から3枚取る
6C2*4C3

3. 取った計5枚を並べる
6C2*4C3*5!=7200通り←不正解

宜しくお願い致します

166 :132人目の素数さん:2017/09/18(月) 21:37:18.74 ID:hC6SXTUk.net
>>165
(12)(345)=(13)(245)

167 :132人目の素数さん:2017/09/18(月) 22:05:00.55 ID:veZ8kvGP.net
>>157
だから変わらないのはなぜか?
って聞かれてるんだけど。

168 :132人目の素数さん:2017/09/18(月) 22:15:35.30 ID:8TT39P7a.net
>>166
ありがとうございます
なるほど!
答えは不正解を10で割った720通りですが、>>165の手順のまま修正を入れるとしたらどこをどう修正すべきでしょうか?

169 :132人目の素数さん:2017/09/18(月) 22:28:02.76 ID:8TT39P7a.net
>>166
つまり手順2で既に順列になっているということですよね?
6C2*4C3=60通り←不正解
となってしまいました

170 :132人目の素数さん:2017/09/18(月) 22:33:37.46 ID:8TT39P7a.net
>>166
解りました!
>>165を修正するとしたら

3. 1.と2.の各々の組み合わせを並べる
中から3枚取る
(6C2*2!)*(4C3*3!)=720通り←正解

テストが近いので本当に助かりました
ありがとうございました!

171 :132人目の素数さん:2017/09/19(火) 00:52:56.95 ID:73pkzlmT.net
油井亀美也さんと上杉謙信はどっちの方が凄いですか?

172 :132人目の素数さん:2017/09/19(火) 01:23:07.93 ID:RekoL3RN.net
神がすごいです

173 :132人目の素数さん:2017/09/19(火) 04:19:28.61 ID:SAZ57hNz.net
>>131 >>143 >>146

0 ≦ x ≦ π/2 のとき
e^{sin(x)} + e^{cos(x)} -2

π/2 ≦ x ≦ π のとき
e^{sin(x)}- e^{-cos(x)}

最大は x=π/4 のとき、2e^(1/√2)-2 = 2.0562300

最小は x=π のとき、1 - e = -1.7182818

174 :132人目の素数さん:2017/09/19(火) 14:28:36.09 ID:PoW4qd26.net
>>171
こいつはヒマラヤ(物理板のあらし)

>>172
こいつは屑神(物理板のあらし)

175 :132人目の素数さん:2017/09/19(火) 17:22:46.51 ID:OVYEjvCL.net
物理空間と情報空間はどっちの方が広いのでしょうか?

176 :132人目の素数さん:2017/09/19(火) 17:50:49.08 ID:RekoL3RN.net
神界が広いです

177 :132人目の素数さん:2017/09/19(火) 17:52:31.50 ID:OVYEjvCL.net
>>176
全と神界だと当然前者の方が広いですよね?
だって、神界も全に含まれるわけですから。

178 :132人目の素数さん:2017/09/19(火) 17:54:38.34 ID:RekoL3RN.net
神界のほうが広いですね

179 :132人目の素数さん:2017/09/19(火) 17:57:30.77 ID:OVYEjvCL.net
>>178
理由を教えてください。
全とは読んで字のごとく、「全て」なので、
当然神界も全に含まれるのです。
だから全の方が広いと思うのですが、違いますか?

180 :132人目の素数さん:2017/09/19(火) 18:04:26.78 ID:RekoL3RN.net
神神は無限な広がりを持つからです

181 :132人目の素数さん:2017/09/19(火) 18:05:17.27 ID:OVYEjvCL.net
その無限な広がりを持つ神界をも全は含んでいるのです。

182 :132人目の素数さん:2017/09/19(火) 20:19:32.53 ID:4RGKyOtl.net
>>164

正解です。

183 :132人目の素数さん:2017/09/19(火) 23:05:26.13 ID:Qeh4b+CS.net
https://i.imgur.com/WLHV9FN.jpg
解答
(1)D
(2)254.34
苦手なので解き方を教えて下さい。
宜しくお願いします。

184 :132人目の素数さん:2017/09/19(火) 23:08:17.17 ID:4RGKyOtl.net
微積分の本に、

Σa_n
Σb_n

が絶対収束するとし、

s = Σa_n
t = Σb_n

とする。

c_i = a_1*b_i + … + a_i*b_1

とする。

このとき、

c_1 + c_2 + c_3 + …

は絶対収束して、

s*t = c_1 + c_2 + c_3 + …

が成り立つ

と書いてあります。

185 :132人目の素数さん:2017/09/19(火) 23:11:43.82 ID:4RGKyOtl.net
s*t = a_1*b_1 + a_1*b_2 + a_2*b_2 + a_2*b_1 + a_1*b_3 + a_2*b_3 + a_3*b_3 + a_3*b_2 + a_3*b_1 …

と右辺を並べてもOKですよね。

なぜ、>>184の形に限定して書いてあるのでしょうか?

186 :132人目の素数さん:2017/09/19(火) 23:51:47.46 ID:7+T0Gh9a.net
>>117-119
遠隔だけど・・、
こういう質問(このa>0、b>0の「0」って三角形のどこの部分を示してるんですかね?)をするのは、女子高生かな〜?
薬学狙いとか、医学の女医狙いだと、多分理系の数学が必要なんでしょうね・・?

1.まず、ご参考: https://detail.chiebukuro.yahoo.co.jp/qa/question_detail/q1479960294 相加平均と相乗平均の大小関係に証明についてです この画像の証明をお願いします。jupiteremperorさん 知恵袋 yahoo 2012/1/23
2.”a>0、b>0”は、「非負条件」と言って、受験数学では頻出事項です。
   特にルート絡みのとき。√a,√b,関連で頻出。a or b が負になると、√(ab) or √a or √bが虚数になるなど、不等式としてまずいことになる*)。(*)注:そもそも「複素数では、大小は定義できない」と言われる。)

3.あと、昔は、代数的簡明な証明で教えられました。これを覚えておく方が、役に立つよ(^^
 1)相加平均>=相乗平均→(a+b)/2 >= √(ab) →(a+b) >= 2√(ab) と頭の中で変形して
   (a+b)− 2√(ab) >= 0 を証明する。
   A=√a,B=√bと置くと、A^2=a,B^2=bに注意すると、(a+b)=A^2+B^2, 2√(ab)=2AB であるから
   (a+b)− 2√(ab) =A~2+b~2 -2AB =(A - B)^2 >= 0 が成り立つ。(∵実数の平方は正又は0)
   等号成立は、A=B即ちa=bのとき。QED
 2)別解:
   左辺の二乗−右辺の二乗=(相加平均)^2 - (相乗平均)^2 を考える。
   {(a+b)/2}^2-{√(ab)}^2 ={(a^2+2ab+b^2)-4ab}/4 =(a^2-2ab+b^2)/4 ={(a-b)^2}/4 >=0 が成り立つ。(∵実数の平方は正又は0)
   これより、(a+b)/2 >=√(ab)が成り立つ。
   等号成立は、A=B即ちa=bのとき。QED

4.上記1)はちょっとした文字の置き換えで√を消すテクニック。計算がすっきりしている。2)の両辺の二乗の差を作って、√を消すテクニック。発想は素直。

5.余談だが、「あとa=bのときってなってますが全然同じ長さに見えないんですがどういうことなんでしょ」というのは、高一的発想だな?(^^
  受験数学では、(記述問題で)「等号成立条件は、書き漏らさないように気を付けろ!」が、”チャート式”発想です。(^^

以上

187 :132人目の素数さん:2017/09/19(火) 23:55:28.67 ID:7+T0Gh9a.net
>>186 訂正

   (a+b)− 2√(ab) =A~2+b~2 -2AB =(A - B)^2 >= 0 が成り立つ。
   ↓
   (a+b)− 2√(ab) =A^2+b^2 -2AB =(A - B)^2 >= 0 が成り立つ。

188 :132人目の素数さん:2017/09/19(火) 23:58:05.57 ID:7+T0Gh9a.net
>>186 訂正追加

2)別解:の方で、等号成立は、A=B即ちa=bのとき。→等号成立は、a=bのとき。

189 :132人目の素数さん:2017/09/19(火) 23:59:34.95 ID:DcUm2ZpE.net
ここの回答者って、簡単な問題だと既に回答が付いてて解決済みの問題にも長文回答つけるんですね

190 :132人目の素数さん:2017/09/20(水) 00:08:58.55 ID:dfpPaMnE.net
>>186 追加

中高一貫またはスーパー高向けに、下記
http://integers.hatenablog.com/entry/2016/04/30/231637
2016-04-30 相加相乗平均の不等式の内田康晴氏による証明の解説 INTEGERS

数々の熱いプレゼンの中、蓑田恭秀氏のプレゼン『意外と深い「平均」の世界』を聞いて大変興味をもったのが、「2008年、高校教師である内田康晴氏が相加相乗平均の不等式の新証明を発見し、それがオーストラリアの研究誌に出版され、日本でもニュースとして取り扱われた」というものです。

帰宅して、この証明が気になって論文を読んだので解説記事を書きます。

http://www.sqr.or.jp/usr/haru/
直線上に配置 内田 康晴
http://www.sqr.or.jp/usr/haru/websitemodel/rezume3.pdf
相加・相乗平均不等式の証明図と新しい一般証明 そして一般証明 内田 康晴

191 :132人目の素数さん:2017/09/20(水) 00:09:39.61 ID:dfpPaMnE.net
>>189
まあな(^^

192 :132人目の素数さん:2017/09/20(水) 16:01:02.42 ID:kYch4U5Y.net
成分表示されていないベクトルの外積って求められないのですか?

193 :132人目の素数さん:2017/09/20(水) 16:52:05.95 ID:PJFWTJM0.net
どうすれば求めたことになるのか聞こうか

194 :132人目の素数さん:2017/09/20(水) 17:22:04.12 ID:ZCVYN7gB.net
成分表示された2つのベクトルの外積ベクトルは簡単に求められるのに
成分表示されていない場合は容易には求められないのですか?
内積の場合は、成分表示か否かはあまり関係ないのに

195 :132人目の素数さん:2017/09/20(水) 17:30:05.01 ID:a67X4UeK.net
成分表示を経由せず外積を定義できるかという質問なら、答えは「できる」

では「求める」とは何か
「計算する」という意味だとしたら、成分表示を用いず計算するとはどういうことか

196 :132人目の素数さん:2017/09/20(水) 17:36:26.55 ID:WN5WbPCj.net
大文字はベクトルを表すものとして

U=aX+bY+cZ
V=dX+eY+fZ
とおいたときに、
U×VをX,Y,Zを用いて表せますか?という質問です

197 :132人目の素数さん:2017/09/20(水) 17:37:02.51 ID:WN5WbPCj.net
先ほどからIDが変わってしまってすみません

198 :132人目の素数さん:2017/09/20(水) 17:40:36.89 ID:NMWCyTB9.net
X,Y,Zとはなんですか?

199 :132人目の素数さん:2017/09/20(水) 17:51:21.36 ID:WN5WbPCj.net
線形独立なベクトルです・・・

200 :132人目の素数さん:2017/09/20(水) 17:57:39.25 ID:NMWCyTB9.net
分配法則を使って
aeX×Y+afX×Z+...
とするのは、あなたの欲しい答えになりますか?

201 :132人目の素数さん:2017/09/20(水) 19:17:56.70 ID:qFYnRMWZ.net
大仏と東大生はどっちの方が凄いですか?

202 :132人目の素数さん:2017/09/20(水) 19:19:30.13 ID:NMWCyTB9.net
神がすごいです

203 :132人目の素数さん:2017/09/20(水) 19:20:35.49 ID:V0NS52T1.net
巨大仏

204 :132人目の素数さん:2017/09/20(水) 20:14:30.94 ID:hPTk+wNe.net
>>200
ありがとうございます
X,Y,Zそれぞれ同士の外積が必要なんですね
低レベルな質問を何度も書いてしまい申し訳ありませんでした

205 :132人目の素数さん:2017/09/20(水) 21:23:58.25 ID:uO4bUoI4.net
確率変数Xの確率密度関数が、1(0<=X<=1)、0(その他)のとき
Xと同様の確率密度関数を持つ互いに独立な確率変数X1,X2,X3を使って
確率変数Z=X1+X2+X3とするとき
Zの確率密度関数はどうなりますか?

206 :痴呆大学:2017/09/20(水) 21:31:11.71 ID:y+wE1/bA.net
∬P(x1)P(x2)P(z-x1-x2)dx1dx2 over 0<x1+x2<z

207 :205:2017/09/20(水) 22:07:55.62 ID:uO4bUoI4.net
期待値が3/2
分散が3/12になるのはわかります
zだけ1変数で表すとどうなりますか

208 :132人目の素数さん:2017/09/20(水) 22:09:49.50 ID:XKoEcctn.net
sin(x)=x-1/6x^3+O(x^5)より1/sin(x)=1/x(1+1/6x^2+O(x^4))
と解答に書いてあるのですが導きかたが分かりません。よろしくお願いします。

209 :132人目の素数さん:2017/09/20(水) 22:37:45.27 ID:qFYnRMWZ.net
超天才数学者と超天才プログラマーはどっちの方が凄いですか?

210 :132人目の素数さん:2017/09/20(水) 22:40:50.54 ID:NMWCyTB9.net
神がすごいです

211 :132人目の素数さん:2017/09/20(水) 22:49:01.82 ID:xLGAaIPA.net
>>185
並べ方なんてそれこそ無数に存在するのに
あらゆる並べ方について説明してないと
難癖つけられるなんて著者はたまったもん
じゃね〜な

つーか、やっぱ基地外なんだな

212 :132人目の素数さん:2017/09/20(水) 22:55:38.08 ID:PJFWTJM0.net
Σ使えばその形が一番書きやすいと思うけど、どうやら松阪君はそうではないようだ

というか、ちょっとでもオリジナリティを出したら「この説明は標準的じゃないですよね。〇〇と書くのが普通ではないでしょうか?この著者は(ry」とか言いますやん君

213 :132人目の素数さん:2017/09/20(水) 22:57:23.05 ID:qFYnRMWZ.net
全=無=永遠=神

なのでしょうか?

214 :132人目の素数さん:2017/09/20(水) 22:58:30.18 ID:V0NS52T1.net
ここはドラゴンボールスレ

215 :132人目の素数さん:2017/09/21(木) 00:47:21.48 ID:1RUtzMNA.net
>>208
sin(x) = x(1 - (x^2)/6 + O(x^4))

逆数とって

1/sin(x) = (1/x)*1/(1 - (x^2)/6 + O(x^4))
= (1/x)*(1 + (x^2)/6 + O(x^4))

ちなみに

1/(1 + ax + O(x^2)) = 1 - ax + O(x^2)

だからな。

216 :132人目の素数さん:2017/09/21(木) 03:05:56.05 ID:4NzsMA+y.net
以下の問題が分かりません。「表せない」の条件の方にexp(-x^2)など有名な関数を入れてみましたが、見当がつきません。

【問題】
∫(0→t) f(x) dx は初等関数の四則演算及びべき乗では表せないが、
∫(0→t) f(x)f(t-x) dx は初等関数の四則演算及びべき乗で表せる

そのようなf(x)の例を一例挙げよ。
ただし ∫(0→t) f(x) dx が初等関数の四則演算及びべき乗で表せないことの証明はしなくてよい。

217 :132人目の素数さん:2017/09/21(木) 04:12:05.04 ID:cwkL2As+.net
f(x)=1/logx

218 :132人目の素数さん:2017/09/21(木) 05:16:28.92 ID:qNWCrw4p.net
A[x1,x2,...,xn]:多項式環
p:その多項式
B:環
とすると
{準同型:A[x1,x2,...,xn]/(p) → B}

{p(b1,b2,...,bn)=0を満たす(b1,b2,...,bn)∈B^n}
に一対一対応があることの証明を教えてください
またこれについて詳しく書いてある本があれば教えてください

219 :132人目の素数さん:2017/09/21(木) 09:05:45.35 ID:eSPYSucK.net
>>215
ありがとうございます。
1/(1 + ax + O(x^2)) = 1 - ax + O(x^2) である理由を詳しく書くと、
1/(1+X)=1-X+O(X^2)
Xにax+O(x^2)を代入して、1/(1+ax+O(x^2))=1-ax+O(x^2)+O((ax+O(x^2))^2)
f(x)=O(g(x)),g(x)=O(x^n)ならばf(x)=O(x^n)である。
これを使って、1/(1+ax+O(x^2))-(1-ax)=O((ax+O(x^2))^2),(ax+O(x^2))^2=a^2x^2+2axO(x^2)+O(x^2)^2=O(x^2)より
1/(1+ax+O(x^2))-(1-ax)=O(x^2)
1/(1+ax+O(x^2))=(1-ax)+O(x^2)
ということですか?

こういうオーダーを含む計算はその場その場で思いついて工夫しなければいけないのですか?

220 :132人目の素数さん:2017/09/21(木) 09:06:25.42 ID:gBRLuwvS.net
このままでは、もろもろ前提の記述が足りないようだ。

221 :132人目の素数さん:2017/09/21(木) 10:26:28.68 ID:eSPYSucK.net
オーダーの計算について詳しく書いてある本はありますでしょうか?

222 :132人目の素数さん:2017/09/21(木) 10:52:42.49 ID:eSPYSucK.net
O(x^n)⇒o(x^(n+1))は成り立ちますが、o(x^n)⇒O(x^(n+1))は成り立ちますか?

223 :132人目の素数さん:2017/09/21(木) 11:38:47.01 ID:6y56Z5gn.net
>>219
そんなに形にとらわれて計算しなくても…

1/(1 + X)
= 1 - X + X^2 - X^3 + X^4 - X^5 + …
= 1 - X + O(X^2)

ってことだし、

X = ax + O(x^2)

は、具体的に書くと

X = ax + Bx^2 + Cx^3 + …

ってことだから、

1/(1 + X)
= 1 - (ax + Bx^2 + Cx^3 + …)
+ (ax + Bx^2 + Cx^3+ …)^2
- (ax + Bx^2 + Cx^3 + …)^3
+ …

なんだが、ほとんどの項は x^2 以上の
高次の項だから

1/(1 + X)
= 1 - ax + O(x^2)

というわけだよ

224 :132人目の素数さん:2017/09/21(木) 11:39:18.53 ID:6y56Z5gn.net
>>222
成り立たない

225 :132人目の素数さん:2017/09/21(木) 11:47:39.66 ID:eSPYSucK.net
>>223-224
ありがとうございます。
f(x) = O(x^2)ならばf(x)=Bx^2 + Cx^3 + …とかならず書けるのでしょうか?

226 :132人目の素数さん:2017/09/21(木) 12:20:26.79 ID:qsDFDKvR.net
>>216

指数関数・対数関数・逆三角関数も「初等関数」に含めるなら
 f(x)= e^(ax)/(多項式)   (a≠0)
でどう?
  
〔例〕
 f(x)= e^(ax)/(x+b)  (a≠0、b>0)
のときは
 ∫(0→t)f(x)f(t-x)dx = 2e^(at)log((1+t/b)/(t+2b),

227 :132人目の素数さん:2017/09/21(木) 12:48:57.24 ID:BXixu0jW.net
>>222
x→0だったらどっちも成り立たないしx→∞だったらどっちも成り立つ

228 :132人目の素数さん:2017/09/21(木) 12:49:10.47 ID:LsrixT9U.net
はじめまして。
箱の中にびんを詰めようと思うのですが、どちらが多く入るのか知りたくて質問させていただきました。
びんの直径は9cmです。購入しようと思っている箱は39cm角と42cm角のに種類があります。
文系脳のわたしには、どちらも4本かける4本の16本しか入らないような気がするのですが
もしかしてくぼみに次の段を入れたらもう一段入るかもしれないと思い質問させていただきました。

たとえば下から 4本、3本、4本、3本、4本 と入れたら18本入りそうです。
こういうのは数学で計算できますか?

229 :132人目の素数さん:2017/09/21(木) 12:51:54.17 ID:LsrixT9U.net
もしかして 42-9☓4 は 6(半径より大きい)だから

○○○○
○○○○
○○○○
○○○○
○○○○

というように4本を5段もできるでしょうか?

230 :132人目の素数さん:2017/09/21(木) 12:53:08.66 ID:LsrixT9U.net
orz 絵を失敗しました…これで思う表示になるかな

○○○○
&nbsp;○○○○
○○○○
&nbsp;○○○○
○○○○

231 :132人目の素数さん:2017/09/21(木) 12:53:51.72 ID:LsrixT9U.net
orz orz だめじゃん、わたし…
絵で表現できないのでお察しください…

232 :132人目の素数さん:2017/09/21(木) 13:26:16.97 ID:BjBw5Tqa.net
こういう積み方を言ってるなら
http://www.jewelry-note.com/narabi.jpg

各円の中心を結ぶとこうなるのを利用して
http://www.ho-yu.ed.jp/material/img/uploads/39/files/2013/05/fig12.png

1段目の中心:9/2 cm
2段目の中心:9/2+(9√3)/2 cm
3段目の中心:9/2+9√3 cm
4段目の中心:9/2+(27√3)/2 cm
5段目の中心:9/2+18√3 cm
5段目の上端:9+18√3 cm = 40.1… cm

1,2段目を合わせた幅:9*4+9/2 cm = 40.5 cm

42角の方を買えば4個×5段積める

233 :132人目の素数さん:2017/09/21(木) 13:31:04.70 ID:BjBw5Tqa.net
まあ、上に積み上げていく場合、幅42cmだと端の瓶と壁の間に隙間が出来るからガタガタになる

234 :132人目の素数さん:2017/09/21(木) 13:40:05.03 ID:BXixu0jW.net
緩衝材なりを詰めれば済む話

235 :132人目の素数さん:2017/09/21(木) 14:28:58.15 ID:m+wQ2Z/z.net
「次の数列の一般項を求めよ」[1,2,6,15,31,56, ・・・]

Bn=1,4,9,16,25...
Cn=3,5,7,9 ....

Σの計算方法がいまいちのみこめてないんでこの先お手上げ
どっか猫でもわかるように書いてるサイトないですかね・・・

あとすいません、今回だけ答え教えてください

236 :132人目の素数さん:2017/09/21(木) 14:45:15.71 ID:qsDFDKvR.net
>>235

階差数列を出してみる。

Dn = 2,
Cn = 2n+1,
Bn = nn,
An = n(n+1)(2n+1)/6 + 1,

今回だけな。

237 :235:2017/09/21(木) 14:50:24.77 ID:m+wQ2Z/z.net
>236
ありがとうございます

238 :132人目の素数さん:2017/09/21(木) 14:54:05.29 ID:yY4X6Mgj.net
>>236
間違ってんぞ

CnはBnの階差数列をとったんだろうが、
Bn=n^2
だからCnは必要ない。

求める数列を{An}、その階差数列{Bn}とおく。
A1=1
n>=2のとき、
An=A1+Σ(k=1,n-1)Bk
=1+(n-1)n(2n-1)/6
=(2n^3-3n^2+n+6)/6 (これはn=1でも成立。)

Σの計算方法というよりは公式覚えるだけ。
今回使ったのは
Σ(k=1,n)k^2=n(n+1)(2n+1)/6
今回はnのところにn-1を代入した。

239 :132人目の素数さん:2017/09/21(木) 14:56:04.97 ID:yY4X6Mgj.net
Σの公式について詳しくはここへ
http://examist.jp/mathematics/sequence/sigma-kousiki/

240 :132人目の素数さん:2017/09/21(木) 15:02:14.47 ID:M5FWDIYe.net
横から失礼します。
連立1次不等式の問題で、よくある型なのですがa<b<cという形に関してなのですが
この類の問題を解く際、a<bとb<cに分けて解くという記述が多いのですが、どうしてこれでよいのか、またなぜこれが正しいのかいまいちわかりません。
例題では、「次の連立不等式を満たすxの値の範囲を求めよ 3x-7<x-1≦-x+3 」という形だったので
考えに当てはめれば3x-7<x-1とx-1≦-x+3に分けるのでしょうが、やはり理由がよくわかりません。
例題を使わずとも説明できるのであるのならそれでも構いません。このやり方が使える理由と、それがなぜ正しいかの説明をお願いします。

241 :132人目の素数さん:2017/09/21(木) 15:07:55.74 ID:yY4X6Mgj.net
a<b<c の必要十分条件が
a<b かつ b<c だから

a<b<c⇔a<bかつb<c

a<bとb<cが成り立っているとき、a<b<cが成り立つことに文句ありますか?
ありませんよね

242 :132人目の素数さん:2017/09/21(木) 15:13:15.80 ID:M5FWDIYe.net
>>241
返信ありがとうございます。

少し前提条件を忘れていたようですね、ありがとうございます。
どういう状態なのかがイメージが湧かない感じで少し分からない状態です。
どうかもう少しだけかみ砕いていただけませんでしょうか。

243 :132人目の素数さん:2017/09/21(木) 15:27:31.71 ID:M5FWDIYe.net
長い時間考えていただいているところ申し訳ないのですが、何とか自力で理解でき、納得出来ました
返答もまだで大変恐縮なのですがこれにて質問を打ち切らせていただきます。>>241さんありがとうございました。

244 :132人目の素数さん:2017/09/21(木) 15:33:17.69 ID:yY4X6Mgj.net
a<b<c と
a<bかつb<c
が同値になることがいまいちつかめないということであってますか?

んーー
一度に二人しか走れない50mのレーンがあると想像してください。
さらにストップウォッチもありません。
この状況で、A君B君C君の三人の足の速さについて、
「Aが一番遅くて、Cが一番速いことを確認して」と言われたらどうしますか?
AとBを競争させませんか?次にBとCを競争させませんか?

245 :132人目の素数さん:2017/09/21(木) 15:38:47.24 ID:M5FWDIYe.net
>>244
あ、失礼しました。自分堪え性が無いもので

いえ、そこは何とか理解しております
イメージと言いましょうか、少し抽象的な物で申し訳ないのですがそれが分らなかった次第です
今は何とか納得に漕ぎつけましたので、重ねてありがとうございました。それとお手数おかけしました。

246 :math:2017/09/21(木) 17:26:56.76 ID:ocNtw4xm.net
Xについての方程式 |x^2-4|=k の実数解が4個となるようにkの範囲を定めよ。


の解答をできるだけ詳しくお願いします。
特に絶対値に関して曖昧な所があります。
よろしくお願いします。

247 :132人目の素数さん:2017/09/21(木) 17:45:55.26 ID:3r33gBcc.net
>>246
y=左辺のグラフを描いて、y=kとの交点が4つになるkを探しましょう

248 :132人目の素数さん:2017/09/21(木) 17:58:08.50 ID:rcmVlakk.net
>>218
お願いします

249 :132人目の素数さん:2017/09/21(木) 18:10:31.95 ID:EgAl7EZK.net
>>227
成り立たないだろ

250 :132人目の素数さん:2017/09/21(木) 18:39:32.34 ID:QtDNaKyz.net
10個の(うちの)全て

と、

10個は全て

この二つはどう違いますか?

251 :132人目の素数さん:2017/09/21(木) 19:15:02.03 ID:a0gyYTa1.net
>>240
>a<bとb<cに分けて解く
???

252 :228:2017/09/21(木) 19:21:47.81 ID:aWhgKQHO.net
>>232さん!ありがとうございます!

まさにそういうことを知りたかったんです!
途中の計算が全然わかりませんが、42cmのほうを買おうと思います。
本当にありがとうございました。

253 :132人目の素数さん:2017/09/21(木) 19:26:59.10 ID:QtDNaKyz.net
尋常じゃないくらい頭が悪いけど、東京大学理学部数学科に入りたい。
やっぱり無理かな・・・?

254 :132人目の素数さん:2017/09/21(木) 19:51:02.10 ID:Qg/ZIZM6.net
頭悪いなら無理でしょ

255 :132人目の素数さん:2017/09/21(木) 20:00:02.78 ID:M5FWDIYe.net
>>251
書き方が悪かったですね、連立不等式a<b<cを

{a<b
b<c

という感じに分けて考えるとよい、と書かれていたのです

256 :132人目の素数さん:2017/09/21(木) 20:04:44.38 ID:a0gyYTa1.net
0<x<y<1
を図に描くくにはどうする?

257 :132人目の素数さん:2017/09/21(木) 22:19:40.68 ID:WG0bDX0K.net
>>256
0<x, x<y, y<1
の三つの領域を考えて、あとは共通部分でおk

結局、0<x<1かつ0<y<1が表す正方形をy=xで切った左上部分

258 :132人目の素数さん:2017/09/21(木) 23:34:18.33 ID:2PN2CMQP.net
>>249
o(x^n)⇒O(x^n)だろ?

259 :132人目の素数さん:2017/09/21(木) 23:46:34.55 ID:R0g+kZqd.net
マンコベクトルとチンコベクトルは直交するのですか?

260 :132人目の素数さん:2017/09/22(金) 00:00:03.23 ID:okMDmmws.net
直交したら入らないよ

261 :SSS Sexually Skebe Sannkonn:2017/09/22(金) 00:18:35.90 ID:YYeemta+.net
|マンコベクトルチン|^2 - |チンココベクトル|^2==0 のセイコウダイスキー空間では入ります ハイルヒトラー

262 :132人目の素数さん:2017/09/22(金) 02:06:49.67 ID:LNCPjhUN.net
>>225
必ず書けるわけではないな

f(x) = Bx^2 + C(x^2)log x

みたいなこ可能性もあるから

263 :132人目の素数さん:2017/09/22(金) 02:07:15.62 ID:LNCPjhUN.net
>>258


264 :132人目の素数さん:2017/09/22(金) 11:50:20.78 ID:2kfJtelJ.net
今年の東大第二問の(1)が5/16と分かった上で(2)をその二乗とするのは勝手に独立だとしていることにあたらないのでしょうか
独立だと証明するには独立であることを前提とせず確率を求めてそれが(5/16)^2だと示すべきですがそれはもはや答えそのものでありわざわざ独立であると宣言して二乗する意味はありません
ちなみに本番の採点としては減点はなかったことが分かっています

265 :132人目の素数さん:2017/09/22(金) 11:52:14.89 ID:dxvc1idi.net
>>246 >>247

k<0 のとき 0個
k=0 のとき 2個{±2(重根)}
0<k<4 のとき 4個{±√(4-k),±√(4+k)}
k=4 のとき 3個{0(重根),±√8}
k>4 のとき 2個{±√(4+k)}

266 :132人目の素数さん:2017/09/22(金) 11:58:40.82 ID:1BMCcREk.net
問題も貼れんのか

267 :132人目の素数さん:2017/09/22(金) 12:15:31.77 ID:FHJ8dFwy.net
>>223

こういうインフォーマルな答案を書いた場合、0点でしょうか?

268 :132人目の素数さん:2017/09/22(金) 12:27:16.87 ID:FHJ8dFwy.net
>>219

こういう類の計算は、↓の本にたくさんありますね。

詳説演習微分積分学 塹江 誠夫 詳説演習微分積分学
塹江 誠夫
固定リンク: http://amzn.asia/3KUeLwC

269 :132人目の素数さん:2017/09/22(金) 12:34:32.23 ID:hZCqVtJ0.net
>>268
かんたんな微積分の本には詳しいんですね

270 :132人目の素数さん:2017/09/22(金) 12:36:37.41 ID:FHJ8dFwy.net
>>268

でも、この問題集、間違いだらけですね。

271 :132人目の素数さん:2017/09/22(金) 12:45:08.64 ID:hZCqVtJ0.net
>>270
誤植一覧をまとめてください

272 :132人目の素数さん:2017/09/22(金) 13:04:44.02 ID:2kfJtelJ.net
>>266
こんな誰でも間違いなく検索できるものを貼る必要が分かりませんね
あなたのそのレスの時間で検索できたのでは?
ちなみに僕はこのレスをする間に貼ることは出来ますがあなたが鬱陶しいのでやりません
よってお前こそという返しは通じない

273 :132人目の素数さん:2017/09/22(金) 13:13:48.95 ID:hZCqVtJ0.net
>>272
今年はダメだったんですか?

274 :132人目の素数さん:2017/09/22(金) 18:41:43.63 ID:FHJ8dFwy.net
藤原松三郎著『微分積分学第1巻』を読んでいます。

ちょっと他の本で見かけない定理が書いてありますね:


定理6

u_n^(k) → u^(k) (n → ∞)
u^(k) → u_0 (k → ∞)

ならば、

n_1, n_2, ... を適当にとれば

u_(n_k)^(k) → u_0

が成立する。

275 :132人目の素数さん:2017/09/22(金) 19:35:45.16 ID:hZCqVtJ0.net
>>274
微積分以外の本は読まないんですか?

276 :132人目の素数さん:2017/09/22(金) 20:05:44.13 ID:wp92pdUZ.net
>>275
読まないんじゃない
読めないんだ!

277 :132人目の素数さん:2017/09/22(金) 21:21:34.52 ID:Cqcoaw4N.net
「最強」の候補

全、無、考えない、どうなってもいい、痛む、痛みに耐える、想像すらできない
なんでもあり、自由自在、不定、観測者不在

一体どれが最強なのか?

278 :132人目の素数さん:2017/09/22(金) 22:29:48.09 ID:ZVDwmYDH.net
Aが1/3、Bが2/3で出るサイコロがあり、3回振るとBが2回以上出る確率が38%位でした。
このように確率が偏ったAとBをn回振るとAが過半数出る確率を求める公式はあるのでしょうか。
nが増えると確率は低いほうが上がっていくのでしょうか。

279 :132人目の素数さん:2017/09/23(土) 07:38:23.65 ID:Qt+O1r6N.net
>>278
> Aが1/3、Bが2/3で出るサイコロがあり、
この時点で確率わかってるじゃん

280 :132人目の素数さん:2017/09/23(土) 07:52:04.05 ID:ArV565v9.net
nをどんどん増やしたら1/3しかAは出ないんだからAが過半数出る確率はどんどん0に収束していく

281 :132人目の素数さん:2017/09/23(土) 12:28:10.92 ID:1ewDozZc.net
全は最強ですか?

282 :132人目の素数さん:2017/09/23(土) 12:35:35.08 ID:0BXfuq/u.net
ヘマラヤのうんこ

283 :132人目の素数さん:2017/09/23(土) 12:43:01.46 ID:s732VrAG.net
>>186
うひゃーなんかすごい人に安価つけて貰えたどうもありがとう

284 :132人目の素数さん:2017/09/23(土) 13:41:24.44 ID:9CUu3vwk.net
すみませんが、以下の問題についてご支援頂けないでしょうか。

くじ引きにおいて、全部で100個しか入っておらず、それぞれの玉には1等から100等まで設定されている。
1度の試行回数で11個の玉をランダムに抽選し、仮に2度目以降も試行する場合、1度引いた玉は全て戻されて引き戻しとなる。
ただし、11個の玉の中に1個も1等〜14等までが入ってなかった場合は、必ずランダムに1等から14等までのどれか1つが当たるようプログラムされている。

1) 1等から10等までを、なるべく被らずに多く集めたい場合、期待値が半数の5を超えるのは試行回数何回目であるか。
2) 1等から8等までを、なるべく被らずに多く集めたい場合、期待値が半数の4を超えるのは試行回数何回目であるか。

どうぞよろしくお願いいたします。

285 :132人目の素数さん:2017/09/23(土) 13:46:14.96 ID:6eUADSdx.net
>>118
発見されたというのはちょっとどうかなあって思うけどね
書かれたのが最初とかじゃなくて?
それも最初かどうか分からんなあとも思うが

286 :132人目の素数さん:2017/09/23(土) 13:50:22.23 ID:pytwCpHO.net
>>186
正確には、複素数には大小(というか順序)
が定義できないわけではなくて、
使い勝手のよい意義のある順序が
定義できないってことだな。

287 :132人目の素数さん:2017/09/23(土) 16:02:23.24 ID:l6/M8Xfh.net
順序体にならないと言えばいいのに

288 :132人目の素数さん:2017/09/23(土) 17:19:30.96 ID:z681e0lu.net
1 / (g1(t) + i*g2(t))

を微分すると

-(g1'(t) + i*g2'(t)) / (g1(t) + i*g2(t))^2

になることを証明せよ。

289 :132人目の素数さん:2017/09/23(土) 18:06:45.62 ID:mDWslwyr.net
全(全て)に勝るものはありますか?

290 :132人目の素数さん:2017/09/23(土) 18:15:08.12 ID:956M/8NA.net
ヘマラヤのうんこ

291 :132人目の素数さん:2017/09/23(土) 18:28:48.30 ID:s2W1J+Wd.net
>>284
ココナラで聞いてみたら?
下のとか
https://coconala.com/services/317502?pos=13

292 :132人目の素数さん:2017/09/23(土) 18:39:25.93 ID:z681e0lu.net
>>288

R → C の関数でも R → R の関数と同じ公式が
成り立つのは単なる偶然でしょうか?

293 :132人目の素数さん:2017/09/23(土) 18:57:24.58 ID:pytwCpHO.net
>>292
そんなの定義と証明見たら明らかじゃん

294 :132人目の素数さん:2017/09/23(土) 18:57:53.46 ID:pytwCpHO.net
>>287
そうだね
ごめんよ

295 :132人目の素数さん:2017/09/23(土) 19:06:36.11 ID:z681e0lu.net
>>293

明らかではないのではないでしょうか?

296 :132人目の素数さん:2017/09/23(土) 19:35:42.67 ID:gf6WjL+P.net
そう思うなら導関数の定義に従って計算すればいい
途中で「なにやってんだ俺wwwww」と思わないなら、なんというか…

297 :132人目の素数さん:2017/09/23(土) 19:37:49.47 ID:5V9HvDIL.net
>>280
>>279
1回だと33%なのに3回振るとAが2回以上出る確率が38%にあがるのが不思議でした。

298 :132人目の素数さん:2017/09/23(土) 20:15:58.31 ID:z681e0lu.net
http://takeno.iee.niit.ac.jp/~shige/math/lecture/basic3/quotef4/node8.html

>>288

はかなり面倒な計算が必要なのではないでしょうか?

299 :132人目の素数さん:2017/09/23(土) 20:30:24.22 ID:mDWslwyr.net
究極集合より上は無いですか?

300 :132人目の素数さん:2017/09/23(土) 20:45:24.53 ID:ArV565v9.net
>>297
> 3回振るとAが2回以上出る確率が38%
本当に?

301 :132人目の素数さん:2017/09/23(土) 21:08:43.69 ID:6eUADSdx.net
>>295
明朗

302 :132人目の素数さん:2017/09/23(土) 21:10:03.81 ID:6eUADSdx.net
>>298
全然

303 :132人目の素数さん:2017/09/23(土) 21:20:54.40 ID:x/fcotj5.net
サイダックの証明とかも絶対誰か思いついてただろ

304 :132人目の素数さん:2017/09/23(土) 22:36:46.05 ID:QI9ouw+g.net
>>274
シュワルツ超関数とかでは普通に使う議論で
そっち方面の本には必ず載ってるという意味で珍しくない

305 :132人目の素数さん:2017/09/23(土) 23:12:00.00 ID:l6/M8Xfh.net
>>298
ライプニッツ則示せば十分
ライプニッツ則満たすことは定義から明らか

306 :132人目の素数さん:2017/09/24(日) 00:36:50.69 ID:5uOx8mHE.net
質問です。

104人で7回戦のじゃんけん大会をします。
試合の進め方はスイスドロー形式です(2回戦以降は成績が近い人同士あたる方式)
7回戦終了時点で5勝2敗以上の人の合計人数、4勝3敗の人数を教えて下さい


※3回戦終了時点で全勝者は13人になるので4回戦で1人だけ2勝1敗の人と当たりますが、その人は負けてしまうと仮定します
※6回戦終了時の全勝者は2人で仮定します

307 :132人目の素数さん:2017/09/24(日) 02:28:11.67 ID:BBiHOutO.net
以下の中で一番強いのはどれだと思いますか?

全(全て)、無、どうなってもいい、考えない、痛む、痛みに耐える、なんでもあり
観測者不在、不定、自由自在、想像することすらできない、感じることすらできない

308 :132人目の素数さん:2017/09/24(日) 03:44:58.74 ID:xX0vyKyK.net
やさしく学べる線形代数という本で
対角化するときの固有値を求める
固有方程式を解くときの変形で
わからないので教えてください。

| x-2 1 1 |
| 1 x-2 1 |
| 1 1 x-2 |
が、
| x 1 1 |
| x x-2 1 |
| x 1 x-2 |
となっています。
1'+2'×1 と 1'+3'×1 と補足があるので
余韻子で操作していると思ったのですが
それでもわかりませんでした。

309 :132人目の素数さん:2017/09/24(日) 05:04:17.04 ID:JigSynkR.net
1行目に2行目と3行目足して転置取ったんでしょ

310 :132人目の素数さん:2017/09/24(日) 06:58:03.68 ID:K2Xyf521.net
>>305

ライプニッツのほうは明らかですが、

>>298

は明らかではないのではないでしょうか?

311 :132人目の素数さん:2017/09/24(日) 08:28:51.99 ID:bpP7J28B.net
>>298
そのサイトが面倒なことをしてるだけ

312 :132人目の素数さん:2017/09/24(日) 08:49:57.21 ID:d8rTCDSk.net
>>310
これが明らかな程度は
実数関数も複素関数も同じ

313 :132人目の素数さん:2017/09/24(日) 09:12:57.08 ID:AsS5CEHB.net
>>310
ライプニッツ則と商の微分は同じものだろ?

314 :132人目の素数さん:2017/09/24(日) 10:49:45.44 ID:mM7D47QN.net
(適当な条件下で)商の導関数の存在を前提にすれば Leibniz rule からすぐに出るのはわかるけど、
存在は自明か?

315 :132人目の素数さん:2017/09/24(日) 11:24:36.28 ID:K2Xyf521.net
>>304

ありがとうございます。

一応、後にこの定理を利用しています。

316 :132人目の素数さん:2017/09/24(日) 11:31:20.08 ID:xX0vyKyK.net
>>309
ありがとうございます!
もう一題同様のものがあったのですが
そたらも解くことができ理解できました。

317 :132人目の素数さん:2017/09/24(日) 13:23:29.91 ID:+GWHsYBd.net
>>267

たぶん零点だけど、もしかしたら特異点。

318 :132人目の素数さん:2017/09/24(日) 13:37:25.37 ID:hINKYO0/.net
>>307
神です

319 :132人目の素数さん:2017/09/24(日) 13:40:34.94 ID:K2Xyf521.net
f, g を [a, b] で連続な関数とする。
f は [a, b] で正かつ単調減少関数とする。

このとき、

∫ f(x) * g(x) dx from x = a to x = b

=

f(a) * ∫g(x) dx from x = a to x = c

となるような c ∈ [a, b] が存在することを証明せよ。

320 :132人目の素数さん:2017/09/24(日) 13:47:27.14 ID:K2Xyf521.net
>>319

まずは、 f が [a, b] で C^1 である場合について考えよ。

次に、一般の場合を考えよ。

321 :132人目の素数さん:2017/09/24(日) 15:50:31.01 ID:tyaHvJ7j.net
>>284
30回11連ガチャを引くのを100回繰り返すのを
Excelで実験した結果を統計処理すると
1)も、2)も 6回だった

322 :132人目の素数さん:2017/09/24(日) 16:20:46.21 ID:8zu9HXsf.net
次の2次関数の最大、最小を求めよ。
また、その時のxの値を求めよ。
1. y=-2x^2+8x+3

2. y=2x^2-4x+3 (-2≦x≦2)

基礎のような問題ですが、お願いします。

323 :132人目の素数さん:2017/09/24(日) 20:13:00.66 ID:d8rTCDSk.net
>>314
それが自明な程度は実数関数と同じ

324 :132人目の素数さん:2017/09/24(日) 21:24:40.12 ID:K2Xyf521.net
赤攝也著『実数論講義』に

どのような物 a, b, c に対しても、 b ≠ c ならば

{a} ≠ {b, c}

である。

などという補題を1ページちょっと使って証明しています。

これは正気の沙汰でしょうか?

もし、こんなことまで証明するというなら、この本の中で他にも証明しないで使っている
様々なことも証明しなければならないのではないでしょうか?

赤さんは、「はじめに」に、

「本書は、これまで誰も言わなかった大事な事柄をたくさん含んでいる。それらが普及すれば、
微分積分学がガラリと体裁を変える可能性がある。」

などと書いています。この本は、そんなにすごい本なのでしょうか?

325 :132人目の素数さん:2017/09/24(日) 21:25:27.31 ID:K2Xyf521.net
訂正します:

赤攝也著『実数論講義』に

どのような物 a, b, c に対しても、 b ≠ c ならば

{a} ≠ {b, c}

である。

などという補題があり、それを1ページちょっと使って証明しています。

これは正気の沙汰でしょうか?

もし、こんなことまで証明するというなら、この本の中で他にも証明しないで使っている
様々なことも証明しなければならないのではないでしょうか?

赤さんは、「はじめに」に、

「本書は、これまで誰も言わなかった大事な事柄をたくさん含んでいる。それらが普及すれば、
微分積分学がガラリと体裁を変える可能性がある。」

などと書いています。この本は、そんなにすごい本なのでしょうか?

326 :132人目の素数さん:2017/09/24(日) 21:27:14.26 ID:K2Xyf521.net
あ、3行でした。証明に使っているのは。

327 :132人目の素数さん:2017/09/24(日) 21:29:12.92 ID:K2Xyf521.net
1ページちょっと使っているのは、

(a, b) = (c, d) ⇔ a = c ∧ b = d

のほうでした。

なぜ、順序対を

(a, b) := {{a}, {a, b}}

と定義すると厳密になるのでしょうか?

328 :132人目の素数さん:2017/09/24(日) 21:30:29.05 ID:K2Xyf521.net
集合を使ってすべてのオブジェクトを定義しないと厳密ではないということでしょうか?

意味が分かりません。

329 :132人目の素数さん:2017/09/24(日) 21:35:28.80 ID:K2Xyf521.net
赤さんは、集合の定義については、普通の本と同じような説明で済ませています。

330 :132人目の素数さん:2017/09/24(日) 21:45:04.05 ID:GfRH4xPO.net
>>327
杉浦や内田も順序対はその定義を使っているよね

331 :132人目の素数さん:2017/09/24(日) 21:59:25.24 ID:K2Xyf521.net
(a, b) = (c, d) ⇔ a = c ∧ b = d

の証明ですが、単なる論理遊びにしか見えません。

332 :132人目の素数さん:2017/09/24(日) 22:04:34.12 ID:d8rTCDSk.net
>>330
他の定義もあるけどこの定義がベストと思うよ

333 :132人目の素数さん:2017/09/24(日) 22:05:22.75 ID:d8rTCDSk.net
>>331
どう定義するといいと思う?

334 :132人目の素数さん:2017/09/24(日) 22:31:14.49 ID:K2Xyf521.net
>>333

一番目が a
二番目が b

である物の並びを

(a, b)

と書く。

でいいのではないでしょうか?

335 :132人目の素数さん:2017/09/24(日) 22:40:09.85 ID:tyPeLRKi.net
>>334
1番目、2番目とはどういうことですか?

336 :132人目の素数さん:2017/09/24(日) 22:50:39.99 ID:K2Xyf521.net
>>335

そういうことを言うのなら、集合の定義での「ものの集まり」とは何ですか?

とかいくらでも文句をつけられると思います。

ちなみに赤さんの本での写像の定義は、よくある普通の定義です。

337 :132人目の素数さん:2017/09/24(日) 23:10:23.27 ID:9vIupRIb.net
>>336
文章を記号の並びと見たときは1番目2番目の区別が付く
それは文章をメタレベルで見て初めて可能となる芸当
文章の表す理論そのものに記号の並びという概念は用意されていない
理論内部に記号の並びに相当する仕組み(a, b) = (c, d) ⇔ a = c ∧ b = d
を実装するための定義式が(a, b) := {{a}, {a, b}}

338 :132人目の素数さん:2017/09/24(日) 23:13:00.92 ID:BBiHOutO.net
宇宙飛行士と一級建築士はどっちの方が凄いですか?

339 :132人目の素数さん:2017/09/24(日) 23:13:09.71 ID:tyPeLRKi.net
>>337
{1,2}→Sへの写像のことを定義とすることはできないんですか?

340 :132人目の素数さん:2017/09/24(日) 23:13:25.31 ID:tyPeLRKi.net
>>338
神が一番すごいですね

341 :132人目の素数さん:2017/09/24(日) 23:15:11.47 ID:9vIupRIb.net
>>339
順序対を使って集合の直積を定義し、直積を使って写像を定義するので
「{1,2}→Sへの写像のこと」とすると循環論法になる

342 :132人目の素数さん:2017/09/24(日) 23:15:53.05 ID:BBiHOutO.net
>>340
神と全(全て)はどっちの方が凄いですか?

343 :132人目の素数さん:2017/09/24(日) 23:17:25.24 ID:tyPeLRKi.net
>>342
神がすごいです

344 :132人目の素数さん:2017/09/24(日) 23:19:05.85 ID:BBiHOutO.net
>>343
理由を教えてください。

345 :132人目の素数さん:2017/09/24(日) 23:31:19.66 ID:d8rTCDSk.net
>>334
>並び
ってどう定義したらいいと思う?

346 :132人目の素数さん:2017/09/24(日) 23:33:58.24 ID:tyPeLRKi.net
>>344
どうしてもです

347 :132人目の素数さん:2017/09/24(日) 23:46:16.89 ID:BBiHOutO.net
>>346
ちゃんと理由を教えてください。

348 :132人目の素数さん:2017/09/25(月) 00:07:12.48 ID:POg/ltn1.net
>>347
続きはドラゴンボールを見てください

349 :132人目の素数さん:2017/09/25(月) 01:04:13.22 ID:n8QgYxGk.net
>>336
>とかいくらでも文句をつけられると思います。

まさか松阪君からこんな言葉を聞くとは思わなかったわ
松阪君のやっていることがまさにその「いくらでもつけられる文句」を吐き出してることに他ならないというのに

350 :132人目の素数さん:2017/09/25(月) 01:23:47.25 ID:lluPnfGs.net
数学の質問です
誰でもいいから殺したいのですが、どうすればいいでしょうか?

351 :132人目の素数さん:2017/09/25(月) 10:12:57.05 ID:OwBXJOHD.net
>>350
ヒマラヤをころせば

352 :132人目の素数さん:2017/09/25(月) 13:01:36.93 ID:NKFaEY3h.net
>>327
他の定義を考えてみて
欠点を見つければ分かる

353 :132人目の素数さん:2017/09/25(月) 13:45:22.83 ID:de+0KVlr.net
>>334
君はたとえば、関数の連続性や、写像、ベクトル空間等といった概念をそれぞれ
"関数が繋がっていること"、"ある元にある元を対応させるもの"、"足し算や掛け算が出来る空間"
だけのふわっとした定義だけで済ませるのか?
大もとの集合論の記号と演算だけを使って定義が表せることに意味があるとは気づかないい?
考えないからずっと微積分レベルの教科書から先に進めないのか?

354 :132人目の素数さん:2017/09/25(月) 15:28:01.15 ID:XMlF/lbK.net
https://i.imgur.com/mB6VYcZ.jpg
この不等式の変形を教えてください
n→∞での不等式です

355 :132人目の素数さん:2017/09/25(月) 16:37:11.90 ID:q+o2dj5M.net
εが何かわからない時点で聞く気ないな

356 :132人目の素数さん:2017/09/25(月) 16:38:05.17 ID:q+o2dj5M.net
n(ε)

357 :132人目の素数さん:2017/09/25(月) 16:50:02.39 ID:zdRHmaSm.net
>>322

1
f(x)=-2(x-2)^2+11
Maxf(x)=f(2)=11

2
g(x)=2(x-1)^2+1
ming(x)=g(1)=1
|-2-1|>|2-1|
Maxg(x)=g(-2)=19

358 :132人目の素数さん:2017/09/25(月) 17:16:36.91 ID:XMlF/lbK.net
>>355
普通の任意のイプシロンです

359 :132人目の素数さん:2017/09/25(月) 17:47:47.74 ID:QDj/RYNC.net
(2)が手が出ません。どこから手を付けたら良いか不明です。ヒントだけでもよろしくお願いします。
(1)は漸化式を解く要領で解決しました。(3)はおそらく易しいと思います。

与えられた三角形を、何本かの直線によりk個の三角形のみに分割する。ただしk=1でも構わない。このとき、以下の問に答えよ。

(1)ある三角形をk個の三角形のみに分割するとき、kとしてあり得る値を全て求めよ。

(2)このようにして出来たk個の三角形のそれぞれの内接円を考える。このk個の内接円の面積の総和について、最大値、最小値、は存在するか。それぞれ、存在するならば値を求めよ。存在しない場合は、上限または下限またはその両方を求めよ。

(3)(2)で求めた最大値または上限をS(k)とし、与えられた三角形の面積をSとする。k→∞としたときの、S(k)/Sの値を求めよ。

360 :132人目の素数さん:2017/09/25(月) 19:10:50.46 ID:1G3CiMLn.net
全て求めよってことは有限なの?
無限に思えるけど

361 :132人目の素数さん:2017/09/25(月) 19:35:02.33 ID:mLskkQAA.net
有限な表現があるんでしよ

362 :132人目の素数さん:2017/09/25(月) 20:14:01.76 ID:8X0Gr8ml.net
「全て求めよってことは有限なの?」

まずこの認識がズレてるから

363 :132人目の素数さん:2017/09/25(月) 20:29:28.36 ID:89aoou8o.net
じゃあすべての整数ってこと?
そんなの問題になるの?

364 :132人目の素数さん:2017/09/25(月) 21:11:06.26 ID:QDj/RYNC.net
>>363
なるよ
任意の正整数が正解
多分出題者の教授が厳密さを求めたんじゃねーのかなと思うわ

365 :132人目の素数さん:2017/09/25(月) 21:28:12.02 ID:EqrqyK/7.net
https://i.imgur.com/aRWC9ha.jpg
この問題がどうしても解けません。教えていただけないでしょうか?
答えは254.34㎠です

366 :132人目の素数さん:2017/09/25(月) 21:32:59.59 ID:tQtJgULU.net
には普通に考えれば、k=1のとき最大で、
k=∞のとき細く分割してけば0になると

367 :132人目の素数さん:2017/09/25(月) 21:35:53.26 ID:Ei19fBUj.net
神と全はどっちの方が凄いですか?

368 :132人目の素数さん:2017/09/25(月) 21:46:30.05 ID:8090xECZ.net
うんこ

369 :132人目の素数さん:2017/09/25(月) 23:31:34.61 ID:c6cqCkK8.net
>>365
できる立体が、「バームクーヘンを縦半分に真っ二つにした形」になるのはわかりますか?
まず切る前のバームクーヘンの体積を求めます。外の円柱から中の空洞の円柱を抜けばいいですね
そのあとは真っ二つにするので、体積は半分です。

いくつかおかしな所がありますね。問題文は「立方体の体積は〜」ですが「立体の体積は〜」ですし、答えが「cm^2」と言ってますが問題が本当に「体積」を聞いているのなら「cm^3」ですよ

370 :132人目の素数さん:2017/09/26(火) 00:01:35.66 ID:2gE7E1k8.net
添削お願いします

f(x)=x^3-6x^2+1 は
x =(@)のとき最大値(A)
x =(B)のとき最小値(C)である
区間1=<x=<6 での最大値は(D)
(E)個の実数解を持ち、正の解は(F)個
-----------------------------------------------
回答
微分して 3x^2-12x であるから解は 0, -2
0のとき1, -2のときは -8-24+1=-31

増減表
x  0  1 6
y ↓1↓-4↑1

3箇所でX軸と交差、マイナス側で1箇所、+側で2箇所

@0 A1 B-2 C-31 D1 E3 F2

371 :132人目の素数さん:2017/09/26(火) 00:43:31.48 ID:WISpZz5V.net
>>370
いいと思う
もし記述式なら増減表に極値とるとこを書いたほうがいいね、程度

372 :132人目の素数さん:2017/09/26(火) 01:15:46.40 ID:jpTA6Ad2.net
>>369
すいません㎤でした教えていただきありがとうございます。

373 :132人目の素数さん:2017/09/26(火) 01:56:49.60 ID:jUmrGTgV.net
全と神はどっちの方が凄いですか?

374 :132人目の素数さん:2017/09/26(火) 03:37:52.97 ID:WISpZz5V.net
3辺の長さがそれぞれ5,12,13である直角三角形がある。
この直角三角形の、長さ12の辺と長さ13の辺で挟まれる鋭角の角度をθ°とするとき、θの整数部分を求めよ。
(なお実数αの整数部分とは、αを十進表示したときの小数点以下を切り捨てた整数を指す。)

30°や18°といった有名角に関連があるかと思ったのですが、上手く行きません。ご教授ください。

375 :132人目の素数さん:2017/09/26(火) 05:45:48.09 ID:j/SJbTWH.net
Arctan(5/12)=θ
Arctan(x)
=Σ[n=0,∞](((-1)^n)/(2n+1))*(x^(2n+1))
=x-(x^3)/3+(x^5)/5-(x^7)/7+…
にx=5/12を入れて、評価

376 :132人目の素数さん:2017/09/26(火) 05:57:05.90 ID:RoYdOwoq.net
頑張って二分法やった結果22
0°と30°で始めたら6ステップで挟めたけど…
綺麗な解き方は知らね

377 :132人目の素数さん:2017/09/26(火) 06:43:20.75 ID:j/SJbTWH.net
度数法か
面倒くさ

378 :132人目の素数さん:2017/09/26(火) 06:48:51.61 ID:jUmrGTgV.net
人は死んだらどこに行くのでしょうか?

379 :132人目の素数さん:2017/09/26(火) 07:08:20.38 ID:2gE7E1k8.net
古代エジプトじゃ死んだら神さまに算数の問題出されたっていうな
間違えると成仏させてもらえない

380 :132人目の素数さん:2017/09/26(火) 11:59:44.18 ID:EHLZfEpP.net
>>374

22.5°を使う

5/12 = 0.4166666
=(√2 -1)+ 0.0024531
= tan(22.5゚)+ δ

22.5°< Arctan(tan(22.5゚)+ δ)
< 22.5°+ δ/{1 + tan(22.5゚)^2}
= 22.5° + δ/(4-2√2)
= 22.5°+ 0.002453/(4-2√2)
= 22.500°+ 0.120°
= 22.620°

* {Arctan(x)} ’ = 1/(1+xx),

381 :132人目の素数さん:2017/09/26(火) 12:26:32.94 ID:EHLZfEpP.net
>>374

arctan(5/12)= θ とおき、倍角 2θ を求める。

tan(90°-2θ)={1 -(tanθ)^2}/(2tanθ)= 1 - 1/120,

45°> 90°-2θ
≧ 45°-(1/2)(1/120)
= 45°- 0.240°

22.5°<θ≦ 22.620°

382 :132人目の素数さん:2017/09/26(火) 16:47:26.84 ID:f4V/l9ng.net
約22度になることを知ってから解法組み立ててない?

383 :132人目の素数さん:2017/09/26(火) 22:18:02.29 ID:WISpZz5V.net
ありがとうございます!
tanと倍角を使うと高校生でも解決できますね
でも、その倍額つかうという着想が得られないというか、言われれば分かるんですけど難しいです

384 :132人目の素数さん:2017/09/26(火) 23:26:45.04 ID:r3fQ2atK.net
完全なる無になってもう二度と目覚めたくない。
どうすれば良い?
死んでもそれは実現できないの?

385 :132人目の素数さん:2017/09/26(火) 23:46:20.94 ID:YpGGsh1V.net
神を信じましょう

386 :132人目の素数さん:2017/09/26(火) 23:46:35.14 ID:RoYdOwoq.net
有名角から倍角半角使う以外に方法が思いつかないし

387 :132人目の素数さん:2017/09/27(水) 03:45:42.53 ID:VohnE2a1.net
Q.【数学できる方!】 標準偏差から上位半分の標準偏差を割り出したい。
どうかお願いします。

1)標準偏差   2)上位半分の標準偏差
3.0〜4.0.... 4.0    (比較:+0.5)
4.0〜5.0.... 4.7    (+0.2)
5.0〜6.0.... 5.4    (−0.1)
6.0〜7.0.... 6.2    (−0.3)
7.0〜8.0.... 6.8    (−0.7)
8.0〜9.0.... 7.4    (−1.1)
9.0〜10.0.... 7.9    (−1.6)
10.0〜11.0.... 8.4    (−2.1)
11.0〜12.0.... 8.6    (−2.8)
12.0〜13.0.... 8.6    (−3.9)
13.0〜14.0.... 8.6    (−4.9)
14.0〜15.0.... 8.6    (−5.9)
15〜20... 8.9

こんなデータがあるとします。
標準偏差5.5の時、上位の標準偏差は5.4
標準偏差8.5の時、上位の標準偏差は7.4 のような感じです。

ここで、[標準偏差から、上位半分の標準偏差を割り出す] 公式を作りたいのですが
どのようにすれば良いでしょうか? 数学のできる方力を貸してほしいです。
厳密にではなくてもざっくばらんに対応できる式であればOKです。よろしくお願いします。
m(_ _)m(お礼用意しております)

388 :132人目の素数さん:2017/09/27(水) 04:28:38.60 ID:VohnE2a1.net
値(2)の上限を9.0として、値(1)15.5〜5.5へかけて比較値が漸減するので
9.0−(15.5−(値1))×**  …を考えてみたのですが×0.2とすると割と言い当てられますね。
しかし正確ではないので平方根等を使うのが良いのでしょうか。

お礼1は『PC光は体(脳)に悪い』という情報です。
中国では被験者に鬱の傾向が発生。画面を長く見るならモニタの輝度を落としたり、PCメガネを着用しましょう。
・東北大学は12月9日、青色光を当てると昆虫が死ぬことを発見したと発表した。
http://soyokazesokuhou.com/post-453/
https://www.ishamachi.com/?p=904
> 体内リズムを乱したり、目に傷害を与えたりすることが指摘されています。厚生労働省でも
> パソコンなど液晶モニターがある「VDT機器」で作業する際のガイドラインで「作業時間が
> 1時間を超えないようにし、10〜15分程度の休憩を取る」よう推奨しています(職場のあんぜんサイトより)。

389 :205:2017/09/27(水) 06:31:54.83 ID:O6xdhlj3.net
>>387
半分は中央値じゃね?
標準偏差は分布の散らばりを表す値

上位半分の標準偏差って何?

390 :132人目の素数さん:2017/09/27(水) 07:10:22.63 ID:CI0V/sK1.net
この式が数学的におかしいのか教えてほしいです

5.105%=5%×102.1%

百分率に百分率をかけるのはおかしいのでしょうか?

391 :132人目の素数さん:2017/09/27(水) 08:54:16.43 ID:xH1+BNpi.net
0.05105=0.05*1.021
と同じことだから正しい

392 :132人目の素数さん:2017/09/27(水) 11:46:27.83 ID:OyWGohGG.net
0<α<π/2 のとき
a_1=sin(α)
b_1=cos(α)

a_(n+1)=sin(b_n)
b_(n+1)=cos(a_n)

と、数列{a_n} {b_n}を定めると

任意の自然数mに対して
a(2m)>b(2m)


証明がわからない(-_-)

393 :132人目の素数さん:2017/09/27(水) 12:12:57.90 ID:UKYLzA5A.net
sin(x)をxに、cos(x)を1-xに、0<α<π/2 を 0<α<1に置き換えた問題を考えると、方針が見えてくると思うよ

394 :132人目の素数さん:2017/09/27(水) 13:11:36.88 ID:jGhKaq0N.net
三角関数の中に三角関数がある式を見ると気持ち悪い
そういうもんじゃねえから

395 :132人目の素数さん:2017/09/27(水) 13:18:23.74 ID:IOYjZ/Ce.net
loglog やexpexp も気持ち悪いのか?

396 :132人目の素数さん:2017/09/27(水) 13:25:06.25 ID:jGhKaq0N.net
テトレーションは演算の拡張と見なせる

397 :132人目の素数さん:2017/09/27(水) 15:19:46.81 ID:OyWGohGG.net
>>393
すいません、よくわかりません(>人<;)

398 :132人目の素数さん:2017/09/27(水) 15:21:09.22 ID:2rqT67c6.net
じゃあ三角関数は「どういうもん」やねん

399 :132人目の素数さん:2017/09/27(水) 16:11:15.94 ID:dNhC2+4n.net
二次関数の最大値最小値で変域あるやつってわざわざグラフ自分で書き起こしてじゃないと解けないものなんですか?
全然解けなくて困ってます。

400 :132人目の素数さん:2017/09/27(水) 16:37:56.75 ID:JgT615C0.net
円筒は切り開いて平らにすることができます。
球は切り開いて平らにすることができません。

円錐を切り開いて平らにすることができることはどうやって証明するのでしょうか?
これは直観的に明らかなことでしょうか?

401 :132人目の素数さん:2017/09/27(水) 16:39:15.04 ID:JgT615C0.net
よく円錐上の2点の最短距離を求める問題で、展開図を考えます。

展開できることはそんなに明らかなことなのでしょうか?

402 :132人目の素数さん:2017/09/27(水) 17:19:43.66 ID:f7+cfmG+.net
>>400
直交変換で

403 :132人目の素数さん:2017/09/27(水) 18:02:24.65 ID:YSKFP/7i.net
F(x)=tan|x|-|x|がx=0における連続性と微分可能性を調べよ。

連続性は分かるのですが、微分可能性がどうしても出来ません。
お手数ですが、分かる方。解説をお願いします。

404 :132人目の素数さん:2017/09/27(水) 18:22:24.56 ID:nGuFQ0y9.net
ボードゲームの展示イベント「ゲームマーケット」の成長記録からこれからの
市場に必要なことを妄想してみた。6年間の来場者数推移(2016年4月時点調べ)
http://bodoge.hoobby.net/columns/00001
ボードゲーム市場がクラウドファンディングの出現で急成長を遂げ市場規模を拡大中
http://gigazine.net/news/20150820-board-game-crowdfunding/
中世っぽいデザインの金属サイコロ&ダイスカップ「Rhythm Metal Gaming Dice」
http://gigazine.net/news/20140207-rhythm-metal-dice/
ファンタジー世界っぽい15種類のデザインのコインセット「Fantasy Coins」
http://gigazine.net/news/20140415-fantasy-coins/
デザイン戦略やタイポグラフィの歴史などがゲームしつつ理解できるトランプ「The Design Deck」レビュー
http://gigazine.net/news/20161027-design-deck-review/
ボードゲームのオリジナルオーダー制作
http://www.logygames.com/logy/ordermade.html
500円ボードゲームのすすめ
http://kazuma.yaekumo.com/c0011_20161206advent.html
はじめてのボードゲーム制作記
http://nrmgoraku.hateblo.jp/entry/2017/05/22/221258
ゼロからボドゲを作って販売して分かった、3つのこと
http://begin-boardgames.seesaa.net/category/23191867-1.html
靴箱でテーブルサッカーゲームの作り方
http://www.handful.jp/curation/4207
ゲームマーケット2017春お疲れ様でした!!来場者は1000人増の13000人!
http://boardgamenumber.tokyo/2017/05/16/post-416/
2017年開催のボードゲームイベント一覧
http://nicobodo.com/archives/19131088.html

405 :132人目の素数さん:2017/09/27(水) 18:38:16.80 ID:V5ez6wrJ.net
>>400
微分幾何でそのようなことを扱う
直観をどのような範囲で意味するかによるが
「気持ちとしては明らか」かもしれないがそれを
数学の問題として定式化して証明をつけたりするのは
決して自明ではない

406 :132人目の素数さん:2017/09/27(水) 19:14:45.90 ID:ksrDff2j.net
>>405

ありがとうございます。

小学生用の問題で、円錐上の2点の最短距離を求めよという問題がありますが、あれは
悪問ですね。

円錐は切り開いて平らにすることができるかどうかを全く問題にせず、あたかも自明な事実
であるかのように扱うのは問題がありますね。

407 :132人目の素数さん:2017/09/27(水) 19:15:03.16 ID:+jk5k2pS.net
理系の方はMathematicaにお世話になったことがあると思います。
そんなMathematicaなのですが、非常に正直で方程式を解くと重解もすべて表示してくれます。
例えば、(x-1)^100=0 を解いてとお願いするとx=1,x=1,x=1,x=1,...と丁寧に全部教えてくれます。
そこで私は意地悪を思いつきました。
「解が無限にある方程式の答えもたくさん表示してくれるのかな?」と。
試しに Cos(x)=1 を解いてもらうとMathematicaは
x=2πC ,C∈Integer
と上手く答えを返してきました。
悔しいです。
そこで皆様に、解が無限にあり解が規則的に表せないような方程式を教えて頂きたいです。
よろしくお願いいたします。

408 :132人目の素数さん:2017/09/27(水) 19:34:15.48 ID:in1X5HTs.net
事実でも証明しないと使っちゃいけないなら、足し算掛け算の交換法則なんかも証明しないといけないのか
大変だな

409 :132人目の素数さん:2017/09/27(水) 19:53:16.81 ID:bhFEZWJa.net
もう一歩進んで、数学的に定義されたものだけが「事実」の対象であると勘違いさせた可能性もある
実に罪深い助言

410 :132人目の素数さん:2017/09/27(水) 20:31:10.54 ID:7KNDMmeu.net
>>403
左右の微分係数計算

411 :132人目の素数さん:2017/09/27(水) 20:32:03.00 ID:7KNDMmeu.net
>>407
tanx=x

412 :132人目の素数さん:2017/09/27(水) 22:01:20.50 ID:Ut35bUl+.net
>>410
ありがとうございます。解決しました

413 :132人目の素数さん:2017/09/28(木) 00:00:55.89 ID:SMz7fgS0.net
>>406
そもそも小学生の算数は
経験で納得するレベルなんだから
問題にはならない

難癖つけて悦に入るアンタみたいな
輩のほうがよっぽど問題だわな

414 :132人目の素数さん:2017/09/28(木) 02:55:46.07 ID:Gtc4wEZ1.net
学問をやっても、人間が本来的に持っている問題の解決にはならないわけじゃん。
例えば、リーマン予想を証明したとしても、
もし密室に閉じ込められてチンパンジーの集団をそこに放り込まれたら・・・・・
と、考えるとやはり、そんな人間が本来的に持っている、「恐怖や苦しみから逃れるにはどうすれば良いか?」
という問題の解決にはならないことをやっていても無意味なんじゃないかと思いませんか?
確かお釈迦様も似たようなことを言ってたような気がする。
自殺をすれば良いじゃんと言う人がいるかもしれないが、
本当に死んで解決できるのだろうか?
死んで完全なる無になり、もう二度と有にならずに済むのなら今すぐにでも自殺したいが、
冷静に「無」に関して考えると、「無」にはなれないような気がしてきたのですが・・・・・。
どうすれば良いのだろう・・・?

415 :132人目の素数さん:2017/09/28(木) 02:59:27.94 ID:K6pbiBGb.net
ID:ksrDff2j = ID:Gtc4wEZ1

416 :132人目の素数さん:2017/09/28(木) 08:50:53.39 ID:Gtc4wEZ1.net
「「無」になってもう二度と「有」にならないようにするにはどうすれば良いか?」
という疑問に対する答えを見つけ出そうとする学問がもしあるとしたら、それはやる意味も価値もあると思う。
なぜなら、「「無」になってもう二度と「有」にならない」というのは、
人間が本来持っている最も重要で根本的な「痛みや苦しみや恐怖などから解放されたい」という切実な願いを叶えるものだからである。
そのような学問があるとしたら、その切実な願いを叶えるための手段になるわけだから、
当然、その学問はやる意味もあるし価値もあるということになる。
寧ろ、人類総出でやっていった方が良いと思う。
しかし、それ以外の学問はその、最も重要且つ重大な問題から解放されるための手段には全くならないので、無意味無価値無駄と言えるのではないでしょうか?

417 :132人目の素数さん:2017/09/28(木) 08:56:42.39 ID:K0ngR2CK.net
>>416
無とはなんでしょうか

418 :132人目の素数さん:2017/09/28(木) 09:04:12.95 ID:Gtc4wEZ1.net
>>417
まさにそれなんだよ。
「無」ってのは何も無いことなんだけど、
その「何も無い」ってのが一見単純そうに見えて実は半端なく難しい。
今俺も「無」に関して考えているけど、奥が深すぎてなかなか理解できない。

419 :132人目の素数さん:2017/09/28(木) 09:04:13.01 ID:Gtc4wEZ1.net
>>417
まさにそれなんだよ。
「無」ってのは何も無いことなんだけど、
その「何も無い」ってのが一見単純そうに見えて実は半端なく難しい。
今俺も「無」に関して考えているけど、奥が深すぎてなかなか理解できない。

420 :132人目の素数さん:2017/09/28(木) 09:11:53.64 ID:K0ngR2CK.net
何もない状態を「無」と定義するなら、それは「無」がある状態になるので、定義と矛盾しますよね?

421 :132人目の素数さん:2017/09/28(木) 09:30:22.48 ID:GL4OOBGE.net
>>407
リーマンζ関数の零点

ζ(1/2 + ix)= 0, xは実数

も出してくれないかな?

422 :132人目の素数さん:2017/09/28(木) 09:33:29.63 ID:fm+6FXNG.net
「無」をどうと定義したのか見返してみ?

423 :132人目の素数さん:2017/09/28(木) 10:25:12.29 ID:kh/qX0Yf.net
荒らしにかまうアホ

424 :132人目の素数さん:2017/09/28(木) 11:04:41.01 ID:2UDPl1Bo.net
e^x, sin(x), cos(x)のテイラー展開は、
e^x = …, sin(x) = …, cos(x) = …と展開されますが、
log(x)のテイラー展開はなぜlog(1+x) = …と展開するんですか?
たとえばlog(2+x) = …と展開しないのはなぜですか?

425 :132人目の素数さん:2017/09/28(木) 13:07:59.33 ID:Gtc4wEZ1.net
ジョン・フォン・ノイマンと法然はどっちの方が頭が良いですか?

426 :132人目の素数さん:2017/09/28(木) 13:13:14.92 ID:iaC+gXkQ.net
荒らしだね

427 :132人目の素数さん:2017/09/28(木) 15:29:50.85 ID:m+SzvOVg.net
>>411>>421
ご回答ありがとうございます。
明日実際にMathematicaで試させていただきます。

428 :132人目の素数さん:2017/09/28(木) 16:21:28.03 ID:lGp1vXS4.net
任意の正方行列AについてAXA=Aなる可逆行列Xが存在することを示せ。

429 :132人目の素数さん:2017/09/28(木) 16:26:54.22 ID:BGCb3Smp.net
度々失礼します

問題

aを自然数とする。 不等式 x−2−7|x−2|+A≧0 を満たす整数xの個数が3個であるようなaのうち、最小の自然数aは何か。

という問題なのですが、一応自力で答えは出せたのですが解答の解き方がよく分からなかったので質問させていただきます。

x≧2のとき、2+a/6≧x≧2

x<2のとき、2>x≧2−a/8 なので2+a/6≧x≧2−a/8…* となるのは分かるのですが

この後回答では、(*)を満たす整数xが3個となるのは 0<2−a/8≦1かつ3≦2+a/6<4の時と書いてあるのですが

なぜこのようになるのかどうか教えてください

430 :132人目の素数さん:2017/09/28(木) 17:11:36.42 ID:pvGvywAu.net
>>418
無はそれ自身無に落ちるという警句がある
無という言葉はいかなる指示内容も持たない名辞なんだよ
これに気づけば「無」についていかなる謎も存在しない

「無」に関して記述しようとすると「存在の否定」としか記述できない
だから「無」に先立って「存在の欠落を認識できる有」が存在しなければならない
だから「はじめにあるものがあった」というしかないわけ

陰陽論でも陰と陽のさらにその前には「一元陽気」が存在したと考えるのが
普通ですよ

431 :132人目の素数さん:2017/09/28(木) 17:13:10.19 ID:pvGvywAu.net
>>424
ちょうどいいからとしかいいようがない.
(係数が一番綺麗になる)

432 :132人目の素数さん:2017/09/28(木) 18:16:51.09 ID:GG4t6iIO.net
数学得意モメン来てくれ [無断転載禁止](c)2ch.net [363682846]
http://leia.2ch.net/test/read.cgi/poverty/1506588618/

433 :132人目の素数さん:2017/09/28(木) 19:43:24.03 ID:cj4g35+u.net
>>424
強いて言えば
log(2+x)=log(2)+log(1+(x/2))
だから log(1+x) が展開できていれば済む。

434 :132人目の素数さん:2017/09/29(金) 02:44:43.31 ID:cI5wzbdU.net
空海とガウスはどっちの方が天才ですか?

435 :132人目の素数さん:2017/09/29(金) 02:55:22.56 ID:OHxEoTEp.net
神です

436 :132人目の素数さん:2017/09/29(金) 03:08:34.90 ID:cI5wzbdU.net
全(全て)と神はどっちの方が凄いですか?

437 :132人目の素数さん:2017/09/29(金) 03:09:12.94 ID:cI5wzbdU.net
全(全て)と神はどっちの方が凄いですか?

438 :205:2017/09/29(金) 06:37:13.56 ID:K+Jo/bNl.net
空の方が凄いです

439 :132人目の素数さん:2017/09/29(金) 08:29:16.76 ID:rKGYFucW.net
行列の正定かどうかについて質問です。

あるn×nのテプリッツ行列 A があるとき、そのテプリッツ行列自信の共役複素転置行列(虚数がなければ転置行列)をかけ、
D=A*A^H
と表したとき、この

行列Dは常に正定行列であるかどうかを教えてください。

できれば理由もお願いします。

440 :132人目の素数さん:2017/09/29(金) 11:30:44.82 ID:oIFvV/UE.net
>>439
当たり前では?

441 :132人目の素数さん:2017/09/29(金) 12:50:44.68 ID:jxyc/YQt.net
>>439
G = A^H A はグラム行列(Grammian)
明らかにエルミート

任意のn次ベクトルxに対し
(x,Gx)=(x,A^H Ax)=(Ax,Ax)≧0,  標準内積

|A|≠0 のとき、Gは正定エルミート
|A|= 0 のとき、Gは半正エルミート

442 :132人目の素数さん:2017/09/29(金) 16:23:57.50 ID:NHnJa4X9.net
吉田伸生著『微分積分』を読んでいます。

p.7 例1.2.6で

a, b ∈ Q, a ≦ b なら、 Z ∩ (-∞, a) ≠ ?, Z ∩ (b, ∞) ≠ ?, また
Z ∩ [a, b] は有限集合である。

という命題を証明しているのですが、それを証明と言っていいのか疑問に思いました。
公理から証明しているわけではないです。

こういう明らかな命題を証明するのなら、設定された公理から証明しなければ意味不明な
証明になってしまうように思います。

それと、この本は非常に癖の強い本であるように思います。

443 :132人目の素数さん:2017/09/29(金) 16:24:35.17 ID:NHnJa4X9.net
吉田伸生著『微分積分』を読んでいます。

p.7 例1.2.6で

a, b ∈ Q, a ≦ b なら、 Z ∩ (-∞, a) ≠ 空集合, Z ∩ (b, ∞) ≠ 空集合, また
Z ∩ [a, b] は有限集合である。

という命題を証明しているのですが、それを証明と言っていいのか疑問に思いました。
公理から証明しているわけではないです。

こういう明らかな命題を証明するのなら、設定された公理から証明しなければ意味不明な
証明になってしまうように思います。

それと、この本は非常に癖の強い本であるように思います。

444 :132人目の素数さん:2017/09/29(金) 16:30:40.60 ID:NHnJa4X9.net
公理を設定していないため、明らかなことを明らかな事実を使って証明しているようにしか
思えません。

公理が設定されているのなら、ここはこの公理を使っている、そこはこの公理を使っている
と確認することができますが、それができません。

非常に問題があるといえます。

445 :132人目の素数さん:2017/09/29(金) 16:38:04.47 ID:NHnJa4X9.net
「癖」についてですが、吉田さんは、例えば、

p, q ∈ Z, p ≦ q のとき、

集合 {p, p+1, …, q} を Z ∩ [p, q] などと書いています。

間違ってはいませんが、直接的な表現である {p, p+1, …, q} を
使ったほうが分かりやすいはずです。

単に、書くときに、文字数を節約できて自分が楽だという理由からこのような
表現を使っているとしか思えません。

446 :132人目の素数さん:2017/09/29(金) 16:42:02.54 ID:NHnJa4X9.net
R^- := R ∪ {±∞} とします。

吉田さんは上界の定義を以下のように定義しています。


A ⊂ R^-, m ∈ R^- とする:

A ⊂ [m, ∞] なら m は A の上界であるという。


これも非常に分かりづらい定義です。

447 :132人目の素数さん:2017/09/29(金) 16:43:39.01 ID:NHnJa4X9.net
訂正します:

R^- := R ∪ {±∞} とします。

吉田さんは上界の定義を以下のように定義しています。


A ⊂ R^-, m ∈ R^- とする:

A ⊂ [-∞, m] なら m は A の上界であるという。


これも非常に分かりづらい定義です。

448 :132人目の素数さん:2017/09/29(金) 16:45:35.54 ID:NHnJa4X9.net
要するに読者のことなど何も考えていない非常に自分勝手な著者だといえます。

449 :132人目の素数さん:2017/09/29(金) 16:49:29.22 ID:1pxV1ZAo.net
何故同じような微積分の本ばかり読んでいるのですか?

450 :132人目の素数さん:2017/09/29(金) 16:51:41.30 ID:NHnJa4X9.net
A ⊂ R^-, m ∈ R^- とする:

A の任意の元 a に対して、 a ≦ m ならば m は A の上界であるという。

と書けばいいはずです。

451 :132人目の素数さん:2017/09/29(金) 16:52:42.71 ID:1pxV1ZAo.net
何故同じような微積分の本ばかり読んでいるのですか?

452 :132人目の素数さん:2017/09/29(金) 16:53:49.41 ID:NHnJa4X9.net
A ⊂ R^-, m ∈ R^- とする:

A ⊂ [-∞, m] なら m は A の上界であるという。


-∞などという不必要な要素が含まれていて分かりにくいです。

453 :132人目の素数さん:2017/09/29(金) 16:55:48.76 ID:1pxV1ZAo.net
何故同じような微積分の本ばかり読んでいるのですか?
無視しないでください

454 :132人目の素数さん:2017/09/29(金) 17:00:39.97 ID:PvasFUwt.net
>>443
キミ自身が相当に癖の強い人であるように
思います。
非常に問題があるといえます。

要するに著者やこのスレの閲覧者のなど
何も考えていない非常に自分勝手なヒト
だといえます。

分からない問題スレに書かなければ
いいはずです。

455 :132人目の素数さん:2017/09/29(金) 17:11:56.54 ID:UvH/q127.net
(2n+1)次正方行列がある
対角成分は全て0で、各行にはn個の1とn個の-1が含まれる。
この行列のrankを求めよ。

456 :132人目の素数さん:2017/09/29(金) 17:31:17.78 ID:mOndYS00.net
IDは変わりましたが>>429です
一日たっても根本的な解決方法がわかりませんでした
どうか、回答へ至る解説をお願いします

457 :132人目の素数さん:2017/09/29(金) 17:41:24.79 ID:NHnJa4X9.net
A ⊂ R^-, m ∈ R^- とする:

A ⊂ [-∞, m] なら m は A の上界であるという。



例1.2.9

-∞ ≦ a < b ≦ ∞, (a, b) ⊂ I ⊂ [a, b] なら、

x が I の上界 ⇔ b ≦ x


証明:

⇒: 仮定より (a, b) ⊂ I ⊂ [-∞, x]。よって b ≦ x。

458 :132人目の素数さん:2017/09/29(金) 17:44:43.89 ID:NHnJa4X9.net
>>457



「よって b ≦ x」はひどすぎます。

b ∈ I でないときに、「よって b ≦ x」とだけ書いてすますのはおかしいと思います。

459 :132人目の素数さん:2017/09/29(金) 17:47:43.80 ID:1pxV1ZAo.net
何故同じような微積分の本ばかり読んでいるのですか?
無視しないでください

460 :132人目の素数さん:2017/09/29(金) 17:53:43.88 ID:NHnJa4X9.net
吉田さんの本を読む人は第1章を読んだだけで嫌気がさすと思います。

461 :132人目の素数さん:2017/09/29(金) 17:55:09.43 ID:1pxV1ZAo.net
何故同じような微積分の本ばかり読んでいるのですか?
無視しないでください
ここの住人は貴方に嫌気がさしていますよ

462 :132人目の素数さん:2017/09/29(金) 18:16:48.94 ID:NHnJa4X9.net
吉田伸生さんの本は読みにくすぎます。

b ∈ Q, 空集合 ≠ A ⊂ Z とする。
A ⊂ (-∞, b] なら max A が存在する。

この命題の証明が以下です:

A ≠ 空集合 より a ∈ A を一つ選ぶ。このとき、 Z ∩ [a, b] は有限集合(例1.2.6)、したがって
その部分集合 A ∩ [a, b] も有限集合である(問1.2.1)。さらに a ∈ A ∩ [a, b] より
A ∩ [a, b] ≠ 空集合。ゆえに m = max(A ∩ [a, b]) が存在する(例1.2.8)。このとき、
m ∈ A だから、任意の x ∈ A に対し x ≦ m なら m = max A である。ところが x ∈ A ∩ [a, b]
なら、 x ≦ m。また、 x ∈ A ∩ (-∞, a) なら、 x < a ≦ m。以上から m = max A。


この明らかな命題に対する吉田さんの証明をすらすら読める人はあまりいないのではないでしょうか?

463 :132人目の素数さん:2017/09/29(金) 18:54:48.18 ID:8jrexOB1.net
>>456
2-a/8≦x≦2+a/6 を満たす整数が3個だけ、ということは
2が解の一つであることを考え合わせると、整数解は
{0,1,2} 、{1,2,3}、{2,3,4}のどれかである。
{0,1,2} が解ならば、 -1<2-a/8≦0 かつ 2≦2+a/6<3 であることが必要十分だが、
これを満たす a は存在しない。(最初の方からは 16≦a<24、後の方からは 0≦a<6となる)
また、{2,3,4}が解ならば、 1<2-a/8≦2 かつ 4≦2+a/6<5 であることが必要十分だが、
同じく、これを満たす a は存在しない。
よって、問題の主張が満たされるなら、解は {1,2,3}しかなく、
そのための必要十分条件は 0<2-a/8≦1 かつ 3≦2+a/6<4 であり、
これを解くと 8≦a<16 かつ 6≦a<12 となり、これより 8≦a<12。
これより、求める a の値は 8 である。

464 :132人目の素数さん:2017/09/29(金) 19:03:33.51 ID:mOndYS00.net
>>463
なるほど、つまりこの場合数直線で考えた場合、範囲内では2を含む連続した整数という考え方で間違いないでしょうか

465 :132人目の素数さん:2017/09/29(金) 21:42:31.85 ID:TOEu8UD3.net
>>443
吉田という人のことは知りませんが恐らく,最初に実数の集合が
ありその中で{1}で生成される部分加法群として整数の集合が
あるのだと理解してるのでしょうね

杉浦解析入門でもそんなスタイルだったと思います

数学基礎論(と言ってもキューネン数学基礎論講義程度の入り口の内容)
を全く知らない人だとその程度の認識なんだと思います

466 :132人目の素数さん:2017/09/29(金) 21:45:29.39 ID:COMZn4di.net
>>462
アンタの書き込みをすらすら読める人も
あまりいないのではないでしょうか。

467 :132人目の素数さん:2017/09/29(金) 22:51:38.08 ID:oIFvV/UE.net
>>458
>b ∈ I でないときに


468 :132人目の素数さん:2017/09/30(土) 02:41:45.17 ID:+K9HN1RE.net
アメリカ合衆国大統領と三菱財閥総帥はどっちの方が凄いですか?

469 :132人目の素数さん:2017/09/30(土) 10:02:44.32 ID:YM5SrsaA.net
高橋幸雄著『確率論』を読んでいます。

事象列 A1, A2, … が単調非減少ならば

P{∪An} = lim P{An}

が成り立つことの証明で、

Bn := A_(n+1) - An とおくと、 ∪An は互いに排反な可算個の和事象として、

∪An = A1 ∪ (∪Bn)

と書ける。

などと書かれていますが、 ∪An は互いに排反な可算個の和事象ではないですよね。

470 :132人目の素数さん:2017/09/30(土) 10:32:39.13 ID:+odscTt+.net
>>469
右辺のこと

471 :132人目の素数さん:2017/09/30(土) 10:35:36.18 ID:YM5SrsaA.net
A1
B1 = A2 - A1
B2 = A3 - A2

一般に、

A1 ∩ B2 ≠ 空集合

です。

472 :132人目の素数さん:2017/09/30(土) 10:45:30.11 ID:8paZwJmN.net
お、別の本読み出したぞww

473 :132人目の素数さん:2017/09/30(土) 11:17:24.73 ID:bZQLymFj.net
非単調減少がわかってないのか

474 :132人目の素数さん:2017/09/30(土) 11:19:01.63 ID:YM5SrsaA.net
>>473

ありがとうございます。

単調非減少であることを見落としていました。

475 :132人目の素数さん:2017/09/30(土) 12:41:06.49 ID:E2ZCextk.net
この【問題】への以下の解答はダメでしょうか。ちゃんと断面積を求めて積分しないとダメでしょうか。

【問題】xyz空間において、領域(x^2n)+(y^2n)+(z^2n)≦1の体積をV_nとする。
ただしnは自然数である。
lim(n→∞)V_nを求めよ。

【解答】nが十分に大きいとき、x<1ならばx^2nはほぼ0とみなせる。同様にx=1なら1である。また、x>1ならば十分に大きくなり、このとき明らかに(x^2n)+(y^2n)+(z^2n)≦1を満たさない。
y,zについても同様である。したがってn→∞においてこの不等式を満たす(x,y,z)は、a,b,c,d,e,fを絶対値1未満の実数として、(±1,a,b)、(c,±1,d)、(e,f,±1)以外ない。
よってこの領域は「一辺の長さ2の立方体から、各辺と各頂点を除いたもの」になるから、求める極限値は8

476 :132人目の素数さん:2017/09/30(土) 12:47:07.98 ID:zT5b3b9x.net
気持ちはわかるが、lim∫=∫lim を当たり前のように使ってるので0点

477 :132人目の素数さん:2017/09/30(土) 13:41:47.54 ID:RiygBVj3.net
立方体の表面しかないのに体積があるのか

478 :132人目の素数さん:2017/09/30(土) 13:47:19.25 ID:jeF4uAmu.net
法然とロスチャイルドはどっちの方が凄いですか?

479 :132人目の素数さん:2017/09/30(土) 19:00:15.80 ID:HlZgXGMw.net
>>474
自分が間違ってたときは
何て言うんだっけ?

480 :132人目の素数さん:2017/09/30(土) 19:08:13.38 ID:RiygBVj3.net
きっしょ

481 :132人目の素数さん:2017/09/30(土) 19:20:21.91 ID:8paZwJmN.net
自分も間違うくせに教科書の不備は許さない奴www

482 :132人目の素数さん:2017/09/30(土) 20:19:44.00 ID:cawMiVwS.net
>>471
単調非減少

483 :132人目の素数さん:2017/09/30(土) 20:20:59.55 ID:cawMiVwS.net
>>474
見落とすって・・・・・・見落とすことってできる?

484 :132人目の素数さん:2017/09/30(土) 20:59:21.55 ID:YM5SrsaA.net
任意の2つの数列 (a_n), (b_n) について

lim sup (a_n + b_n) ≦ lim sup a_n + lim sup b_n

が成り立つことを証明せよ。ただし、右辺は ∞ - ∞ の形ではないとする。


この問題の解答に以下のように書かれています。

「lim sup a_n, lim sup b_n の一方が +∞ で、他方が +∞ または有限ならば両辺とも +∞」

lim sup a_n, lim sup b_n の一方が +∞ で、他方が有限のとき、左辺が +∞ になることは
どうやって証明するのでしょうか?

485 :132人目の素数さん:2017/09/30(土) 22:26:24.82 ID:YM5SrsaA.net
>>484

lim sup a_n, lim sup b_n の一方が +∞ で、他方が +∞ または有限ならば不等式は成り立つ

とだけ書けば十分なはずですが、わざわざ以下のように問題の解答には不必要な余計なことまで書いていますね。

「lim sup a_n, lim sup b_n の一方が +∞ で、他方が +∞ または有限ならば両辺とも +∞」

486 :132人目の素数さん:2017/09/30(土) 22:55:51.35 ID:YM5SrsaA.net
>>484

は松坂和夫著『解析入門1』に載っている問題およびその解答の一部です。

(1)
a_n = (-1)^n * n
b_n = -a_n

(2)
a_n = 0 if n ≡ 1 (mod 2)
a_n = n if n ≡ 0 (mod 2)
b_n = -a_n

という反例がありますので、松坂和夫さんは間違っていますね。

問題の解答には必要ではない余計なことを書いて失敗しましたね。

487 :132人目の素数さん:2017/09/30(土) 23:06:55.85 ID:K8Vrcy/A.net
反例になってないんですが

488 :132人目の素数さん:2017/09/30(土) 23:18:24.31 ID:YM5SrsaA.net
>>484

こういうのは質の悪い間違いですよね。

おかげで、時間を無駄にしました。

489 :132人目の素数さん:2017/09/30(土) 23:23:36.24 ID:M3CBbzup.net
ひたすら微積の本を読んでることが一番の無駄なのに

490 :132人目の素数さん:2017/09/30(土) 23:32:10.25 ID:cawMiVwS.net
反応するのがばからしいことは分かった

491 :132人目の素数さん:2017/09/30(土) 23:39:12.59 ID:QFGPsK6W.net
>>475

問題の領域は、|x|≦ 1,|y|≦1,|z|≦1 で定まる一辺の長さが2の立方体に含まれる。

一方、|x|≦ 1 - log(3)/2n,|y| ≦ 1 - log(3)/2n,|z|≦ 1 - log(3)/2n により定まる立方体は
 x^(2n)≦ 1/3,y^(2n)≦ 1/3,z^(2n)≦ 1/3 ゆえ、問題の領域に含まれる。

∴ 8 > V_n > 8{1-log(3)/2n}^3 > 8{1 - 3log(3)/2n}→ 8 (n→∞)

* (1-ε)^(2n)≧ e^(-2nε)を使った。

492 :132人目の素数さん:2017/09/30(土) 23:43:45.72 ID:QFGPsK6W.net
>>491 訂正

* (1-ε)^(2n)≦ e^(-2nε)を使った。

493 :132人目の素数さん:2017/10/01(日) 00:02:21.33 ID:DkFaFrBo.net
こういう頓珍漢なこと言うだけ言って消える奴、極めて不愉快だからいなくなってほしい

494 :132人目の素数さん:2017/10/01(日) 01:57:43.24 ID:1bn6Tflu.net
>>486
松坂和夫の本なんか読むのは時間のムダ

495 :132人目の素数さん:2017/10/01(日) 02:22:58.08 ID:DkFaFrBo.net
ずーっと微積の似たような本読み続けるの、マジで時間の無駄でしかない
学生じゃなくてきっとクッソ暇な人なんだろうな

496 :132人目の素数さん:2017/10/01(日) 05:47:41.44 ID:DJQ/O/L3.net
数学は一種の魔術なのでしょうか?誰か教えてください。お願いします。

497 :132人目の素数さん:2017/10/01(日) 05:55:19.00 ID:DJQ/O/L3.net
ISのカリフとアメリカ合衆国大統領はどっちの方が凄いですか?

498 :132人目の素数さん:2017/10/01(日) 05:55:19.17 ID:DJQ/O/L3.net
ISのカリフとアメリカ合衆国大統領はどっちの方が凄いですか?

499 :132人目の素数さん:2017/10/01(日) 08:18:13.04 ID:cje2lbG+.net
>>486

松坂和夫さんは、一見、丁寧な教科書作りをしているように見えて、実際は、
この問題の解答で分かるように、ちゃんと検証もせずに、なんとなく正しそう
だからそう書くという人なんですね。

500 :132人目の素数さん:2017/10/01(日) 08:22:17.27 ID:cje2lbG+.net
松坂和夫さんの本に自明と書いてあれば、本当に自明だろうと思ってしまいますよね。
あれだけ丁寧な教科書作りをしている松坂和夫さんがそう書くのだからと思ってしまいますよね。

でも、たまに、そうじゃないことがあります。

そういうむらは非常に質が悪いですね。

501 :132人目の素数さん:2017/10/01(日) 10:20:51.23 ID:cje2lbG+.net
基幹講座 数学 微分積分
砂田 利一
固定リンク: http://amzn.asia/37V1lE2

↑10月10日に発売予定ですね。
この本はどうですかね?

502 :132人目の素数さん:2017/10/01(日) 10:22:17.42 ID:cje2lbG+.net
第1章 準備
第2章 数列の収束
第3章 実数の「実相」
第4章 無限級数の収束
第5章 関数の連続性と微分可能性
第6章 積分
第7章 関数列の収束
第8章 多変数関数

理工系の学生が数学を学ぶ際に幹となるべきものをまとめた教科書。
奇をてらわず正攻法で、体系的に王道を歩む、骨太の内容・構成。
数学者かつ教育者である砂田利一先生による教科書を、満を持して
刊行する。

まず、「数学の文法」の観点から、集合の基礎やイプシロンデルタを
解説。そして、関数列の収束や、多変数関数の微積分まで、丁寧に
進む。歴史の流れの中で、微分積分が数学や自然科学全体のどこに
位置し、どのような拡がりがあるのか、を意識した。

問や章末の課題も充実。詳しい解答を巻末に付した。

大学1年生のみならず、上級生、大学院生、学者、教員まで、幅広く
読まれる書籍である。

503 :132人目の素数さん:2017/10/01(日) 10:55:19.00 ID:dBoavXS/.net
あぼーんがいっぱい

504 :132人目の素数さん:2017/10/01(日) 11:39:31.55 ID:DJQ/O/L3.net
世親とガウスはどっちの方が頭が良いですか?

505 :132人目の素数さん:2017/10/01(日) 14:02:56.28 ID:1bn6Tflu.net
>>500
死人に鞭打つようで気が引けるが
松坂和夫の書いたものはゴミだよ

506 :132人目の素数さん:2017/10/01(日) 14:26:07.93 ID:wWmWOnTK.net
>>505
えー、数学読本とか良い本だと思うけど

507 :132人目の素数さん:2017/10/01(日) 14:37:32.88 ID:1bn6Tflu.net
>>506
その本は読んでないや。自分が目を通したのは集合とか位相を扱った本。
丁寧に書いているようでいて丁寧さのポイントがずれてるんだよね

508 :132人目の素数さん:2017/10/01(日) 16:32:12.42 ID:RBDelyZL.net
>>486
何で反例?

509 :132人目の素数さん:2017/10/01(日) 16:54:42.70 ID:56k9BbQU.net
>>508
解答(松坂先生の証明)に対する反例だと思うよ

510 :132人目の素数さん:2017/10/01(日) 17:28:36.79 ID:RBDelyZL.net
>>509
limsup an=limsup bn=+∞
じゃないの?

511 :132人目の素数さん:2017/10/01(日) 17:29:27.81 ID:RBDelyZL.net
あーわかった「両辺とも」ね

512 :132人目の素数さん:2017/10/01(日) 17:35:18.10 ID:A/Zq+bDC.net
誤植の一つ二つでここまでネチネチ言われたんじゃたまらんな

513 :132人目の素数さん:2017/10/01(日) 17:57:47.79 ID:j43nP+d7.net
http://zinziroge.hatenablog.com/entry/2015/01/19/211634
これの

H はn倍しても等価なので最後の行は削除できる。

っていうのがわからないんですけど、なんでn倍して等価なら行を削除できるんです?

514 :132人目の素数さん:2017/10/01(日) 18:09:19.34 ID:ZeWOicuP.net
今スマホから書き込んでるんでて気づいたんだけど
スマホで指数が入力できる!

3⁴

これって周りの人も指数に見えてる?

515 :132人目の素数さん:2017/10/01(日) 18:40:28.33 ID:y4yznyGC.net
見えてます
iPhoneですか?やり方知りたいです

516 :132人目の素数さん:2017/10/01(日) 19:23:00.78 ID:53IBgaOl.net
数列です。規則が分かりません。

1,2,4,7,8,?,19,?,…

19の前と後は幾つになりますかね?
どなたかよろしくお願いします。
m(__)m

517 :132人目の素数さん:2017/10/01(日) 19:27:42.70 ID:E82kjJKT.net
>>514
3^4 にみえる

518 :132人目の素数さん:2017/10/01(日) 20:34:18.80 ID:cje2lbG+.net
>>516

1, 2, 4, 7, 8, 14, 19, 34, 40, 55, 178, 251, 314, 410, 982, 1198, 1604, 2486, 2642, 3928, 19231, 24874, 28850

(19*10^k + 467)/9 が素数であるような k ではないでしょうか?

http://oeis.org/search?q=1%2C2%2C4%2C7%2C8%2C14%2C19&sort=&language=english&go=Search

519 :132人目の素数さん:2017/10/01(日) 20:39:10.37 ID:RBDelyZL.net
>>516
1+1=2 2×2=4
4+3=7 7+1=8 8×2=16
16+3=19 19+1=20 20×2=40
40+3=43 43+1=44 44×2=88

520 :132人目の素数さん:2017/10/01(日) 20:40:40.07 ID:RBDelyZL.net
1+1=2 2×2=4 4+3=7
7+1=8 8×2=16 16+3=19
19+1=20 20×2=40 40+3=43
43+1=44 44×2=88 44+3=47

521 :132人目の素数さん:2017/10/01(日) 20:56:12.46 ID:0BQTmUSh.net
皆さん、ありがとうございました!
m(._.)m

522 :132人目の素数さん:2017/10/01(日) 22:27:03.16 ID:mkh5ZHbD.net
高校生です。二項定理の解き方を教えてくださいお願いします
https://i.imgur.com/DWekR2A.jpg

523 :132人目の素数さん:2017/10/01(日) 22:27:33.72 ID:mkh5ZHbD.net
>>522
6aの(1)を教えてくださいすみません

524 :132人目の素数さん:2017/10/01(日) 22:55:48.60 ID:wUq8QzHF.net
サピックスの小学4年生に負けてるぞ

525 :132人目の素数さん:2017/10/01(日) 22:59:46.75 ID:nFl37AAG.net
勉強する前に問題解こうと思うなよ

526 :132人目の素数さん:2017/10/02(月) 01:15:09.61 ID:sn5N4/3r.net
中学生の子に教えていて『あれ?』と思ったんですけど
n次式の定義として
「項に分けて最大の次数がnならn次式」
とありますが、たとえば
y = x + 1/x
は1次式でOKですか?

527 :132人目の素数さん:2017/10/02(月) 01:16:41.81 ID:sn5N4/3r.net
あ、ちょっと訂正します

x + 1/x     は一次式で
y = x + 1/x   は一次関数 でOKですか?

528 :132人目の素数さん:2017/10/02(月) 01:30:08.53 ID:3MmmLRV7.net
次数は多項式について定義されますから、分数が含まれるものの次数は考えません

529 :132人目の素数さん:2017/10/02(月) 06:16:53.29 ID:ckEyy4yN.net
ツォンカパとベルンハルト・リーマンはどっちの方が天才ですか?

530 :132人目の素数さん:2017/10/02(月) 06:29:02.18 ID:ckEyy4yN.net
test

531 :132人目の素数さん:2017/10/02(月) 10:57:37.41 ID:Xsro3yo0.net
>>526
あなた…何も教えないほうがいいよ

532 :132人目の素数さん:2017/10/02(月) 11:11:55.90 ID:sn5N4/3r.net
>>528
ありがとうございます。

多項式関数とは
f(x)=a[0]x^n+a[1]x^(n−1)+……+a[n−1]x+a[n](nは0以上の整数)
で表される関数,すなわち整式のこと。

ということだったんですね。初めて知りました・・・。

533 :132人目の素数さん:2017/10/02(月) 12:23:44.54 ID:XfmB4Ipg.net
整関数と紛らわしいから整式という言葉はあまり使わない方がいい(多項式関数とは整式のこと、とするならそもそも「整式」の定義は何?)

534 :132人目の素数さん:2017/10/02(月) 12:41:07.69 ID:SsJP5a6n.net
小中学生に教えるとなると、学校数学専門用語も覚えないといけないんだぜ
特に学校についていけてない子は、学校と同じ言葉を使ってやらないとすぐに混乱する

535 :132人目の素数さん:2017/10/02(月) 12:47:22.32 ID:4RaO+alS.net
>>515
「⁴」は「4」とは別の文字コードなんよ

536 :132人目の素数さん:2017/10/02(月) 13:20:22.71 ID:1/e5G/IP.net
東大文系数学で確率の問題が初見で解けたことがほとんど無い。何かいい演習ありますか?

537 :132人目の素数さん:2017/10/02(月) 14:14:42.52 ID:BRdVxhBp.net
別の分野を詰めて、確率は当日捨てる

538 :132人目の素数さん:2017/10/02(月) 19:05:33.45 ID:bBu/uZ2B.net
Rick Durrett著『Probability Theory and Examples』に以下の記述があります。

定理1.1.1

μ を (Ω, F) 上の測度とする。

(iv) Coninuity from above.

If A_i ↓ A (i.e. A_1 ⊃ A_2 ⊃ ... and ∩ A_i = A), with μ(A_1) < ∞ then μ(A_i) ↓ μ(A)


なぜ、 μ(A_1) < ∞ という仮定が書いてあるのでしょうか?

μ の定義では、 μ : F → R と書かれているので、 μ(A_1) < ∞ は当然です。

539 :subo:2017/10/02(月) 19:17:46.46 ID:DNso6s4Z.net
部屋に4X4のマスの盤があります。
悪魔はA、Bを部屋の外に待機させ、この盤にコインをランダムに置いていきます
尚、各マスに置けるコインは一つです
コインを配置したあとAを部屋の中に入れ1以上16以下の整数のどれか一つを告げます
Aは4X4のマスの盤の上に
@コインが置いていないマスに一つだけコインを置く
Aコインが置いてあるマスから一つだけコインを取り除く
のいずれかの操作を一回だけ行います
その後Bを部屋の中に入れ、Bは盤の様子を見てAに告げられた整数を当てます

A,Bはどのような戦略を取ればよいでしょうか?
尚A、Bは初めの配置を知りません
ルールを知った上で開始前に戦略を打ち合わせることができます

540 :subo:2017/10/02(月) 19:29:28.17 ID:DNso6s4Z.net
>>539
についてコインの裏、表は関係ありません

碁の黒石みたいに

541 :132人目の素数さん:2017/10/02(月) 20:08:44.99 ID:Dx9Bw9Mv.net
分からないんですね(笑)

542 :subo:2017/10/02(月) 20:18:35.21 ID:DNso6s4Z.net
パズルスレは過疎ってるし、とりあえず数学スレで投稿して解いてもらおうと

思ってな、問題に不備はないと思うけど

543 :132人目の素数さん:2017/10/02(月) 21:10:54.64 ID:sWP39mzZ.net
マルチ 残念

544 :subo:2017/10/02(月) 21:12:16.84 ID:DNso6s4Z.net
少しは考えろ

545 :132人目の素数さん:2017/10/02(月) 21:13:00.44 ID:sj+1NUGP.net
悪魔の役割がわかんない

546 :132人目の素数さん:2017/10/02(月) 21:44:36.56 ID:sWP39mzZ.net
>>542
不備だらけ

547 :subo:2017/10/02(月) 21:48:17.80 ID:DNso6s4Z.net
>>545
悪魔は俺だ、Bが俺がAに言った整数を答えられなければA、B両方とも処刑になるんだ

処刑されたくなければよく考えて100%当てることができる戦略をかんがえるんだぞ

548 :subo:2017/10/02(月) 21:49:15.72 ID:DNso6s4Z.net
>>546

どこが不備なんだ?

549 :subo:2017/10/02(月) 22:10:36.70 ID:DNso6s4Z.net
コインは黒石に変更する、悪魔はマスにまったく置かないか、全部のマス(16)
に置くことも出来る

でもそれではある意味簡単なので右上から順に番号を1から16に割り振って
例えば2、4、5、6、8、14、15に黒石を置き、13の整数を告げ
Aに一回だけ操作してもらう、そのあとBを呼びその盤面を見せてAにいった整数
を当ててもらうんだ、言うのは一回限りだ

550 :132人目の素数さん:2017/10/02(月) 22:34:08.20 ID:32ZOrIhi.net
確率の問題です。
2個並列の電池があります。
1個の電池が100時間で切れる確率は1%。
ただし1個が切れると、もう片方により負荷がかかるので、残った2個目の電池が100時間で切れる確率は2%に上がります。
さて、2個並列の電池が100時間のうちに@0個切れるA1個切れるB2個切れる確率をそれぞれ求めよ。

551 :132人目の素数さん:2017/10/02(月) 22:50:19.76 ID:bBu/uZ2B.net
松坂和夫著『解析入門1』に以下の定理があります。

定理11

Σa_n を条件収束する級数とし、 α, β を

-∞ ≦ α ≦ β ≦ +∞

を満たす R^- の元とする。このとき、 Σa_n の適当な配列がえ級数 Σ a'_n を
作って、その部分和 s'_n が

lim inf s'_n = α, lim sup s'_n = β

を満たすようにすることができる。


いかにも面倒くさそうな定理なので、嫌な予感がしたのですが、それが的中しました。
まず、この定理の証明だけは、なぜか説明が丁寧ではなくなっています。面倒くさが
っているなーというのが分かります。その証明ですが、 α, β は -∞, +∞ の場合も
考えなければならないにもかかわらず、何の断りもなしに、途中から、 α, β ∈ R
であると決めてかかっています。

この定理の証明だけがこの本の中で浮いてしまっています。

書くのが面倒で嫌なら書くなと言いたいです。

552 :132人目の素数さん:2017/10/02(月) 23:01:21.59 ID:rUWf3064.net
お前が一番面倒だから二度と現れるな

553 :132人目の素数さん:2017/10/02(月) 23:33:45.92 ID:bBu/uZ2B.net
>>551

今、 Walter Rudin の本をチェックしたところ、松坂和夫さんはおそらく Rudin の本を
丸写ししたということが分かります。

Rudin の本でなされている無意味な仮定もそのまま松坂和夫さんの本に引き継がれて
います。

α, β ∈ R の場合しか証明しないのも Rudin と同じです。

554 :132人目の素数さん:2017/10/02(月) 23:44:28.73 ID:sWP39mzZ.net
±∞の場合は自明だからでしょ。

555 :132人目の素数さん:2017/10/03(火) 00:24:58.67 ID:e/Xev6We.net
>>527
人にものを教える資格がないと思う

556 :132人目の素数さん:2017/10/03(火) 00:44:16.02 ID:lbEugLj1.net
>>555
既に似たような煽りレスがついてるのにもかかわらず、遅レスでまた煽る書き込みするようなあなたに質問です

ある無矛盾な公理系τの任意のモデルに対してある論理式φが常に真となる時、LKにおいてτからφが証明可能であることを示せ、という問題がわかりません

557 :132人目の素数さん:2017/10/03(火) 00:50:06.22 ID:PmCavwN2.net
>>555

その通りだと思う。
527氏が>>527の後、「多項式関数とは・・・初めて知りました」、というような書き込みをしているのが実に嘆かわしい。

おそらく、多項式関数と多項式の違いも理解していないのだろう。
「整」という漢字と「式」という漢字を知っていることから「整式」の定義を確認することなく分かっているつもりになっていることがありあり。

558 :132人目の素数さん:2017/10/03(火) 00:51:05.71 ID:lbEugLj1.net
>>557
>>556よろしくお願いします

559 :132人目の素数さん:2017/10/03(火) 00:52:49.56 ID:ss3RJEfB.net
>>557
あなたの日本語は変なので小学1年生からやり直すことをお勧めします。

560 :132人目の素数さん:2017/10/03(火) 00:55:29.00 ID:PmCavwN2.net
>>558
あなたが何がわからないのかがわからないので答えようがありません。

>>559
読解力を身につけましょう。

561 :132人目の素数さん:2017/10/03(火) 00:57:12.91 ID:ss3RJEfB.net
数学って国語が分からないと話にならないからなあ…
アルファベットとアラビア数字が読み書きできるだけで威張ってる連中ばっかなんだよねここは

562 :132人目の素数さん:2017/10/03(火) 01:01:11.47 ID:lbEugLj1.net
>>560
どうやって証明するかがわかりません
試しにあなたが証明して見せてくれませんか?

563 :132人目の素数さん:2017/10/03(火) 01:02:45.83 ID:PmCavwN2.net
>>561
あなたがその一人であることは.>>559からよ〜く分かりますよ。

564 :132人目の素数さん:2017/10/03(火) 01:04:40.65 ID:PmCavwN2.net
>>562
おやおや、これまでの記述程度ではあなたの基礎学力がどの程度なのかもわたしにはわからないので、答えようがありません。
アシカラズ

565 :132人目の素数さん:2017/10/03(火) 01:06:17.27 ID:lbEugLj1.net
>>564
私のことはどうでもいいですから、あなたの回答を見たいです

まさかとは思いますが、わからないんですか?

566 :132人目の素数さん:2017/10/03(火) 01:08:28.57 ID:PmCavwN2.net
勿論ですよ。
あなたへの回答はわかりません。
そんなことも分かりませんかこのやりとりで、ね、おバカさん。

567 :132人目の素数さん:2017/10/03(火) 01:09:52.97 ID:lbEugLj1.net
>>566
とりあえず、任意の整合的な理論に対して、それを充足させるような少なくとも一つの解釈が存在する、ということはわかっているつもりです

このくらいの情報があれば教えてくださいますか?

568 :132人目の素数さん:2017/10/03(火) 01:11:45.94 ID:PmCavwN2.net
ダ〜メ
数学に関して共通認識を共有しているとはとても思えないので。

569 :132人目の素数さん:2017/10/03(火) 01:12:36.29 ID:lbEugLj1.net
>>568
そのようですね
あなたには形式論理の知識が全くないようです

素直にわかりません、と認めたらどうなんですか?

570 :132人目の素数さん:2017/10/03(火) 01:14:03.89 ID:ss3RJEfB.net
>>567
あなたには児童文学がオススメです。
簡潔で分かりやすく日本語をお勉強できます^^

571 :132人目の素数さん:2017/10/03(火) 01:14:38.14 ID:lbEugLj1.net
>>570
どうやら、あなたも>>556の意味すらわからない低レベルな人のようですね

572 :132人目の素数さん:2017/10/03(火) 01:15:51.67 ID:ss3RJEfB.net
>>571
こんな板に張り付いてわずか十数秒で返信とはご苦労様
普通はその努力を違う所に向けるけどね^^

573 :132人目の素数さん:2017/10/03(火) 01:16:53.45 ID:lbEugLj1.net
>>572
わからないようなレベルの低い人が、こんなところで何をしてるんですか?
ここは数学板ですよ?

574 :132人目の素数さん:2017/10/03(火) 01:19:24.91 ID:PmCavwN2.net
御冗談を・・・

575 :132人目の素数さん:2017/10/03(火) 01:24:40.92 ID:lbEugLj1.net
なら証明を教えてください

576 :132人目の素数さん:2017/10/03(火) 01:30:03.91 ID:PmCavwN2.net
数学板をダメにしてしまったオマエになんか、仮に解答が分かったとしても金輪際教えるもんか、というある意味の老婆心を理解してね。

これでおしまい。

577 :132人目の素数さん:2017/10/03(火) 01:31:10.06 ID:lbEugLj1.net
質問者を蔑むようなレスをネチネチと書き込み続けることは数学板をダメにするレスではないんですか?

578 :132人目の素数さん:2017/10/03(火) 01:33:17.66 ID:lbEugLj1.net
てか、やっぱりわかってなかったんですね(笑)

>>576
>仮に解答が分かったとしても

579 :132人目の素数さん:2017/10/03(火) 01:38:33.29 ID:PmCavwN2.net
ほら、引っ掛かった。

580 :132人目の素数さん:2017/10/03(火) 07:37:35.55 ID:o8kwb0ZL.net
公理系芸が流布しすぎてて本物かどうかもうわかんねーな

581 :132人目の素数さん:2017/10/03(火) 09:19:28.87 ID:iNUdJbtw.net
以下は、松坂和夫著『解析入門1』の問題です。

Σa_n が絶対収束するならば Σa_n^2 は収束することを示せ。
この逆は成り立つか?

Σa_n が収束するならば Σa_n^2 は収束することを示せ。

という問題にしなかったのか理解に苦しみます。

この問題はおそらく松坂和夫さんのオリジナルの問題だと思います。

582 :132人目の素数さん:2017/10/03(火) 09:20:04.42 ID:iNUdJbtw.net
以下は、松坂和夫著『解析入門1』の問題です。

Σa_n が絶対収束するならば Σa_n^2 は収束することを示せ。
この逆は成り立つか?

なぜ、 Σa_n が収束するならば Σa_n^2 は収束することを示せ。

という問題にしなかったのか理解に苦しみます。

この問題はおそらく松坂和夫さんのオリジナルの問題だと思います。

583 :132人目の素数さん:2017/10/03(火) 09:21:20.76 ID:iNUdJbtw.net
あ、やっぱり絶対収束じゃないとダメですね。

584 :132人目の素数さん:2017/10/03(火) 09:23:07.97 ID:iNUdJbtw.net
ということで、以下の問題を出題させていただきます:

Σa_n は収束するが、 Σa_n^2 は収束しないような数列 (a_n) の例を挙げよ。

585 :132人目の素数さん:2017/10/03(火) 09:28:44.33 ID:iNUdJbtw.net
あ、簡単でしたね。

Σ(-1)^n * 1/(n^α)

1/2 < α < 1

586 :132人目の素数さん:2017/10/03(火) 09:29:53.05 ID:iNUdJbtw.net
訂正します:

あ、簡単でしたね。

Σ(-1)^n * 1/(n^α)

0 < α < 1/2

587 :132人目の素数さん:2017/10/03(火) 10:27:32.87 ID:/5TlxbPt.net
>>546
不備ではないよ
2マスで1,2を当てるなら
○○1
○×2
×○1
××2
と決めておきAは教えられた数に従って石を変更すればいい

588 :132人目の素数さん:2017/10/03(火) 10:28:49.01 ID:KZxkgPrQ.net
>>582
そんな勘違いをするところに
解析を「感覚では」理解できていない
ことが如実に表れている。

細かい論理ばっかり追っかけてるから
条件収束じゃ無理だということが
直感的に分からないんだよ。
本の読み方を改めた方がいいよアンタ。


それから、人にイチャモンつけといて、
それが自分の誤りに起因するものだった
場合は、謝罪の一言があって然るべきだと
思うのだが、キミはヒトとして未熟だとでも
主張したいのかね?

589 :132人目の素数さん:2017/10/03(火) 10:31:49.48 ID:/5TlxbPt.net
2^16のパターンを16通りに分類してどのパターンからも16通りのどの分類にも距離が1になるパターンが存在するようにせよという問題

590 :132人目の素数さん:2017/10/03(火) 14:59:56.33 ID:iNUdJbtw.net
吉田伸生さんの微分積分の本のどこがいいのかさっぱり分かりません。

これほど読みにくい本も珍しいと思います。

読みにくさの原因は、著者が書くには楽であるが、読者には非常に分かりにくい表現ばかり
使っているからです。

例えば、

交代級数の定義が以下です。

a_n = (-1)^n |a_n|
|a_n| ≧ |a_(n+1)|
a_n → 0

これって分かりにくいと思いませんか?

何を意味しているのかいちいち少し考えなくてはいけません。

591 :132人目の素数さん:2017/10/03(火) 15:03:25.76 ID:o8kwb0ZL.net
>>590
分からないんですね(笑)

592 :132人目の素数さん:2017/10/03(火) 15:06:36.46 ID:iNUdJbtw.net
プログラムのソースコードにも、確かに間違いではないが、非常に分かりづらいソースコード
というものがあります。プログラミングした人の意図が伝わりにくいソースコードです。

そういう分かりづらいソースコードを書く人のようなもんですね>吉田伸生さん。

593 :132人目の素数さん:2017/10/03(火) 15:08:24.35 ID:o8kwb0ZL.net
この書き込みをマルチして何を伝えたいんですかね...

594 :132人目の素数さん:2017/10/03(火) 15:12:57.32 ID:iNUdJbtw.net
式だけで定義するとしても、

a_n * a_(n+1) ≦ 0
|a_n| ≧ |a_(n+1)|
a_n → 0

のほうが分かりやすいですね。

595 :132人目の素数さん:2017/10/03(火) 17:50:21.36 ID:KZxkgPrQ.net
目糞鼻糞だろ

それはそれとして
>>583 の件について
謝罪はないの?

プライドが邪魔して
謝れないのかwww

596 :132人目の素数さん:2017/10/03(火) 19:22:12.98 ID:95sCEXGx.net
>>582
これはヒドイ
これだけ微積の本を読み漁っていてこれは
数学的センスが恐ろしくない

597 :132人目の素数さん:2017/10/03(火) 19:24:56.09 ID:95sCEXGx.net
>>590
?普通に分かり易いが

598 :132人目の素数さん:2017/10/03(火) 19:39:28.04 ID:uvS8dfqz.net
知恵袋で質問されていた因数分解が解けない、悔しい・・・
解があるのかどうかの判断もできない自分が情けない
https://detail.chiebukuro.yahoo.co.jp/qa/question_detail/q12180218427
x^2+ y^3+y+xy

599 :132人目の素数さん:2017/10/03(火) 19:51:30.28 ID:XFJiChts.net
符号が反転する性質がわかりやすいのは
>>590

600 :132人目の素数さん:2017/10/03(火) 20:32:28.00 ID:20sbtdIj.net
http://www.wolframalpha.com/input/?i=factor+x%5E2+%2B+y%5E3+%2B+x*y+%2B+y

601 :132人目の素数さん:2017/10/03(火) 21:20:49.95 ID:uvS8dfqz.net
>>600
よくわかんねぇ

602 :132人目の素数さん:2017/10/03(火) 22:14:43.49 ID:k0v983h1.net
http://www.imojp.org/challenge/old/jjmo15yq.html

これの12番です

603 :132人目の素数さん:2017/10/03(火) 23:40:30.80 ID:/5TlxbPt.net
>>598
x^2+xy+y^3+yが因数分解できたとすると(x+A)(x+B)でA,Bはyの多項式
A+B=y
AB=y^3+y
A,Bの次数の和が3だからA,Bの次数は異なりA+Bは高い方の次数に一致
それが1次になることはあり得ない

604 :132人目の素数さん:2017/10/03(火) 23:43:07.50 ID:/5TlxbPt.net
次数の問題で係数の問題でないから標数はどうでもいい

605 :132人目の素数さん:2017/10/04(水) 01:22:34.38 ID:MEHx2v7h.net
>>603
つまり、これが出題されている問題集側のミスか、
問題集からの写し間違えではないというのが嘘なのか

いずれにせよ解なしか
正直 >>603で説明されたことを理解しきれてないがそういうことか

606 :132人目の素数さん:2017/10/04(水) 01:42:10.02 ID:uJQ5xxL4.net
解の公式使って無理矢理やればいいんじゃない?(なげやり)

607 :132人目の素数さん:2017/10/04(水) 02:06:33.76 ID:O+2F7sG1.net
c ∈ [a, b], lim_[c →∞] f(x) = ∞
となるが、
∫_a^b f(x) dx < ∞
となるような例ってありますか?

608 :132人目の素数さん:2017/10/04(水) 04:34:54.69 ID:qAqYX8Os.net
間違えました。正確には
c ∈ (a, b), lim_[c →∞] f(x) = ∞
となるが、
∫_a^b f(x) dx < ∞
となるような例ってありますか?

609 :132人目の素数さん:2017/10/04(水) 05:38:24.44 ID:LDgcmAWi.net
多分まだ書き方間違えてると思う
意味の分からないリミット

610 :132人目の素数さん:2017/10/04(水) 06:47:15.51 ID:kgS25JCa.net
お前らこれら全部解けるか?
http://www.ms.u-tokyo.ac.jp/kyoumu/docs/27a%201203.pdf
http://www.ms.u-tokyo.ac.jp/kyoumu/docs/27b%201203.pdf
http://www.ms.u-tokyo.ac.jp/kyoumu/20160210a.pdf
http://www.ms.u-tokyo.ac.jp/kyoumu/20160210b.pdf
http://www.ms.u-tokyo.ac.jp/kyoumu/a20170524.pdf
http://www.ms.u-tokyo.ac.jp/kyoumu/b20170524.pdf

611 :132人目の素数さん:2017/10/04(水) 06:51:11.41 ID:XgFrbgQj.net
マルチガイジ

612 :132人目の素数さん:2017/10/04(水) 10:02:46.88 ID:ZqxLrpzI.net
>>590
吉田伸生さんの本を褒めてる人など見たことも聞いたこともないけど
この交代級数の定義は明快ですね

613 :132人目の素数さん:2017/10/04(水) 10:13:17.09 ID:ZlQw24SJ.net
松坂和夫著『解析入門2』を読んでいます。

リーマン積分についてですが、

「不連続点が無限に存在しても、それらを、長さの和が任意に小なる有限個の
区間の和集合のうちに包含させ得るならば、やはり f は積分可能である。」

という定理が書いてあります。

杉浦光夫著『解析入門I』には書いてありそうですが、他にこの定理の証明が書いてある
本はありますか?

614 :132人目の素数さん:2017/10/04(水) 10:14:16.68 ID:ZlQw24SJ.net
小平邦彦さんの本には書いてありませんでした。

615 :132人目の素数さん:2017/10/04(水) 10:15:01.41 ID:ZlQw24SJ.net
藤原松三郎さんの本にも書いてありませんでした。

616 :132人目の素数さん:2017/10/04(水) 10:54:59.13 ID:ZlQw24SJ.net
>>613

の定理の証明ですが、簡単なアイディアに基づいているのだと思われますが、
分かりにくいですね。

617 :132人目の素数さん:2017/10/04(水) 12:31:01.63 ID:ZlQw24SJ.net
>>613

の定理の証明ですが、重大な誤りがあります。

618 :132人目の素数さん:2017/10/04(水) 12:36:08.73 ID:ZlQw24SJ.net
証明のすぐ後のところに、注意として、

f がリーマン積分可能であるための必要十分条件は、不連続点の集合 E の
ルベーグ測度が 0 であることである。

ということが書いてあります。

>>613

の定理は中途半端な定理であるため、他の本には書いていないのではないでしょうか?

そこで、松坂和夫さんが自分で証明を書いたのではないでしょうか?

619 :132人目の素数さん:2017/10/04(水) 12:58:53.69 ID:QuS0Ynfq.net
>f がリーマン積分可能であるための必要十分条件は、不連続点の集合 E の
>ルベーグ測度が 0 であることである。
ピーマン積分でも考えてるのかw

620 :132人目の素数さん:2017/10/04(水) 13:00:41.92 ID:ZlQw24SJ.net
定理5

f は区間 [a, b] で有界であるとし、 [a, b] における f の不連続点の集合を E とする。任意の ε>0 に対し、

a ≦ u_1 < v_1 < u_2 < v_2 < … < u_s < v_s ≦ b,

Σ(v_j - u_j) from j = 1 to j = s < ε

を満たす有限個の点 u_j, v_j (j = 1, …, s) を適当にとれば、 E ∩ (a, b) の点はすべて、
開区間 (u_1, v_1), …, (u_s, v_s) の和集合に含まれると仮定する。そのとき、 f は [a, b]
で積分可能である。

621 :132人目の素数さん:2017/10/04(水) 13:01:01.28 ID:ZlQw24SJ.net
証明

仮定に述べた u_j, v_j は、もし a ∈ E ならば a = u_1, また b ∈ E ならば v_s = b で
あるようにとることができる。以下そのように u_j, v_j をとったと仮定する。

さて、 [a, b] から (u_1, v_1), …, (u_s, v_s) の和集合をとり除いた集合を K とする。
K は有限個の閉区間の和集合で、各閉区間において f は連続、したがって一様連続である。
よって、与えられた ε > 0 に対し、 δ > 0 を、 x, y ∈ K, |x - y| < δ ならば、

|f(x) - f(y)| < ε

となるようにとることができる。

622 :132人目の素数さん:2017/10/04(水) 13:01:21.37 ID:ZlQw24SJ.net
そこで、 [a, b] の分割 P = (x_0, x_1, …, x_n) を次の(1),(2),(3)が満たされるように定める:

(1) u_j, v_j (j = 1, …, s) はすべて P の分点のうちに現れる。
(2) 区間 (u_j, v_j) (j = 1, …, s) に P の分点は現れない。
(3) 分点 x_(i-1) がどの u_j にも一致しないときには、 Δx_i = x_i - x_(i-1) < δ である。

623 :132人目の素数さん:2017/10/04(水) 13:01:39.66 ID:ZlQw24SJ.net
そのとき、 i = 1, …, n を、ある u_j に対して x_(i-1) = u_j となるような i の集合 A と、
どの u_j に対しても x_(i-1) ≠ u_j であるような i の集合 B とに分ければ、

U(P, f) - L(P, f)
=
Σ(M_i - m_i) * Δx_i from i = 1 to i = n
=
Σ(M_i - m_i) * Δx_i for i ∈ A + Σ(M_i - m_i) * Δx_i for i ∈ B

であるが、 [a, b] における |f(x)| の上限を M とすれば、 Σ(M_i - m_i) * Δx_i for i ∈ A において
M_i - m_i ≦ 2*M であるから

Σ(M_i - m_i) * Δx_i for i ∈ A ≦ 2*M*Σ(v_j - u_j) from j = 1 to j = s < 2*M*ε。

624 :132人目の素数さん:2017/10/04(水) 13:01:53.28 ID:ZlQw24SJ.net
一方 i ∈ B ならば M_i - m_i < ε であるから

Σ(M_i - m_i) * Δx_i for i ∈ B < ε*ΣΔx_i for i ∈ B ≦ ε*(b - a)。

よって

U(P, f) - L(P, f) < (2*M + b - a)*ε

である。ここで ε は任意の正数であった。ゆえに f は [a, b] で積分可能である。

625 :132人目の素数さん:2017/10/04(水) 13:03:42.76 ID:ZlQw24SJ.net
以上が松坂和夫さんの証明ですが、


Σ(M_i - m_i) * Δx_i for i ∈ A ≦ 2*M*Σ(v_j - u_j) from j = 1 to j = s


は成り立たないですよね?

626 :132人目の素数さん:2017/10/04(水) 13:40:05.96 ID:/zJIohP0.net
自分で証明できる事を読むのはめんどくさい

627 :608:2017/10/04(水) 14:29:41.81 ID:NT1bhTp2.net
limのところが間違っていました>>608
書き直すと
c ∈ (a, b), lim_[x →c] f(x) = ∞ かつ
x<c なら -∞<f(x) ≦ 0, x> c なら 0 ≦ f(x) < ∞
となる x≠c で連続な関数 f(x) で
∫_a^b f(x) dx < ∞
となるような例ってありますか?

628 :132人目の素数さん:2017/10/04(水) 14:38:07.43 ID:mbPwiodO.net
定理:(Lebesgue)
有界閉区間上の関数 f がリーマン積分可能であるための必要十分条件は、
f が有界であり、なおかつ、f の不連続点の集合のルベーグ測度が 0 であること。

この定理は、まともなルベーグ積分の教科書なら大抵は載っている。
そして、この定理を見つけたのは、ルベーグ積分の創始者であるルベーグ本人である。

ルベーグ本人が、自分の作り上げたルベーグ測度の概念を使って、
「リーマン積分可能である」ということがどういうことなのかを
ルベーグ測度の言葉で特徴づけしたのである。それがこの定理の1つの大きな意義である。
リーマン積分可能であるための条件を、自分が新しく作った積分の道具で特徴づけしてみたいと
思うことは極めて自然であり、実際にルベーグが発見した上記の定理は見事なものである。

この定理を知らない人間にこの定理を紹介すると、すごく怪訝な顔をされて、
>>619 みたいなバカにした反応が返ってくることが多い。
ルベーグ積分を表面的に理解した気になっているだけの連中には、
この定理を見せても何も響くところがなく、むしろ
「この定理はマチガッテイル」という逆向きの直観が働いてしまうようである。

これは大変に嘆かわしいことである。
お前らは一体ルベーグ積分の何を勉強してきたのだと言いたい。
ルベーグ本人が見つけた定理なのに、それを知らない時点で問題外であるばかりか、
百歩譲って知らなかったことには目を瞑るにしても、この定理を初めて見たときの
第一印象が「マチガッテイル」という腐った直観に支配されてしまって何も響くところが無く、
>>619 のようなバカにした反応しかできないようでは、ルベーグ積分の こころ を
何1つとして理解してないと言わざるを得ない。バカタレどもが。

629 :132人目の素数さん:2017/10/04(水) 15:42:58.46 ID:QuS0Ynfq.net
自己紹介乙

630 :132人目の素数さん:2017/10/04(水) 17:13:23.52 ID:yst4hrJp.net
なにこれ、コピペ?

631 :132人目の素数さん:2017/10/04(水) 18:43:12.26 ID:kf833om4.net
ただのやばい人

632 :132人目の素数さん:2017/10/04(水) 19:25:10.32 ID:ZlQw24SJ.net
>>625

Σ(M_i - m_i) * Δx_i for i ∈ A ≦ 2*M*Σ(v_j - u_j) from j = 1 to j = s

などと松坂さんは書いていますが、

(v_j - u_j) ≦ Δx_i

ですよね。

633 :132人目の素数さん:2017/10/04(水) 20:11:19.93 ID:PlCAtIFe.net
畳み込み積分の意味合いがよくわかりません

統計学で畳み込み積分が出てきまして
計算自体はできたのですが、何故このような積分をしているのかがわかりません
独立な2つの変数の項目で出てきたので
2変数→1変数に変換することで計算が楽にできる?などと考えてみtましたが
今いちピンとこないので質問をしてみることにしました

634 :132人目の素数さん:2017/10/04(水) 20:23:12.41 ID:2Kzo39QM.net
身長180cm
ウエスト(直径)50cmの人がいる
雨の角が15度のとき雨が体にかからないためには傘の長さは最小で何cmいるか
https://dotup.org/uploda/dotup.org1355013.jpg

635 :132人目の素数さん:2017/10/04(水) 20:37:03.19 ID:D1mhM0Sk.net
数学者は神の化身なのでしょうか?

636 :132人目の素数さん:2017/10/04(水) 20:47:19.92 ID:D1mhM0Sk.net
身長が無限だったらどんな感じになるのでしょうか?

637 :132人目の素数さん:2017/10/04(水) 22:07:27.01 ID:/zJIohP0.net
>>633
確率変数の和で確率計算してみなよ

638 :132人目の素数さん:2017/10/04(水) 22:34:18.30 ID:j8RLbgl3.net
よろしくおねがいします
https://i.imgur.com/F8eIt64.jpg

639 :132人目の素数さん:2017/10/05(木) 00:40:47.89 ID:pKtIjAZq.net
>>632

x_(i-1) = u_j とすると、 x_i = v_j なので、

Δx_i = (v_j - u_j)

ですね。

なので、

>>625

は成り立ちますね。

640 :132人目の素数さん:2017/10/05(木) 00:49:40.07 ID:pKtIjAZq.net
>>621

仮定に述べた u_j, v_j は、もし a ∈ E ならば a = u_1, また b ∈ E ならば v_s = b で
あるようにとることができる。以下そのように u_j, v_j をとったと仮定する。


この仮定をなぜしているのか分かりませんでしたが、分かりました。
大した理由じゃないんですね。

641 :132人目の素数さん:2017/10/05(木) 00:51:33.62 ID:pKtIjAZq.net
>>616

に「簡単なアイディアに基づいている」と書きましたが、実際にそうですね。

でも言葉で書こうとすると少し面倒ですね。

そういう定理ですね。

642 :132人目の素数さん:2017/10/05(木) 12:48:40.17 ID:valqwWrF.net
Σ[n→∞]n^(-n)=∫[0→1]x^(-x)dx を示してください

643 :132人目の素数さん:2017/10/05(木) 16:42:45.06 ID:WtKKolNw.net
http://www.imojp.org/challenge/old/jjmo15yq.htmlこれを教えてください

644 :132人目の素数さん:2017/10/05(木) 16:43:38.39 ID:WtKKolNw.net
>>643
12番です

645 :132人目の素数さん:2017/10/05(木) 18:04:18.81 ID:4G4knTHb.net
松坂和夫さんの解析入門ですが、他の日本語の微分積分の本と比べて、
同じような定理でもより一般的な定理を書いていますね。

それで、Walter Rudinの本を見てみると、大抵、松坂さんの本に書いてあるより一般的な
定理が書いてあります。

Rudinの本をそのまま丸写ししているだけですね。

楽な仕事ですね。

646 :132人目の素数さん:2017/10/05(木) 18:08:12.57 ID:4G4knTHb.net
例えば、以下の定理です。

定理6

(f_n) は区間 [a, b] で微分可能な関数列で、次の仮定 1), 2) を満たすとする。

1) [a, b] の1点 x^* において数列 (f_n(x^*)) は収束する。

2) 関数列 (f'_n) は [a, b] において一様収束する。

そのとき (f_n) は [a, b] で一様収束し、その極限を f とすれば、 f も [a, b] で微分可能、
[a, b] の任意の点 x において

lim f'_n(x) = f(x)

が成り立つ。

647 :132人目の素数さん:2017/10/05(木) 18:08:55.92 ID:DWW9UCvZ.net
貴方のお仕事は?

648 :132人目の素数さん:2017/10/05(木) 18:11:53.58 ID:4G4knTHb.net
松坂和夫さんの本はすべてこんな調子なのでしょうか?

例えば、『集合・位相入門』も誰かの本を丸写ししているだけなのでしょうか?

649 :132人目の素数さん:2017/10/05(木) 19:38:35.20 ID:SSUnrYpO.net
>>637
返事が遅くなりましたがありがとうございました

650 :132人目の素数さん:2017/10/05(木) 23:10:05.67 ID:X+2fyClp.net
>648

>誰かの本を丸写し

したら、その丸写ししたものの内容が公知の事実ならば別ですが、
その著者の主張であれば、独自性があきらかですから、
それを丸写ししたら、その量にもよりますが、あきらかに著作権法違反
になりますから、松坂先生ともあろう方がそんなことをするとは
考えられませんね。

651 :132人目の素数さん:2017/10/05(木) 23:28:20.52 ID:/Q8CWL1G.net
必死にdisる奴って気持ち悪いな

652 :132人目の素数さん:2017/10/06(金) 00:51:51.72 ID:/nlSdbgW.net
以下のBが言ってる「無」とは「相対無」のことで合っていますか?

A:「「無」は無い。」

B:「当たり前だ。「無」ってのは何も無いことなんだから。」

653 :132人目の素数さん:2017/10/06(金) 01:08:35.47 ID:wMLtLsLn.net
解き方を教えて下さい

https://i.imgur.com/qESsjwE.jpg

654 :132人目の素数さん:2017/10/06(金) 11:21:51.91 ID:/nlSdbgW.net
大天才の中の大天才として生まれたかった・・・・・。
自殺をして大天才の中の大天才に生まれ変わるのを期待した方が良いのかな・・・・・。
大天才の中の大天才とまではいかなくても良いから、
せめて、超余裕で、東京大学理学部数学科卒 → 東京大学大学院数理科学研究科数理科学専攻修士課程修了 →
東京大学大学院数理科学研究科数理科学専攻博士課程修了
というルートを辿れるぐらいの頭脳が欲しかった・・・・・。

655 :132人目の素数さん:2017/10/06(金) 12:54:15.97 ID:+Zf/gMuM.net
>>650

例えば、

R で連続であるが、 R のどの点でも微分不可能な関数の例についてですが、
Rudinの本の例と全く同じです。

そのような関数の存在証明もRudinよりも説明を少し丁寧にしただけです。

656 :132人目の素数さん:2017/10/06(金) 12:54:47.91 ID:3fgNd1CC.net
すでに人生捨ててる奴か

657 :132人目の素数さん:2017/10/06(金) 12:57:21.79 ID:7Isq8GAI.net
もう少し経験を積めば、それはワイエルシュトラスが初めて発見し最も有名になった例だと分かるのにな

658 :132人目の素数さん:2017/10/06(金) 13:01:00.29 ID:+Zf/gMuM.net
>>657

ワイエルシュトラスのオリジナルの例は、藤原松三郎に載っていますが、
それとは違うもっと証明しやすい関数を扱っています。

その関数がRudinと松坂さんの本で全く同じです。

よく見てみたら、松坂さんの本に「次の証明はRudinによる」と書かれていました。

659 :132人目の素数さん:2017/10/06(金) 13:02:58.17 ID:+Zf/gMuM.net
いろいろな本のいいところをブレンドしているのが松坂和夫さんの正体なのかもしれませんね。


他の著者も似たようなもんですかね?

660 :132人目の素数さん:2017/10/06(金) 13:06:38.68 ID:4g6nxihH.net
微積の本にだけはすごく詳しくて、得体の知れない気持ち悪さがある

661 :132人目の素数さん:2017/10/06(金) 13:45:34.22 ID:AUq8I2P8.net
唯の馬鹿アスペだよ

662 :132人目の素数さん:2017/10/06(金) 15:36:56.48 ID:IgHVmcgc.net
>>638

(1)
v(3)=9-3k
速さ|v(3)|が最小0になるのはk=3のとき

(2)
変位≦移動距離であるから変位/移動距離が2になることはない

これは学部の教養科目の力学?
なんか問題の質が悪くないか?


>>653

(1)
分子と分母の偏角の差になる
(e^(j(π/2)))/(e^(j(π/6)))=e^(j((π/2)-(π/6))=e^(j(π/3))
偏角はπ/3

(2)
√(1^2+(√3)^2)=2でくくると
-1-j√3=2(-1/2-j(√3)/2)=2(cos((2π)/3)+jsin((2π)/3))=2e^(j((2π)/3))
偏角は2π/3

こっちは電磁気かな

663 :132人目の素数さん:2017/10/06(金) 16:10:42.49 ID:RqD03f4d.net
微積より先の内容が理解できないからって、とうとう微積レベルの出版物に新規性を求め出したぞコイツ
基礎レベルの話の本なんてサーベイと同じで分かりやすくまとまってるのが第一なのに
微分積分学の研究者にでもなるのかな?

664 :132人目の素数さん:2017/10/06(金) 18:34:49.03 ID:RZa8ZShZ.net
微積の本で無理矢理オリジナリティ出そうとしたら多様体の本になるか、もしくは圏論を前面に出すか微分代数的に展開するかかな

果たしてそのようなものを松阪君が受け入れられるのか

665 :132人目の素数さん:2017/10/06(金) 19:14:45.49 ID:4g6nxihH.net
たぶん理解できないでしょ

666 :132人目の素数さん:2017/10/06(金) 21:37:31.57 ID:+Zf/gMuM.net
級数について質問です。

root testのほうがratio testよりも適用範囲が広いと書いてあるのですが、
root testでは収束性を判定できるがratio testでは判定できないような例
を教えてください。

667 :132人目の素数さん:2017/10/06(金) 22:24:45.96 ID:+Zf/gMuM.net
松坂和夫著『解析入門2』を読んでいます。

root testとratio testもRudinと全く同じです。

他の本には載っていない形なので、丸写しですね。

668 :132人目の素数さん:2017/10/07(土) 00:16:10.65 ID:lmSm3Ibz.net
>>666
an=1+(-1)^n

669 :132人目の素数さん:2017/10/07(土) 08:42:36.55 ID:H/ABkJFx.net
>>668

ありがとうございます。

root testでは収束と判定されるが、ratio testでは判定できない例はないでしょうか?
さらに、収束性が自明ではない例だともっと望ましいです。

670 :132人目の素数さん:2017/10/07(土) 10:40:06.31 ID:cBajcvhO.net
分からないんですね(笑)

671 :132人目の素数さん:2017/10/07(土) 10:53:53.03 ID:c+Yoymsk.net
>>662
>v(3)=9-3k
違う

672 :132人目の素数さん:2017/10/07(土) 10:56:08.41 ID:c+Yoymsk.net
>>662
>変位≦移動距離であるから変位/移動距離が2になることはない


673 :132人目の素数さん:2017/10/07(土) 14:29:14.80 ID:tlBi3LcY.net
全知全能の神と超絶天才数学者はどっちの方が凄いですか?

674 :132人目の素数さん:2017/10/07(土) 15:10:35.95 ID:3ug4S9PE.net
何が違って何が逆なのかくらい書けよ…

675 :132人目の素数さん:2017/10/07(土) 16:28:01.85 ID:R+PbO4IV.net
∫{cos(x)・[sin^2(x)+a・cos^2(x)]^1/2 }/a dx

を宜しくお願いします。

676 :132人目の素数さん:2017/10/07(土) 16:31:18.24 ID:RRHCD2KD.net
>>675
https://www.wolframalpha.com/input/?i=%E2%88%AB%7Bcos(x)*%5Bsin%5E2(x)%EF%BC%8Ba*cos%5E2(x)%5D%5E1%2F2+%7D%2Fa+dx

677 :132人目の素数さん:2017/10/07(土) 17:46:32.91 ID:H/ABkJFx.net
>>669

https://math.stackexchange.com/questions/1708595/root-test-is-stronger-than-ratio-test

678 :132人目の素数さん:2017/10/07(土) 18:00:28.87 ID:c+Yoymsk.net
>>674
解釈が違くて定義が逆

679 :132人目の素数さん:2017/10/07(土) 18:06:25.01 ID:lKxjyQqB.net
だから正しい解釈と正しい定義を書けよ
638のガバガバ問題文からは662の解答以上のものは無いぞ

680 :132人目の素数さん:2017/10/07(土) 18:25:07.38 ID:c+Yoymsk.net
>>679
0点

681 :132人目の素数さん:2017/10/07(土) 18:37:42.74 ID:cBajcvhO.net
ガイジかな

682 :132人目の素数さん:2017/10/07(土) 18:41:05.70 ID:ouwOZ2Xu.net
ちょっと会話が成り立たないかな

683 :132人目の素数さん:2017/10/07(土) 18:42:26.35 ID:c+Yoymsk.net
>>681
ほほー
>>638

>>662
とはね

684 :132人目の素数さん:2017/10/07(土) 18:52:07.05 ID:H/ABkJFx.net
>>669

https://www.maa.org/sites/default/files/0025570x33450.di021200.02p0190s.pdf

685 :132人目の素数さん:2017/10/07(土) 18:59:24.62 ID:c+Yoymsk.net
>>682
解いてみ

686 :132人目の素数さん:2017/10/07(土) 19:06:08.21 ID:cBajcvhO.net
専門板と言えど、やっぱり2chはレベル低いっすね

687 :132人目の素数さん:2017/10/07(土) 19:19:56.32 ID:c+Yoymsk.net
>>686
考えもしない奴には泣けるね

688 :132人目の素数さん:2017/10/07(土) 19:22:50.75 ID:cBajcvhO.net
>>687
考えた結果が>>680ですか?w

689 :132人目の素数さん:2017/10/07(土) 19:43:32.86 ID:nOn1Thvs.net
>>675

s = sin(x)とおくと
(与式)=(1/a)∫√{a+(1-a)ss}ds
 =(s/2a)√{a+(1-a)ss}+(1/2)∫1/√{a+(1-a)ss} ds,

・a>1 のとき
 √{(a-1)/a}s = S とおくと
∫1/√{a-(a-1)ss}ds ={1/√(a-1)}∫1/√(1-SS) dS ={1/√(a-1)}arcsin(S),

・0<a<1 のとき
∫1/√{a+(1-a)ss}ds ={1/√(1-a)}{Log|a+(1-a)ss|+(1-a)s},

・a=1 のとき
(与式)=∫ cos(x)dx = sin(x),

690 :132人目の素数さん:2017/10/07(土) 19:52:16.58 ID:c+Yoymsk.net
>>688


691 :132人目の素数さん:2017/10/07(土) 19:54:03.68 ID:nOn1Thvs.net
>>689 (訂正)

・0<a<1 のとき
∫1/√{a +(1-a)ss}ds ={1/√(1-a)}Log{√[a +(1-a)ss]+ √(1+a)・s}

692 :132人目の素数さん:2017/10/07(土) 19:58:27.37 ID:cBajcvhO.net
>>690
分からないんですね(笑)

693 :132人目の素数さん:2017/10/07(土) 20:13:15.43 ID:c+Yoymsk.net
>>692
煽るしか能がないのは悲しむべきね

694 :132人目の素数さん:2017/10/07(土) 20:23:52.16 ID:H/ABkJFx.net
https://www.maa.org/sites/default/files/0025570x33450.di021200.02p0190s.pdf

数列 (a_n) を以下で定義する。

a_n := (1/2)^n if n ≡ 1 (mod 2)
a_n := (1/2)^(n-2) if n ≡ 0 (mod 2)

明らかに、 Σ a_n は初項 1, 公比 1/2 の等比級数を並べ替えた級数であるから、
2 に収束する。

a_(n+1) / a_n = (1/2)^(n-1) / (1/2)^n = 2 if n ≡ 1 (mod 2)
a_(n+1) / a_n = (1/2)^(n+1) / (1/2)^(n-2) = 1/8 if n ≡ 0 (mod 2)

であるから、

lim sup a_(n+1) / a_n = 2 > 1

であり、

a_(n+1) / a_n < 1 となるような n が無限に存在する(n が偶数のとき

)から

ratio testでは、収束するか発散するか分からない。

695 :132人目の素数さん:2017/10/07(土) 20:26:48.05 ID:H/ABkJFx.net
(a_n)^(1/n) = 1/2 if n ≡ 1 (mod 2)
(a_n)^(1/n) = (1/2)^(1-2/n) if n ≡ 0 (mod 2)

であるから、

lim sup (a_n)^(1/n) = lim (a_n)^(1/n) = 1/2 < 1

となり、 root testにより、収束することが分かる。

696 :132人目の素数さん:2017/10/07(土) 20:31:14.49 ID:tlBi3LcY.net
ピタゴラスとカルロス・スリム・ヘルはどっちの方が凄いですか?

697 :132人目の素数さん:2017/10/07(土) 20:32:40.17 ID:H/ABkJFx.net
>>694

https://imgur.com/RmGc41w.jpg

↑Mathematicaでチェックしました。

698 :132人目の素数さん:2017/10/07(土) 20:55:34.49 ID:cBajcvhO.net
>>693
けど、貴方は分からないんですよね?

699 :132人目の素数さん:2017/10/07(土) 21:01:26.81 ID:c+Yoymsk.net
>>698
これも煽り

700 :132人目の素数さん:2017/10/07(土) 21:04:07.15 ID:cBajcvhO.net
>>699
けど、貴方には分からない(笑)

701 :132人目の素数さん:2017/10/07(土) 21:09:26.69 ID:c+Yoymsk.net
>>700
あらら

702 :132人目の素数さん:2017/10/07(土) 21:11:38.17 ID:c+Yoymsk.net
>>679
ID:3ug4S9PE
ID:lKxjyQqB
は解釈の違いが分かったかな?

703 :132人目の素数さん:2017/10/07(土) 21:15:44.36 ID:cBajcvhO.net
>>702
貴方の書き込みが0点であり、かつエスパーは存在しないので、貴方の考える正しい解釈、定義など誰も分かりませんよ

704 :132人目の素数さん:2017/10/07(土) 21:20:52.18 ID:c+Yoymsk.net
>>703
何で問題文読まないの?それと定義が逆だって何の定義か1つ(2つ)しかあり得ない
何も考える気が無いんじゃどうしようもない
>>638
v'=0v''>0∫vdt=0(2,k)で

705 :132人目の素数さん:2017/10/07(土) 21:21:44.52 ID:c+Yoymsk.net
>>703
ともかく君も0点

706 :132人目の素数さん:2017/10/07(土) 21:26:31.32 ID:cBajcvhO.net
>>704
何が逆なのか、おおよその予想は当然できていますが、貴方が明言しないのは何故ですか?
また、逆である証拠は示せますか?

707 :132人目の素数さん:2017/10/07(土) 21:42:42.94 ID:c+Yoymsk.net
>>706

解いてみて

708 :132人目の素数さん:2017/10/07(土) 21:45:06.38 ID:cBajcvhO.net
>>707
示せないのですか?

709 :132人目の素数さん:2017/10/07(土) 21:47:45.02 ID:c+Yoymsk.net
>>708
解けないようですね

710 :132人目の素数さん:2017/10/07(土) 21:51:05.68 ID:cBajcvhO.net
>>709
貴方も示せないようですね
示せないなら何故逆だと考えたかを教えていただいていいですか?

711 :132人目の素数さん:2017/10/07(土) 21:54:58.36 ID:cBajcvhO.net
>>693>>707>>709が同じ人間の書き込みだと思うと、何か考えてしまうものがありますね

712 :132人目の素数さん:2017/10/07(土) 21:55:03.95 ID:c+Yoymsk.net
>>710
定義ですから

713 :132人目の素数さん:2017/10/07(土) 21:56:45.44 ID:c+Yoymsk.net
>>711
ID:cBajcvhO
全部見てみると
何も解こうとしない
何も考える気が無い
泣けるね

714 :132人目の素数さん:2017/10/07(土) 21:58:18.96 ID:cBajcvhO.net
>>712
何の定義、解釈が逆だと考えたか
何故そう考えかた
教えていただいていいですか?

715 :132人目の素数さん:2017/10/07(土) 23:21:26.72 ID:cBajcvhO.net
そろそろIDが変わる心配をする時間ですね

716 :132人目の素数さん:2017/10/07(土) 23:48:34.75 ID:udEbLB1H.net
閻魔大王と神武天皇はどっちの方が凄いですか?

717 :132人目の素数さん:2017/10/08(日) 00:49:36.44 ID:MpaBQY8Y.net
>>716


神武天皇、加藤、谷川、羽生、渡辺、藤井の順です。

718 :132人目の素数さん:2017/10/08(日) 02:35:38.40 ID:MpaBQY8Y.net
>>694

部分和をとれば

S_n = Σ[k=1,n] a_k < Σ[k=1,n](1/2)^(k-2)= 4 -(1/2)^(n-2)< 4

単調増加かつ上に有界なので収束する。

719 :132人目の素数さん:2017/10/08(日) 08:36:16.66 ID:EPCGZi6J.net
基幹講座 数学 微分積分 (基幹講座数学)
砂田 利一
固定リンク: http://amzn.asia/fZYj7Mc

↑の本はどうですか?

720 :132人目の素数さん:2017/10/08(日) 08:39:35.66 ID:4CCYiw8x.net
気持ち悪い

721 :132人目の素数さん:2017/10/08(日) 12:39:39.69 ID:921rOBNV.net
>>689
本当にありがとうございます。
もう一度、考えてみます。
また質問するかもしれませんが、その時は、宜しくお願いします。

722 :132人目の素数さん:2017/10/08(日) 17:50:56.37 ID:EPCGZi6J.net
a, b を

a ≧ b ≧ 0

を満たす整数とする。


a, b の最大公約数をユークリッドの互除法で求める際、
余りを計算する回数を R(a, b) と書くことにする。

(F_n) は フィボナッチ数列 0, 1, 1, 2, 3, …, とする。

n を F_n ≧ a ≧ b ≧ 0 を満たす整数とするとき、

R(a, b) ≦ n

が成り立つことを示せ。

723 :132人目の素数さん:2017/10/08(日) 18:04:05.55 ID:EPCGZi6J.net
F_n ≧ a > b > 0

を仮定すれば、

R(a, b) ≦ n - 2

が成り立つことを示せ。

n ≧ 3 のとき、 R(F_n, F_(n-1)) = n - 2 が成り立つことを示せ。

724 :132人目の素数さん:2017/10/08(日) 19:16:32.47 ID:fdjrkdb3.net
無になってもう二度と有にならなくて済むのなら今すぐにでも自殺したいが、
そうなれる保障は無いから、なかなか実行に移せない。
そもそも、今が有な時点でまた有になる可能性はあると思う。
一体どうすれば良いんだ・・・・・。

725 :132人目の素数さん:2017/10/08(日) 19:20:28.80 ID:acRVPW/y.net
>>724
生き続けるしかない

726 :132人目の素数さん:2017/10/08(日) 20:28:31.78 ID:EPCGZi6J.net
(3*t - t^2) * sin(t) の t = 0 のまわりでの級数展開における係数を求めよ。

727 :132人目の素数さん:2017/10/09(月) 00:23:40.14 ID:Kt44Cz1i.net
(3t - t^2)sin(t)=(3t - t^2)Σ(-1)^n t^(2n+1)/(2n+1)!
=Σ(-1)^n 3t^(2n+2)/(2n+1)! + Σ(-1)^(n+1) t^(2n+3)/(2n+1)!

728 :132人目の素数さん:2017/10/09(月) 02:02:52.34 ID:t7NhaD6v.net
n - (n - n / Z * Y) * 0.2 = Z
この式で x=200, Y=133, Z=100 であることがわかっています。
nを求める方法を教えてください。

729 :728:2017/10/09(月) 02:03:57.77 ID:t7NhaD6v.net
すみません、訂正させて下さい。
n - (n - n / X * Y) * 0.2 = Z
です。お願いします。

730 :132人目の素数さん:2017/10/09(月) 02:54:53.31 ID:a6Ruo5ry.net
XYn-0.2(XYn-n)=XYZ
XYn-0.2(XY-1)n=XYZ
(XY-0.2(XY-1))n=XYZ
(0.8XY+0.2)n=XYZ
n=XYZ/(0.8XY+0.2)
n=10XYZ/(8XY+2)
n=5XYZ/(4XY+1)

のうちで好きな式にぶちこむ

731 :728:2017/10/09(月) 03:17:52.38 ID:t7NhaD6v.net
>>730
早速のご回答ありがとうございます。
検算してみたのですが、たとえば下から3番目の式ですと

XYZ = 200 * 133 * 100 = 2660000 ・・・(A)
(0.8XY+0.2) = 0.8 * 200 * 133 + 0.2 = 21280.2 ・・・(B)
n = (A) / (B) = 2660000 / 21280.2 = 124.998

これを元の式に当てはめると

124.998 - (124.998 - 124.998 / 200 * 133) * 0.2 = 100

となりますが、この式の計算結果は 116.623 であり 100 ではありません。
検算方法がおかしいのでしょうか?

732 :132人目の素数さん:2017/10/09(月) 03:26:41.02 ID:BMPLILPx.net
n/X*Y (左から計算)

n/(XY)
と勘違いしていた

Xn-0.2(Xn-Yn)=XZ
0.8Xn+0.2Yn=XZ
(0.8X+0.2Y)n=XZ
n=XZ/(0.8X+0.2Y)
n=10XZ/(8X+2Y)
n=5XZ/(4X+1)

のうちで好きな式にぶちこむ

733 :132人目の素数さん:2017/10/09(月) 03:27:29.65 ID:BMPLILPx.net
一番下はn=5XZ/(4X+Y)

734 :728:2017/10/09(月) 03:32:02.83 ID:t7NhaD6v.net
>>732
夜遅くまでお付き合い頂き、ありがとうございます。

XZ = 200 * 100 = 20000 ・・・(A)
(0.8X+0.2Y) = 0.8 * 200 + 0.2 * 133 = 186.6 ・・・(B)
n = (A) / (B) = 20000 / 186.6 = 107.181

107.181 - (107.181 - 107.181 / 200 * 133) * 0.2 = 100

ピッタリ合いました!ありがとうございました。

735 :132人目の素数さん:2017/10/09(月) 12:59:01.56 ID:7XVwJ0zt.net
Σx^n は (-1, 1) で一様収束しないことを証明せよ。

736 :132人目の素数さん:2017/10/09(月) 20:46:17.58 ID:SdExnwjd.net
数学者と宇宙飛行士はどっちの方が頭が良いですか?

737 :132人目の素数さん:2017/10/09(月) 20:46:40.16 ID:q8aDrGm/.net
>>689>>691
>>675で質問したものです。
とても素晴らしい解答をありがとうございます。

ところで
1/a∫√{a+(1−a)s^2}ds
=(1/2a)【s√{a+(1−a)s^2}
+{a/√(1−a)}logTs+√[{a/(1−a)}+s^2]T】+c
ではないでしょうか?

それから
>>・a>1 のとき
>>√{(a-1)/a}s = S とおくと
>>∫1/√{a-(a-1)ss}ds ={1/√(a-1)}∫1/√(1-SS) dS >>={1/√(a-1)}arcsin(S),

ここで
{1/√(a-1)}∫1/√(1-SS) dS ={1/√(a-1)}arcsin(S)
となっていますが、Sを通常の変数として扱って良いのでしょうか?
SはS=√{(a-1)/a}sin(x)という関数だったはずです。
S=sin(x)と置いて良いのでしょうか?
S=√{(a-1)/a}sin(x)に戻して計算すると、
{1/√(a-1)}∫1/√(1-SS) dS
={1/√(a−1)}∫cos(x)/√{(a/a−1)−sin^2(x)}dx
となってしまいます。

ご回答を宜しくお願い致します。

738 :132人目の素数さん:2017/10/09(月) 20:54:57.89 ID:7XVwJ0zt.net
>>735

|x| < 1 / lim sup |a_n|^(1/n)

ならば

Σa_n*x^n は一様収束する。

↑は正しいか?

の答えは「正しくない」ですね。

739 :132人目の素数さん:2017/10/09(月) 20:57:49.48 ID:q8aDrGm/.net
>>737の訂正

×ところで
1/a∫√{a+(1−a)s^2}ds ・・・
○ところで0<a<1の場合
1/a∫√{a+(1−a)s^2}ds ・・・

それから
=(1/2a)【s√{a+(1−a)s^2}
+{a/√(1−a)}log?s+√[{a/(1−a)}+s^2]?】+c
と?が出てしまいましたが、この?のところは絶対値記号の縦線です。

740 :132人目の素数さん:2017/10/09(月) 21:01:06.41 ID:SdExnwjd.net
コンピュータ技術者は数学に詳しいですか?

741 :132人目の素数さん:2017/10/09(月) 21:18:57.54 ID:yFYs86n6.net
人による

742 :132人目の素数さん:2017/10/10(火) 00:45:14.80 ID:h4u4sSCs.net
>>736

もちろん興行会社です。

http://www.foxmovies-jp.com/dreammovie/

743 :132人目の素数さん:2017/10/10(火) 03:12:02.11 ID:h4u4sSCs.net
>>642

マクローリン展開で
x^(-x)= e^{-x・log(x)}= Σ[n=1,∞]{1/(n-1)!}{-x・log(x)}^(n-1)
よって
∫[0→1]x^(-x)dx

= Σ[n=1,∞]{1/(n-1)!}∫[0→1]{-x・log(x)}^(n-1)dx

= Σ[n=1,∞]1/(n^n){1/(n-1)!}∫[0→∞]t^(n-1)e^(-t)dt
 
= Σ[n=1,∞] 1/(n^n)

*) x = e^(-t/n)とおいた。

744 :132人目の素数さん:2017/10/10(火) 03:26:27.58 ID:h4u4sSCs.net
>>737

S=√{(a-1)/a}sin(x)とおいたので、同じ文字を使ってはいけません。

S=sin(y)とおくと、
∫1/√(1-SS) dS =∫cos(y)/√{1-sin(y)^2}dy = ∫ dy = y = arcsin(S),

745 :132人目の素数さん:2017/10/10(火) 12:09:20.81 ID:j/MGWuQk.net
松坂和夫著『解析入門2』を読んでいます。

アーベルの定理の証明の直前に以下の記述があります。

「以下では簡単のため R = 1 として論ずる。一般に整級数
Σa_n * x^n の収束半径が R のとき、 x を x/R におきかえて

Σa_n/R^n * x^n

を考えれば、この整級数の収束半径は 1 であるから、はじめから
R = 1 と仮定しても議論の一般性は失われない。」

明らかに、間違っていますね。

正しくは、以下ですね:

「x を R*x におきかえて

Σa_n*R^n * x^n

を考えれば」

746 :132人目の素数さん:2017/10/10(火) 12:58:19.72 ID:SrLyS62r.net
こいつの頭が明らかに間違ってそう

747 :132人目の素数さん:2017/10/10(火) 13:25:33.01 ID:j/MGWuQk.net
3 ≦ n とする。

n 次正方行列 A_n = (a_{i, j}) を以下で定義する。

a_{1, 1} = 1, a_{1, 2} = 1, 第 1 行の他の成分 = 0

2 ≦ i ≦ n - 1 とする。
a_{i, i-1} = a_{i, i} = a_{i, i+1} = 1, 第 i 行の他の成分 = 0

a_{n, n-1} = a_{n, n} = 1, 第 n 行の他の成分 = 0



det(A_n) を計算せよ。

748 :132人目の素数さん:2017/10/10(火) 15:00:48.49 ID:GS9SgjOk.net
扇形の弧の長さを求める方法に
L = 2r * π * (180θ/π)/360°
= 2rπ * θ/2π = 2rπθ/2π
= rθ
L = rθより、θ = L/r
というのがあったんですけど、
L = rθはどういう経緯でθ = L/r に変換されるんでしょうか。
これは移項によるものなんでしょうか?よろしくお願いします。

749 :132人目の素数さん:2017/10/10(火) 17:02:03.66 ID:1KPCDrtH.net
>>744
了解しました。

ところで>>691>>737>>739では、0<a<1の場合の答が違うのですが、これはどうでしょうか?

750 :132人目の素数さん:2017/10/10(火) 18:51:16.24 ID:uiwamXA9.net
数学&物理学&計算機科学軍 vs 哲学&神学&宗教学軍


ファイッ!!!!!!!!!!!!!!!!!!!!!!!!!!!!!!!

751 :132人目の素数さん:2017/10/10(火) 19:01:18.15 ID:qGzzWouP.net
>>744
>>675で質問したものです。

私もうっかりしてましたが、それ以前に>>689はcos(x)が抜けてませんか?

752 :132人目の素数さん:2017/10/10(火) 21:56:59.86 ID:bP691gSB.net
ds=cosxdx

753 :ウンピー:2017/10/10(火) 23:04:28.81 ID:D6dE2obF.net
41%の確率で当選するクジが23回が引いて外れる確率って何%ですか?

754 :132人目の素数さん:2017/10/10(火) 23:22:37.82 ID:b+AZraNZ.net
わからないんですね(笑)

755 :132人目の素数さん:2017/10/11(水) 00:18:47.25 ID:VVa9c/LF.net
(.59)^23 = .0005 %

756 :132人目の素数さん:2017/10/11(水) 00:32:06.06 ID:acXKCTJa.net
>>747

a_{i,i}= 2x,  (本問では x=1/2)
a_{i,j}= 1 (|i-j|= 1 )
とする。

D_n(x)= det(A_n)とおく。
1行目で展開すると漸化式が出る。
D_{n+1}= 2x D_n - D_{n-1},
(和積公式に似てる・・・("^ω^)
D_1(x)= 2x,
D_2(x)= 4xx-1,
これより
D_n(x)= U_n(x),   第2種チェビシェフ多項式

U_n(cosθ)= sin{(n+1)θ}/sinθ,

本問では θ=±π/3 だから
D_n = U_n(1/2)
= sin{(n+1)π/3}/sin(π/3)
=(2/√3)sin{(n+1)π/3},

757 :132人目の素数さん:2017/10/11(水) 00:36:25.28 ID:acXKCTJa.net
>>747

D_1 = 1,
D_2 = 0,
D_3 = -1,
以後は
D_{n+3}= - D_n,
で繰り返す。

758 :132人目の素数さん:2017/10/11(水) 00:43:46.28 ID:bcH7G9G3.net
二項定理を使って
x=Σ[k=0,n](2n+1)Ck
の時のxを求めたいのですが、どなたか回答お願いします。

759 :132人目の素数さん:2017/10/11(水) 00:55:13.15 ID:acXKCTJa.net
>>758

2x = Σ[k=0,n](2n+1)_C_k + Σ[k=0,n](2n+1)_C_(2n+1-k)
= Σ[k=0,2n+1](2n+1)_C_k
=(1+1)^(2n+1)
= 2^(2n+1),

x = 2^(2n)= 4^n,

760 :132人目の素数さん:2017/10/11(水) 03:41:57.74 ID:xb6VTS04.net
>>753誰か解る方いませんか?

761 :132人目の素数さん:2017/10/11(水) 05:10:21.02 ID:acXKCTJa.net
>>756
ポリエン C_n H_{n+2}のπ電子準位を簡単に概算するときに
使いますな。

ポリアセチレンともいいますが。

762 :132人目の素数さん:2017/10/11(水) 07:18:30.02 ID:YwkestEE.net
1 / ln x = 1 / ln 2 + 1 / ln 3 + 1 / ln 4 + ... + 1 / ln nの時、xの値はいくらか?
この問題が解けません。教えてくださいm(_ _)m

763 :132人目の素数さん:2017/10/11(水) 09:48:46.47 ID:t54YPMN3.net
オイラー定数やe+πというような数学定数が有理数であると示されたらq/pなるpとqは求まるんでしょうか

764 :132人目の素数さん:2017/10/11(水) 10:40:05.82 ID:HgIAH4lS.net
アーベルの定理ですが、なぜ以下のように書かないのでしょうか?

x = r > 0 で級数 Σa_n * r^n が収束していれば、 Σa_n * x^n は区間 (-r, r) で収束する。
(-r, r] で定義された関数 f(x) = Σa_n * x^n は x = r で連続である。

765 :132人目の素数さん:2017/10/11(水) 11:54:04.36 ID:u5OtejXr.net
算数問題なんでしょうか?

b+c+d=18
b+c+f=19
c+d+f=18

これで、b,c,d,f の数値を求められますか?
解法を教えて下さい

766 :132人目の素数さん:2017/10/11(水) 13:09:00.27 ID:JGswejKG.net
b=f=d+1, c=17-2d

767 :132人目の素数さん:2017/10/11(水) 14:00:26.48 ID:EEiBQN0U.net
AB=2(A-2)(B-2)

A=4+8/(B-4)

これの持っていきかたがわからないんですが、途中どうなりますか?

768 :132人目の素数さん:2017/10/11(水) 14:04:17.17 ID:4gMCQUWS.net
算数板かな?

769 :753:2017/10/11(水) 14:14:59.30 ID:xb6VTS04.net
解る方居ないようなのでしめますね

770 :729:2017/10/11(水) 14:57:24.49 ID:cXXCBauC.net
>>767
それ間違ってね?

771 :132人目の素数さん:2017/10/11(水) 15:09:46.54 ID:u5OtejXr.net
>>766
どもです。

でも、なんで
>>766
になるか解りません

頭の中の途中式を教えて下さい

772 :132人目の素数さん:2017/10/11(水) 15:15:31.16 ID:u5OtejXr.net
b+c+d=18
b+c+f=19

b=18-(c+d)
b=19-(c+f)

とかは、あってます?
それでも、わからないです


>>768
マジで、さんすう板があれば
そこに行くような質問ですね。

773 :132人目の素数さん:2017/10/11(水) 15:22:09.30 ID:EEiBQN0U.net
>>770
本にそう載ってるんだけど、ならないですよね?
AとBに適当な数はめてみると確かに合うんですが…

774 :132人目の素数さん:2017/10/11(水) 15:49:44.67 ID:Mns4pPjX.net
P(X)を冪集合として写像s:P(X)×P(X)→P(X)を
s(A,B)=(A∪B)-(A∩B), A,B∈P(X) と定めるとき
ベクトル空間の公理のうち和に関する公理を満たすことを示せ
またこのとき0と逆ベクトルに対応する要素は何か?

775 :132人目の素数さん:2017/10/11(水) 16:00:04.73 ID:+19mPQDu.net


776 :132人目の素数さん:2017/10/11(水) 16:28:16.76 ID:4RhSgzf6.net
AB=2(A-2)(B-2)

AB = 2(AB-2A-2B+4)

AB = 2AB-4A-4B+8

0 = AB-4A-4B+8

0 = A(B-4)-4B+8

4B-8 = A(B-4)

4(B-4)+8 = A(B-4)

4+8/(B-4) = A

777 :132人目の素数さん:2017/10/11(水) 16:47:38.96 ID:xQ4Pu0AN.net
>>774
和って?sのこと?ちゃんと書いてよ

778 :132人目の素数さん:2017/10/11(水) 16:51:19.51 ID:xQ4Pu0AN.net
つかコレだとベクトル空間持ち出すわけわかんねー
単にPAが加法群だって言わせたらいいだけじゃん

779 :132人目の素数さん:2017/10/11(水) 16:53:27.82 ID:xQ4Pu0AN.net
あそうかF_2上のベクトル空間だって言わせたいのか
なる〜

780 :132人目の素数さん:2017/10/11(水) 17:15:16.99 ID:Mns4pPjX.net
先に抽象ベクトル空間の問題と書いた方が良かったですね
申し訳ありません

781 :132人目の素数さん:2017/10/11(水) 17:33:48.60 ID:EEiBQN0U.net
>>776
あざーす!

782 :132人目の素数さん:2017/10/11(水) 17:54:45.05 ID:imWJNFIv.net
問題のだしっこ

783 :132人目の素数さん:2017/10/11(水) 17:58:53.79 ID:JGswejKG.net
善哉

784 :132人目の素数さん:2017/10/11(水) 18:35:24.22 ID:c0zjB6hR.net
sgn(x)を解析接続するとどうなるんですか?
sgn(Rex)でいいんですか?

785 :132人目の素数さん:2017/10/11(水) 18:41:40.61 ID:PBLxDg/9.net
>>762
解けまつよ。
x = e^{1/[1/ln(2)+1/ln(3)+ …… + 1/ln(n)]}

786 :132人目の素数さん:2017/10/11(水) 19:02:43.58 ID:ienMogdH.net
>>784
そもそもそれは適用外

787 :132人目の素数さん:2017/10/11(水) 19:22:46.12 ID:PBLxDg/9.net
>>762

x ≒ e^{1/Li(n)}

Li(n)= ∫[2,n] 1/log(z) dz ={n/log(n)}{1 + 1!/log(n) + 2!/log(n)^2 + …}

788 :132人目の素数さん:2017/10/11(水) 19:45:33.02 ID:HgIAH4lS.net
基幹講座 数学 微分積分
砂田 利一
固定リンク: http://amzn.asia/37V1lE2

↑この本ですが、何の役にも立たないくだらない本のようでした。

序文と参考文献に、高木貞二という名前が出てきますが、高木貞二とは誰でしょうか?

789 :132人目の素数さん:2017/10/11(水) 19:49:39.08 ID:HgIAH4lS.net
>>788

それと参考文献が情報量ゼロでした。

微分積分とは関係のない自分の著作と現在の大学の同僚の本が
挙げられているのが目につきました。

790 :132人目の素数さん:2017/10/11(水) 19:51:34.21 ID:HgIAH4lS.net
>>788

それと序文が意味のない長文です。

791 :132人目の素数さん:2017/10/11(水) 20:03:17.75 ID:4gMCQUWS.net
意味のない書き込みしかできない人が、人の本の文章を意味がないと批判するのは何かのジョークですか?
つまんないですよ

792 :132人目の素数さん:2017/10/11(水) 20:21:06.89 ID:ienMogdH.net
ちゃんとした大学に行く機会があれば
この人ももう少しまともな書き込みしたかもしれないね

793 :132人目の素数さん:2017/10/11(水) 20:54:20.16 ID:BBUVopz1.net
病気は治らないだろ

794 :132人目の素数さん:2017/10/11(水) 21:00:13.94 ID:thiPv21l.net
>>786
そうですか
ありがとうございます

795 :132人目の素数さん:2017/10/11(水) 22:08:23.37 ID:VVa9c/LF.net
>>765
b+d=18-c
b+f=19-c
d+f=18-c
辺々足して2で割ると
b+d+f=(55-3c)/2
これと第1式、第2式、第3式との差を
それぞれとって
f=(19-c)/2
d=(17-c)/2
b=(19-c)/2

796 :132人目の素数さん:2017/10/11(水) 22:34:34.17 ID:qkrUva81.net
ゴミならくれよ
まさか買いもせずに文句だけ言ってるとかはないよな

797 :132人目の素数さん:2017/10/11(水) 23:05:40.26 ID:fmj8TdpR.net
乞食も参戦の模様

798 :132人目の素数さん:2017/10/11(水) 23:05:56.82 ID:HgIAH4lS.net
松坂和夫著『解析入門5』ですが、

p.80に「テイラー(Tayror)の定理」などとと書かれています。

799 :132人目の素数さん:2017/10/11(水) 23:11:30.10 ID:FHQrRiBV.net
>>780
ベクトル空間の問題にする必要が無いってことよ

800 :132人目の素数さん:2017/10/11(水) 23:11:34.00 ID:4gMCQUWS.net
どうでもいい誤植の報告は心底要らないです

801 :132人目の素数さん:2017/10/11(水) 23:58:47.24 ID:BsXPvCcd.net
などとと

802 :132人目の素数さん:2017/10/12(木) 00:19:07.28 ID:8MrJICvq.net
>>771
式1と式2の左側の違いはdがfに変わっているだけだからfはdよりも1大きい
式1と式3はbがfに変わっているだけだからb=f
b,c,d,f は正の整数という条件があったとしても答えは8パターンあるので決めらんない

803 :132人目の素数さん:2017/10/12(木) 01:54:48.00 ID:esD3VYKn.net
おしぇーてhttps://i.imgur.com/lFhjgaX.jpg

804 :132人目の素数さん:2017/10/12(木) 02:22:05.90 ID:XRTxKb5K.net
>>803
AとBが同値関係にあることを A◎B と書くことにしてt、
最初の問では
・A◎A を示す
・A◎B ⇒B◎A を示す
・A◎BかつB◎C⇒C◎A を示す
どれも簡単な計算式の操作

Xが4元を持つとき、2元からなるXの部分集合の間で推移律が成り立たない例を作る

805 :132人目の素数さん:2017/10/12(木) 04:45:11.98 ID:saIb7jMi.net
>>798

なおときおり「Tailor展開」と誤記している人があるが、これでは「洋服屋展開」になってしまう。

〜 数セミ増刊「100人の数学者」日本評論社(1989)p.78(テイラー) 〜

806 :132人目の素数さん:2017/10/12(木) 04:55:15.71 ID:r3IcUw8p.net
誰か円の内角の和を教えてくれ

807 :132人目の素数さん:2017/10/12(木) 06:32:34.66 ID:SJpqtPNt.net
n→∞で180(n-2)→∞

808 :132人目の素数さん:2017/10/12(木) 08:49:49.97 ID:IVBPcmrA.net
M坂和夫というのはそんなにレベルの高い数学者ではなかった。
語り口は丁寧だが、あれは「わかってる人の丁寧」じゃあないね

数学を理解する力も浅く、教える力も弱い。
そういう人の本に粘着して欠点をあげつらうのは
サディズム以外の何物でもないと思うよ。

809 :132人目の素数さん:2017/10/12(木) 09:39:46.36 ID:qdxU76Qd.net
>そんなにレベルの高い数学者ではなかった。

て言うか、数学者というほどのレベルではなかった。と思うよ。
(まあ、時代の違いを加味すると俺もそんなとこかもしれんが)

後の意見には同意!

810 :132人目の素数さん:2017/10/12(木) 09:54:56.67 ID:uHMGsKLb.net
>>808
ずいぶん偉そうだなw

811 :132人目の素数さん:2017/10/12(木) 10:00:35.38 ID:qdxU76Qd.net
うん、偉い人じゃないの

812 :132人目の素数さん:2017/10/12(木) 10:11:53.46 ID:KdzIdWhi.net
どうでもいいが松坂アスペ君にのるアホ

813 :132人目の素数さん:2017/10/12(木) 10:37:07.11 ID:uHMGsKLb.net
>>811
お前がだよw
ここがね
>>808
>あれは「わかってる人の丁寧」じゃあないね
>数学を理解する力も浅く

814 :132人目の素数さん:2017/10/12(木) 11:59:22.58 ID:qFha4xIg.net
松坂和夫著『解析入門2』を読んでいます。

「a_n ≧ 0 (n = 1, 2, …) とし、

b_n = (1 + a_1) * (1 + a_2) * … * (1 + a_n)

とおく。級数 Σa_n の収束・発散と数列 (b_n) の収束・発散とは一致することを証明せよ。」

という問題があります。

Σa_n 収束 ⇒ (b_n) 収束

の証明がおかしいです。

「Σa_n が収束すれば、 a_n → 0 で、 lim log(1 + a_n) / a_n = 1 であるから、
Σ log(1 + a_n) も収束。」

と書いています。

a_n = 0 となるような n が無限に存在する場合にはまずいですよね。

f(x) = x - log(1 + x)
f'(x) = 1 - 1 / (1 + x) ≧ 0 (x ≧ 0)

だから

f(x) ≧ f(0) = 0 (x ≧ 0)

である。

よって、

a_n ≧ log(1 + a_n) (n = 1, 2, …, n)

よって、

Σa_n 収束 ⇒ (b_n) 収束

としなければだめですよね。

815 :132人目の素数さん:2017/10/12(木) 12:06:26.69 ID:qFha4xIg.net
>>808

松坂和夫さんの本は別に悪い本ではないと思います。
ただ、級数のところは Rudin の本をほぼ丸写ししていますね。
それも、Rudin のオリジナリティーが発揮されているであろう箇所を丸写ししています。

複素関数論は Ahlfors を参考にしていると書いているので、そこも似たような
状況なのではないかと推測します。

悪い本といえば、小林昭七さん、砂田利一さんらの本のことだと思います。

小林昭七さんは比較的有名な数学者だと思いますが、小林さんの微分積分の本を
読んで「分かっている人」が書いた本だと思う人がいるでしょうか?
「数学を理解する力も浅く、教える力も弱い。」人だとみな思うのではないでしょうか?

816 :132人目の素数さん:2017/10/12(木) 12:46:34.59 ID:uRw3SxoL.net
>>815
貴方の頭が弱いからでは?

817 :132人目の素数さん:2017/10/12(木) 12:50:28.55 ID:4VI+8AGQ.net
なぜ何もないのではなく、何かがあるのでしょうか?

これはリーマン予想を証明するのより遥かに難しいのではないでしょうか?

818 :132人目の素数さん:2017/10/12(木) 12:51:39.82 ID:Y6xYw2zt.net
使い古されたコピペ

819 :132人目の素数さん:2017/10/12(木) 15:09:47.94 ID:qFha4xIg.net
2項級数が x = -1, 1 で収束するか否かについて書いてある本はありますか?
松坂和夫さんの『解析入門2』には書いてあります。

820 :132人目の素数さん:2017/10/12(木) 15:15:05.87 ID:EStcVAtU.net
1行目で質問して2行目で答えるスタイル

821 :132人目の素数さん:2017/10/12(木) 15:18:36.93 ID:4VI+8AGQ.net
宇宙飛行士は超絶エリートですか?

822 :132人目の素数さん:2017/10/12(木) 15:26:33.17 ID:onGpjLp/.net
うんこヘマラヤの妄想

823 :132人目の素数さん:2017/10/12(木) 15:35:48.11 ID:4VI+8AGQ.net
フィールズ賞受賞者と最高裁長官はどっちの方が頭が良いのでしょうか?

824 :132人目の素数さん:2017/10/12(木) 17:48:11.22 ID:sAH6k+cP.net
何この人、認知症?怖い

825 :132人目の素数さん:2017/10/12(木) 19:15:57.73 ID:7KGwbcwP.net
ヘマラヤと松坂君と劣等感の中で一番賢いのは誰ですか?

826 :132人目の素数さん:2017/10/12(木) 19:28:33.31 ID:0NKgpsEq.net
日本人は全員ゴミ

827 :132人目の素数さん:2017/10/12(木) 20:44:26.23 ID:XSyb1nQ+.net
>>825
同一人物である可能性は?

828 :132人目の素数さん:2017/10/12(木) 22:07:15.82 ID:l3bawjyR.net
別スレに書きましたが過疎過ぎて反応がないのでここに書かせて下さい

塾で出された問題ですがさっぱり分かりません
これって具体的に求まりますかね?

【問題】
f(x)、g(x)は次の等式を満たす整式とする

(x+2)f(x)+(x-1)^2 g(x)=g(x-3)

このとき f(x)を(x-1)^2で割った余り及びg(x)を(x+2)で割った余りを求めよ

829 :132人目の素数さん:2017/10/12(木) 22:13:07.91 ID:H8zsmg3Q.net
無限大のものを消滅させることって可能ですか?
また、可能だとしたらどんな感じで消滅させるのでしょうか?
でも、消滅させられるってことは、有限ってことになりますよね・・・・・?
どうなんでしょう?

830 :ル.ヌー:2017/10/12(木) 22:21:23.99 ID:Hbkmuqaq.net
f(z)=z/sinz,z∈Cにおいて,
(1) z=0はf(z)の除去可能特異点であることを示せ。
(2) f(z)の極をすべて求めよ、また、極での留数を求めよ。
(3) z=0まで定義域を拡大したf(z)のz=0におけるマクローリン展開の2次の項までを求め  よ。
(1).(2).(3)の解答をお願い致します。 👀
Rock54: Caution(BBR-MD5:ae2afb6cd11f3e92f5cd12f037b4c3ac)


831 :132人目の素数さん:2017/10/12(木) 22:30:49.20 ID:Hbkmuqaq.net
f(z)=z/sinh z,z∈Cにおいて,
(1) f(z)はC上正則であることを示せ。
(2) z=0はf(z)の除去可能特異点であることを示せ。
(3) z=0まで定義域を拡大したf(z)のz=0におけるマクローリン展開の2次の項までを求めよ。
(1).(2).(3)の解答をお願い致します。

832 :132人目の素数さん:2017/10/12(木) 22:40:53.36 ID:QRJc344P.net
どなたか答えを教えてください。

https://i.imgur.com/TqPrpIt.jpg

833 :132人目の素数さん:2017/10/12(木) 23:40:47.03 ID:uHMGsKLb.net
>>828
解は任意ね
つまり
問題とは言えない

834 :132人目の素数さん:2017/10/13(金) 00:33:01.92 ID:cW10d3wP.net
覆面算です

SUN
LOSE
UNTIE
BOTTLE
ELISION
NINETEEN
NONENTITY
EBULLIENT
+)INSOLUBLE
NEBULOSITY

835 :132人目の素数さん:2017/10/13(金) 00:39:20.11 ID:J+azbOaN.net
面白問題スレへどうぞ

836 :132人目の素数さん:2017/10/13(金) 00:40:03.43 ID:J+azbOaN.net
>>830
福田カズ君?

837 :132人目の素数さん:2017/10/13(金) 02:11:33.64 ID:OUKo6kqm.net
>>804
なるほど、助かった!

838 :132人目の素数さん:2017/10/13(金) 10:07:21.75 ID:utILla7I.net
>>833
有り難うございます

839 :132人目の素数さん:2017/10/13(金) 15:14:09.24 ID:dW66WeDf.net
上野健爾著『複素数の世界』を読んでいます。

p.194の参考書のところに、

「L. Ahrfors」

などと書かれています。

840 :132人目の素数さん:2017/10/13(金) 15:17:25.38 ID:JmPvGaJe.net
まーた難癖かよ

841 :132人目の素数さん:2017/10/13(金) 16:57:37.89 ID:bAr2djX8.net
>>752

>>675で質問したものです。
>>689は、与式、sと記述されてるので、sin(x)を略して記述したものだと勘違いしてました。
よく見るとdsになってます。私の間違いでした。
教えて頂いているのに、恐縮なのですが、私のような馬鹿でも分かる記述をして頂けたらありがたいです。

>>737の私の
1/a∫√{a+(1−a)s^2}ds
=(1/2a)【s√{a+(1−a)s^2}
+{a/√(1−a)}log?s+√[{a/(1−a)}+s^2]?】+c
という解は与式のcos(x)が抜けてるものを計算したので間違いでした。
(?マークは絶対値の縦線)

それから>>691
・0<a<1 のとき
∫1/√{a +(1-a)ss}ds ={1/√(1-a)}Log{√[a +(1-a)ss]+ √(1+a)・s}
はどうやって導出したのでしょうか?
また最後の項は、√(1−a)・s
ではないでしょうか?
最後の項を直すと、おそらく正しいと思います。
 
私が計算すると、
∫1/√{a+(1−a)s^2}・ds={1/√(1−a)}∫1/√{(a/1−a)+s^2}・ds
s+√{(a/1−a)+s^2}=tと置くと
s=[t^2−{a/(1−a))}]/2t
ds={t^2+(a/1−a)}/2t^2・dt
よって与式は
{1/√(1−a)}∫【1/√〔{a/(1−a)}+[t^2−{a/(1−a)}]^2/(4t^2)〕】・[t^2+{a/(1−a)}]/2t^2・dt
={1/√(1−a)}log〔s+√[s^2+{a/(1−a)}]〕+c
となってしまうのですが、どこに間違いがあるのでしょうか。

またs=√{a/(1−a)}・tanθと置いた方法でも全く違う解が出てしまいます。

導出を教えて頂けないでしょうか?

842 :132人目の素数さん:2017/10/13(金) 17:42:59.08 ID:XMHlrXdN.net
f(z)=z/sinz,z∈Cにおいて,
(1) z=0はf(z)の除去可能特異点であることを示せ。
(2) f(z)の極をすべて求めよ、また、極での留数を求めよ。
(3) z=0まで定義域を拡大したf(z)のz=0におけるマクローリン展開の2次の項までを求めよ。
(1).(2).(3)の解答をお願い致します

843 :132人目の素数さん:2017/10/13(金) 20:14:33.18 ID:XMHlrXdN.net
830 は福田カズですけど 問題の解答は、頂けないのですか?

844 :132人目の素数さん:2017/10/13(金) 22:55:41.04 ID:3HtQr/J7.net
法華経と聖書はどっちの方が凄いですか?

845 :132人目の素数さん:2017/10/13(金) 23:16:36.73 ID:wpcFMDqv.net
dx分のdxの2自乗がxと答えたのですが、回答だと2xになっています

2はどこからきたんですか?

846 :132人目の素数さん:2017/10/13(金) 23:49:30.53 ID:kV3HhDTH.net
n^2 - m(m+1)/2 = 2 を満たす正の整数(m,n)が存在しないことの証明を御願い致します

847 :132人目の素数さん:2017/10/14(土) 00:44:01.31 ID:jpWhavfj.net
>>845
{(x+h)^2-x^2}/h=(x^2+2hx+h^2-x^2)/h=2x+h→2x(h→0)

848 :132人目の素数さん:2017/10/14(土) 01:01:49.74 ID:4clY7sjY.net
>>847
hってなんですか?

849 :132人目の素数さん:2017/10/14(土) 01:11:14.30 ID:jpWhavfj.net
なんだろ?
教科書にあった式を適当に省略してかいただけなので、あとは自分で調べてちょ

850 :132人目の素数さん:2017/10/14(土) 01:13:09.27 ID:4clY7sjY.net
dxの自乗をdxで割ったら、dとx1個が約分されますよね?
xだけが残るはずですが、2はどこから来るんですか、という質問です

851 :132人目の素数さん:2017/10/14(土) 03:25:27.67 ID:NjrsV7WB.net
(d×x×x)÷(d×x)=x
なるほど確かに君は正しい
きっと回答が間違ってるのだろうね

852 :132人目の素数さん:2017/10/14(土) 04:43:42.90 ID:R16ycQzr.net
1/2の階乗を部分積分を使って解ける事を最近知りました。解けるという事実は特に興味はないのですが階乗って自然数以外の時、どういう意味があるのでしょうか?というか自然数じゃなくてもいいのでしょうか?

853 :132人目の素数さん:2017/10/14(土) 05:57:14.32 ID:bhk6txZ8.net
任意のnで
∫[0,1] (xlogx)^n dx = n!(-1)^n/(n+1)^(n+1)
が成り立つことの証明を教えてください

854 :132人目の素数さん:2017/10/14(土) 06:43:44.33 ID:NeZ/jqZV.net
>>852
階乗じゃなくてΓ関数な

855 :132人目の素数さん:2017/10/14(土) 07:34:29.81 ID:9j4Cf2uu.net
dx=(x+h)-x=h
dx^2=(x+h)^2-x^2=2xh+h^2

dは小さな区間を表すからh→0に近づける

dx^2/dx=2x+h→2x

df(x)=f(x+h)-f(x) h→0

856 :132人目の素数さん:2017/10/14(土) 10:15:13.14 ID:CDOAZ8iY.net
リーマン球面って重要ですか?

本を読んでいると、なんか ∞ を合理化するために存在するようにしか思えません。

857 :132人目の素数さん:2017/10/14(土) 10:38:48.31 ID:NeZ/jqZV.net
まあそうよ
無限大での正則性を定義したい

858 :132人目の素数さん:2017/10/14(土) 11:21:53.28 ID:P95wvq7Q.net
どの方向の無限大も区別しないなんて
不合理のような気もしないではないけど
逆にそれが合理的だという点を
驚き賞賛しながら楽しく学べばいい

批判するのは簡単だけどな

859 :132人目の素数さん:2017/10/14(土) 12:29:10.21 ID:NeZ/jqZV.net
実数で+0と0と-0を区別しないようなもの

860 :132人目の素数さん:2017/10/14(土) 12:31:27.82 ID:4clY7sjY.net
↑これが数学板の実力です↑
専門板なのに異常にレベルが低い
せいぜい数学の少しできる高校生レベル

861 :132人目の素数さん:2017/10/14(土) 12:36:33.92 ID:NeZ/jqZV.net
>848 名前:132人目の素数さん Mail:sage 投稿日:2017/10/14(土) 01:01:49.74 ID:4clY7sjY
>>>847
>hってなんですか?
>850 名前:132人目の素数さん Mail:sage 投稿日:2017/10/14(土) 01:13:09.27 ID:4clY7sjY
>dxの自乗をdxで割ったら、dとx1個が約分されますよね?
>xだけが残るはずですが、2はどこから来るんですか、という質問です
>860 名前:132人目の素数さん Mail:sage 投稿日:2017/10/14(土) 12:31:27.82 ID:4clY7sjY
>↑これが数学板の実力です↑
>専門板なのに異常にレベルが低い
>せいぜい数学の少しできる高校生レベル

下らんこと書いとらんで勉強にいそしみ賜え

862 :132人目の素数さん:2017/10/14(土) 12:39:51.67 ID:4clY7sjY.net
実数において、+0や-0はどのように定義されるんですか?

863 :132人目の素数さん:2017/10/14(土) 12:42:15.70 ID:NeZ/jqZV.net
0に符号を付けたのが+0と-0

864 :132人目の素数さん:2017/10/14(土) 12:47:32.98 ID:NXqNNRVl.net
後藤さん?

865 :132人目の素数さん:2017/10/14(土) 13:04:13.38 ID:HwFJEqND.net
>>856
読んでねーのが丸分かり

866 :132人目の素数さん:2017/10/14(土) 13:39:46.05 ID:9pQwxPIT.net
>>862
まーた劣等感婆かよ
邪魔だし不愉快だからいなくなってくれ

867 :132人目の素数さん:2017/10/14(土) 13:40:58.47 ID:U/KcWapL.net
>>864
おいおい、私は最近ここには書いていないぞ。
最近、間違った人物特定されたのを見たのはこれで2回目だ。

868 :132人目の素数さん:2017/10/14(土) 13:51:14.65 ID:NXqNNRVl.net
そういえば後藤さんにしてはやけに短文だし、文字列の模様も句点の扱いも違うな

869 :132人目の素数さん:2017/10/14(土) 13:57:57.78 ID:W6mvcj2z.net
>>860
私の様なレベルの低い者が、質問してるのですから、

レベルの低い質問になってしまいます。

解答者がレベルが低いのではないと思います。

870 :132人目の素数さん:2017/10/14(土) 14:01:00.26 ID:W6mvcj2z.net
>>860
因みに私は>>675で質問した者です。

私のレベルが低いだけで解答者のレベルは高いと思います。

871 :132人目の素数さん:2017/10/14(土) 14:57:11.78 ID:4clY7sjY.net
ある無矛盾な公理系τの任意のモデルに対してある論理式φが常に真となるならば、τからφがLKにおいて証明可能となることを示せ

ここの回答者は何度貼ってもこの問題はわからないですからね

872 :132人目の素数さん:2017/10/14(土) 15:01:59.73 ID:o183pcNN.net
>>871
その質問の間抜けさに気付かないのが君が馬鹿にされる数多の理由の一つ

873 :132人目の素数さん:2017/10/14(土) 15:03:21.90 ID:4clY7sjY.net
間抜けな質問だろうがなんだろうが、わかるなら答えが返って来るはずですね

874 :132人目の素数さん:2017/10/14(土) 15:04:45.83 ID:o183pcNN.net
言っても分からんか
普通に教科書読めで仕舞いだ

875 :132人目の素数さん:2017/10/14(土) 15:05:09.23 ID:bhk6txZ8.net
>>853
お願いします

876 :132人目の素数さん:2017/10/14(土) 15:06:18.55 ID:4clY7sjY.net
教科書レベルなのはわかってますが、それすらわからないのがここの人たちですよね?

877 :132人目の素数さん:2017/10/14(土) 15:09:44.80 ID:o183pcNN.net
ここまで言ってもまだ分からん辺り、数学の教科書を読んだことがないんだろうな
「教科書レベルすら」
いやいや、「教科書レベルだから」君の要求は間抜けなんだよ
こう書くとまた誤解するんだろうけどw

878 :132人目の素数さん:2017/10/14(土) 15:11:05.75 ID:4clY7sjY.net
でも、あなたは実際解けって言われても解けませんよね?

879 :132人目の素数さん:2017/10/14(土) 15:13:56.41 ID:o183pcNN.net
思った通りの誤解っぷりだね

880 :132人目の素数さん:2017/10/14(土) 15:15:04.19 ID:4clY7sjY.net
わからないんですね(笑)

わかってるなら数行で終わることを書かないということは、そういうことです

881 :132人目の素数さん:2017/10/14(土) 15:16:05.85 ID:o183pcNN.net
とうとう決定的なミスを犯したね
「わかってるなら数行で終わること」
そういうことだよ、君の誤解は

882 :132人目の素数さん:2017/10/14(土) 15:16:51.78 ID:4clY7sjY.net
そうやって反論してるうちに回答かけますよね?

もし本当にわかってるなら(笑)

883 :132人目の素数さん:2017/10/14(土) 15:19:17.09 ID:o183pcNN.net
反論?君は議論でもしてるつもりだったのかw
俺は間抜けをからかうことで、他の人にも分かりやすく君の間抜けっぷりを晒しただけだぞ

884 :132人目の素数さん:2017/10/14(土) 15:20:12.87 ID:4clY7sjY.net
あなたの今までしてきたレス数使えば答えは完成しますね

答えではなく煽りを書き込むということは、わからないということです…(笑)

885 :132人目の素数さん:2017/10/14(土) 15:20:51.02 ID:o183pcNN.net
なんにせよ、
「わかってるなら数行で終わること」
この認識を改めない限り他の人との会話にはついていけない
高校スレに帰れって何度も言われたことあるだろ?

886 :132人目の素数さん:2017/10/14(土) 15:23:13.27 ID:4clY7sjY.net
任意の整合的な理論に対してそれを充足させるような少なくとも一つの解釈が存在する、ということを既知とします

これでも数行で答え書けないんですか?

887 :132人目の素数さん:2017/10/14(土) 15:28:03.88 ID:o183pcNN.net
いよいよ切羽詰ってるようだね
今の君は何が何でも「わかってるなら数行で終わること」という失言を取り繕おうとしてるだけだ
それを既知とする、ほんの「教科書レベル」が、一体どこにある?

888 :132人目の素数さん:2017/10/14(土) 15:28:56.43 ID:4clY7sjY.net
ここにあります

こんな簡単な問題なんですから、もちろんわかりますよね?
次のレスで回答以外のレスが返ってきた場合、あなたはわからないのだと判断します(笑)

889 :132人目の素数さん:2017/10/14(土) 15:32:01.54 ID:o183pcNN.net
ないよ、そんなものは
せめて本当に存在する教科書でも持ってこい
自明に同値な条件を書いて「証明せよ」とは、これまた間抜けだと気付いてない

890 :132人目の素数さん:2017/10/14(土) 15:41:06.88 ID:ikRP/D0j.net
>>889
わからないんですね(笑)
わからないレベルの低い人の相手をする暇はないですね

891 :132人目の素数さん:2017/10/14(土) 16:26:43.80 ID:NeZ/jqZV.net
>>886
数行で終わるって何で?
相当掛かると思うけど

892 :132人目の素数さん:2017/10/14(土) 16:27:06.34 ID:4clY7sjY.net
>>891
わからないんですね(笑)

893 :132人目の素数さん:2017/10/14(土) 16:30:58.81 ID:NeZ/jqZV.net
>>892
教えてください

894 :132人目の素数さん:2017/10/14(土) 16:31:45.19 ID:4clY7sjY.net
>>874さんによると、教科書を読め、だそうです(笑)

895 :132人目の素数さん:2017/10/14(土) 16:33:56.19 ID:NeZ/jqZV.net
>>894
それは知らないしどうでもいい
数行で書けるんなら書いてみてよ

896 :132人目の素数さん:2017/10/14(土) 16:37:08.36 ID:Zd83HJfE.net
じゃ、まずあなたの回答から見たいですね
あなたの場合は、問題文理解してるかすら怪しいですから

897 :132人目の素数さん:2017/10/14(土) 16:39:45.98 ID:o183pcNN.net
>>896
>>877

おそらく既に全ての応答パターンが出尽くしてるんじゃないかな
君の反応は余りにも型通りだから

898 :132人目の素数さん:2017/10/14(土) 16:39:48.74 ID:dtRv0inc.net
∫(0→π) (2/πi)(cosθ + isinθ)=4/π
となりますが、この答えは、

πを求める連分数計算における
1+1/(3+(1+3)/(5+(1+3+5)/(7+(1+3+5+7)/9+...=4/π
と等しいです

等しくなることの意味は何なのでしょうか?

899 :132人目の素数さん:2017/10/14(土) 16:42:01.27 ID:Zd83HJfE.net
>>897
レベルの低い人には聞いてません

900 :132人目の素数さん:2017/10/14(土) 16:43:42.83 ID:o183pcNN.net
>>899
都合が悪くなると主張を翻して俺をダシに使ったのは君だぞ
早く当初の主張通りに数行で証明してやれ

901 :132人目の素数さん:2017/10/14(土) 16:44:15.04 ID:Zd83HJfE.net
>>900
あなたがすればいいんじゃないですか?
できるものなら(笑)

902 :132人目の素数さん:2017/10/14(土) 16:46:11.42 ID:o183pcNN.net
>>901
「わかってるなら数行で終わること」と誤解していたのは君だ
俺はそれを否定し続けてきた
もう少し論理的に話そうよ

903 :132人目の素数さん:2017/10/14(土) 16:47:36.40 ID:B872vbOC.net
法華経と六法全書はどっちの方が凄いですか?

904 :132人目の素数さん:2017/10/14(土) 16:47:43.61 ID:Zd83HJfE.net
>>902
>>891>>886を用いてもわからないらしいですよ?

わかるそぶりを見せてたはずですが、もしかしてあなたもわからなかったんですか?
ま、そうでしょうね(笑)

905 :132人目の素数さん:2017/10/14(土) 16:48:49.25 ID:o183pcNN.net
>>904
>>889

906 :132人目の素数さん:2017/10/14(土) 16:49:43.47 ID:Zd83HJfE.net
>>905
自明でも少しは説明できるはずですよね?
で、その説明は数行以内で終わります

わからないんですか?

907 :132人目の素数さん:2017/10/14(土) 16:51:10.31 ID:o183pcNN.net
>>906
>>900

908 :132人目の素数さん:2017/10/14(土) 16:51:50.68 ID:Zd83HJfE.net
>>907
ループしましたね(笑)

ま、わからないんでしょうね

私はわかりますけど

909 :132人目の素数さん:2017/10/14(土) 16:52:48.50 ID:JUUr/Kyi.net
そろそろ次スレ必要かなと思うw

910 :132人目の素数さん:2017/10/14(土) 17:03:29.10 ID:B872vbOC.net
空海とマキシム・コンツェビッチはどっちの方が天才ですか?

911 :132人目の素数さん:2017/10/14(土) 17:08:02.85 ID:9pQwxPIT.net
公理系がどうとか、難しいことを知っている方に何度か

2つの基点付き空間A、Bに対して、ホモトピー同値写像
S(A×B)→S(A∧B)∨S(A)∨S(B)
が存在することを示せ

という問題を教えてもらおうとしているのですが、解答が頂けません
何故でしょうか?

912 :132人目の素数さん:2017/10/14(土) 17:10:21.65 ID:dtRv0inc.net
>>898
早急に答えが欲しいです
おねがいします

913 :132人目の素数さん:2017/10/14(土) 17:14:34.54 ID:2R5WAXw4.net
日本人は全員ゴミ

914 :132人目の素数さん:2017/10/14(土) 17:20:50.07 ID:NeZ/jqZV.net
>>901
数行で書けるって言っていたID:4clY7sjYと同じ人なのかな?

915 :132人目の素数さん:2017/10/14(土) 17:21:52.23 ID:Zd83HJfE.net
>>914
そうですね

916 :132人目の素数さん:2017/10/14(土) 17:27:22.56 ID:CDOAZ8iY.net
数学基礎論は完成しているのでしょうか?

完成していないとするといつ完成するのでしょうか?

917 :132人目の素数さん:2017/10/14(土) 17:28:10.34 ID:CDOAZ8iY.net
役にも立たない未完成品を勉強したいと思う人は少ないのではないでしょうか?

918 :132人目の素数さん:2017/10/14(土) 17:29:40.31 ID:9pQwxPIT.net
>>915
>>911に答えていただけませんか?

919 :132人目の素数さん:2017/10/14(土) 17:30:38.89 ID:NeZ/jqZV.net
>>915
数行で書けるのなら書いてよ

920 :132人目の素数さん:2017/10/14(土) 17:34:01.36 ID:Zd83HJfE.net
>>919
わからないんですね(笑)

921 :132人目の素数さん:2017/10/14(土) 17:35:30.40 ID:9pQwxPIT.net
>>920
>>911に答えていただけませんか?
無視しないでください

922 :132人目の素数さん:2017/10/14(土) 17:36:05.06 ID:NeZ/jqZV.net
>>917
完成って?
たしかに勉強したい人は少ないかも知れないけど未完成だから少ないというのはどうかしらね

923 :132人目の素数さん:2017/10/14(土) 17:36:20.47 ID:NeZ/jqZV.net
>>920
分かりません

924 :132人目の素数さん:2017/10/14(土) 17:37:25.61 ID:CDOAZ8iY.net
浅野孝夫著『アルゴリズムの基礎とデータ構造』を読んでいます。

「上の挿入ソートの例のように、基本演算回数(比較回数)は入力サイズ n にのみ
依存するとは言えない。そこで、入力サイズ n の入力のうちでアルゴリズムが最も
多くの基本演算を必要とする入力を考えて、それに対する基本演算回数を、本書ではん、
サイズ n の入力に対するアルゴリズムの計算量(time complexity of an algorithm)と
呼ぶ。すなわち、最悪の場合を想定してアルゴリズムの計算量を定めていることになる。
このようにして定められたアルゴリズムの計算量 T はもちろん n にのみ依存する関数で
あるので T(n) と書ける。上の挿入ソートの例では T(n) = n*(n-1)/2 である。」

と書いてあります。

その後、マージソートのところには、

「マージソートの計算量は T(n) = O(n*log(n)) である」

と書いてあります。

T(n) は最悪の場合の計算量ですから、

T(n) = Θ(n*log(n)) が正しいのではないでしょうか?

ちなみに、浅野さんは、この本の最初のほうで O, Ω, Θ を定義しています。

もちろん、 f(n) ∈ Θ(n*log(n)) ⇒ f(n) ∈ O(n*log(n)) ですが。

浅野さんは、挿入ソートの計算量を

O(n^2)

と書いています。

これも

Θ(n^2)

と書くべきですよね。

「上の挿入ソートの例では T(n) = n*(n-1)/2」

ですから。

925 :132人目の素数さん:2017/10/14(土) 17:40:26.70 ID:Zd83HJfE.net
>>923
そうですか

残念ですね(笑)

926 :132人目の素数さん:2017/10/14(土) 17:42:07.72 ID:NeZ/jqZV.net
>>925
あなたは不誠実な人だと思います

927 :132人目の素数さん:2017/10/14(土) 17:42:15.87 ID:9pQwxPIT.net
>>925
>>911に何故答えてくれないのですか?

928 :132人目の素数さん:2017/10/14(土) 17:47:06.69 ID:Zd83HJfE.net
τ|-φではないとします
{τ,¬φ}は無矛盾となります
もし矛盾しているならば、τ,¬φ|-すなわちτ|-φが証明可能となるので仮定に反します
>>886より{τ,¬φ}はモデルを持ちます
このモデルにおいては、τと¬φが真となりますが、これは仮定に反します

929 :132人目の素数さん:2017/10/14(土) 17:49:18.66 ID:iFT4waZT.net
揃いも揃ってID真っ赤っかで草

930 :132人目の素数さん:2017/10/14(土) 17:49:33.69 ID:9pQwxPIT.net
>>928
やはり賢いのですね
>>911をお願いします!

931 :132人目の素数さん:2017/10/14(土) 17:51:50.52 ID:CDOAZ8iY.net
上野健爾さんが、

a_n → α (n → ∞)

であるとき、

(a_1 + a_2 + … + a_n) / n → α (n → ∞)

が成り立つことは直観的な収束の定義からは導くことはできないと書いています。

でも、成り立つことは、直観的に明らかですよね。

932 :132人目の素数さん:2017/10/14(土) 17:53:59.76 ID:CDOAZ8iY.net
イプシロンデルタ論法を知らない高校生が

この結果を導いたとしても誰も驚きませんし、褒めることさえしないですよね。

933 :132人目の素数さん:2017/10/14(土) 18:02:13.28 ID:69ErbfRL.net
n^2 - m(m+1)/2 = 2 を満たす正の整数(m,n)が存在しないことの証明を重ねて御願い致します

934 :132人目の素数さん:2017/10/14(土) 18:04:50.73 ID:Ihl9MReE.net
結論

ID:4clY7sjY はただ煽るだけ

935 :132人目の素数さん:2017/10/14(土) 18:05:32.59 ID:Ihl9MReE.net
ID:Zd83HJfE

も同一人物

936 :132人目の素数さん:2017/10/14(土) 18:07:33.75 ID:Zd83HJfE.net
>>928
答え書きましたよね?

937 :132人目の素数さん:2017/10/14(土) 18:12:24.65 ID:9pQwxPIT.net
>>936
もしかして>>911は分からないんですか?

938 :132人目の素数さん:2017/10/14(土) 18:13:49.74 ID:QshKtkmn.net
>>928
伺いたいのですが,「証明可能」の定義は何ですか?

939 :132人目の素数さん:2017/10/14(土) 18:16:36.65 ID:Zd83HJfE.net
>>938
あるシークエントAから別のシークエントBへの証明図が存在することです

940 :132人目の素数さん:2017/10/14(土) 18:18:41.36 ID:QshKtkmn.net
>>939
シークエントと証明図の定義は何ですか?

941 :132人目の素数さん:2017/10/14(土) 18:23:06.55 ID:Zd83HJfE.net
論理式a1,a2,...,anおよびb1,b2,...,bmがあるとき
a1,a2,...,an|-b1,b2,...,bmをシークエントといいます

A,Bをシークエントとするとき、推論規則を用いて
A
---
B
というようにAからBを導く操作を推論といい、↑の図を推論図といいます
AからBへの証明図とは一番上のシークエントがAで、一番下のシークエントがBであるような推論図のことです

942 :132人目の素数さん:2017/10/14(土) 18:27:35.28 ID:QshKtkmn.net
>>941
論理式,推論規則,推論の定義は何ですか?

943 :132人目の素数さん:2017/10/14(土) 18:28:46.91 ID:Ihl9MReE.net
ID:QshKtkmn

こいつはこいつで...

944 :132人目の素数さん:2017/10/14(土) 18:29:07.48 ID:Zd83HJfE.net
>>942
いくつかの記号の集まりをL-言語(C,F,P)として以下で定義します
C:定数記号

F:関数記号

P:命題記号
述語記号

変数記号
論理記号

C,F,Pはある言語特有のものですが、変数記号と論理記号はいかなる言語でも共通のものが使われます
関数記号と述語記号にはアリティと呼ばれる自然数が対応付けられています

L-言語の項を以下で定義します
•定数記号は項である
•変数記号は項である
•アリティnの関数記号Fに対して、t1〜tnを項とすれば、F t1 t2 ... tnは項である
•以上で定められたものだけが項である

L-言語の論理式を以下で定義します
以下、t1〜tnは項、A,Bを論理式とします
•命題記号は論理式である
•アリティnの述語記号Pに対して、P t1 t2 ... tnは論理式である
•上で定めたP t1 t2 ... tnが変数記号xを含む時、∀x P t1 t2 ... tn、∃x P t1 t2 ... tnは論理式である
•¬Aは論理式である
•A∧Bは論理式である
•A∨Bは論理式である
•A→Bは論理式である
•以上で定められたものだけが論理式である

945 :132人目の素数さん:2017/10/14(土) 18:30:17.80 ID:Zd83HJfE.net
あるシークエントから別のシークエントへと書き換える操作を推論といい、推論を行う際の規則群を推論規則といいます

946 :132人目の素数さん:2017/10/14(土) 18:34:49.95 ID:Ihl9MReE.net
http://www.nue.riec.tohoku.ac.jp/user/aoto/lecture/15Logic/lecture6.pdf
http://www.sakabe.i.is.nagoya-u.ac.jp/~nishida/DB/pdf/nagashima04ss2004-33.pdf

947 :132人目の素数さん:2017/10/14(土) 18:35:18.57 ID:NeZ/jqZV.net
>>929
変な人を相手にしちゃったって感じでスマン

948 :132人目の素数さん:2017/10/14(土) 18:36:09.06 ID:QshKtkmn.net
>>944-945
集まり,言語,記号,定数〜論理記号の定義は何ですか?

949 :132人目の素数さん:2017/10/14(土) 18:38:11.83 ID:NeZ/jqZV.net
>>934
たぶんだけど
数学を考えることが不自由な人じゃないかなって気がする
ペダンティックな雰囲気を好むだけで
実質的にナニカしようとはしない人じゃないかなあ

950 :132人目の素数さん:2017/10/14(土) 18:39:08.10 ID:Zd83HJfE.net
>>948
集まり、言語、はメタのレベルでの素朴な意味だとします

記号はメタの意味での自然数のことです

定数〜論理記号は、記号に特定の意味を付加したもので、その意味は上に書いてあります

でも、論理記号がぬけていましたね
論理記号とは∀∃¬∧∨→のことです

>>949
私は答えを書きましたよ?
なんとか言ったらどうなんですか?

951 :132人目の素数さん:2017/10/14(土) 18:39:53.94 ID:Mp3l531I.net
>>949
劣等感ウンコ婆という荒らしをしらないのか?

952 :132人目の素数さん:2017/10/14(土) 18:40:42.71 ID:NeZ/jqZV.net
>>943
うむ

953 :132人目の素数さん:2017/10/14(土) 18:41:24.69 ID:QshKtkmn.net
>>950
それでは,「集まり」においてラッセルのパラドックスが回避できませんが,それでよいのですか?
また,「メタ」とが何でしょうか?
∀,∃,¬,∧,∨,→の定義は何ですか?

954 :132人目の素数さん:2017/10/14(土) 18:41:32.52 ID:NeZ/jqZV.net
>>950
まだ書いてないのでは?
数行でしょうに

955 :132人目の素数さん:2017/10/14(土) 18:43:33.43 ID:Zd83HJfE.net
>>954
928 名前:132人目の素数さん [sage] :2017/10/14(土) 17:47:06.69 ID:Zd83HJfE
τ|-φではないとします
{τ,¬φ}は無矛盾となります
もし矛盾しているならば、τ,¬φ|-すなわちτ|-φが証明可能となるので仮定に反します
>>886より{τ,¬φ}はモデルを持ちます
このモデルにおいては、τと¬φが真となりますが、これは仮定に反します

956 :132人目の素数さん:2017/10/14(土) 18:44:51.16 ID:Zd83HJfE.net
>>953
物理板でいいましたよね?
記号の集まりにラッセルのパラドックスは発生し得ません

メタとは、われわれの直観的な理解をそのまま議論に取り入れることです

記号の定義をみてください

957 :132人目の素数さん:2017/10/14(土) 18:45:55.44 ID:NeZ/jqZV.net
>>955
そこには説明するべき事柄が相当あるでしょ?
それにモデルが存在するというのはなぜ?
>>888
は大道具過ぎるし

958 :132人目の素数さん:2017/10/14(土) 18:46:39.84 ID:Zd83HJfE.net
>>957
>>886を認めることを前提としてます
あなたはそこにレスつけて突っ込んだんですよ
忘れたんですか?

959 :132人目の素数さん:2017/10/14(土) 18:46:46.57 ID:Mp3l531I.net
埋めるの手伝うね

960 :132人目の素数さん:2017/10/14(土) 18:47:33.94 ID:Mp3l531I.net
埋め

961 :132人目の素数さん:2017/10/14(土) 18:48:19.45 ID:Mp3l531I.net
埋め

962 :132人目の素数さん:2017/10/14(土) 18:48:54.39 ID:Mp3l531I.net
埋め

963 :132人目の素数さん:2017/10/14(土) 18:49:37.20 ID:Mp3l531I.net
埋め

964 :132人目の素数さん:2017/10/14(土) 18:49:47.54 ID:QshKtkmn.net
>>957
いえ,結局あなたは解答せずにいなくなりましたよね?

「記号は自然数」とありますが,そこに∀,∃,¬,∧,∨,→の定義があるのですか?
分からないので教えてください。

また,無矛盾(矛盾),モデルの定義もわかりません。
背理法による証明が正しいことの証明も教えてほしいです。

965 :132人目の素数さん:2017/10/14(土) 18:50:48.77 ID:Mp3l531I.net
埋め

966 :132人目の素数さん:2017/10/14(土) 18:50:50.28 ID:NeZ/jqZV.net
>>958
確かにそうか
でも説明するべき事柄はまだ相当あるよ

967 :132人目の素数さん:2017/10/14(土) 18:51:26.24 ID:Mp3l531I.net
埋め

968 :132人目の素数さん:2017/10/14(土) 18:51:40.54 ID:Zd83HJfE.net
>>964
なんらかのメタの意味での自然数を表すと考えれば良いですね

ある公理系τが矛盾するとは、τ|-が証明可能であることを言います

メタレベルでの証明の正当性を保証することはできません

969 :132人目の素数さん:2017/10/14(土) 18:52:11.72 ID:Zd83HJfE.net
>>966
ないですよねw

970 :132人目の素数さん:2017/10/14(土) 18:53:53.36 ID:NeZ/jqZV.net
>>949
これは撤回
数学を考えることはできるのかも知れないが
答えを考える新しく生み出すという数学の営みはしない人かもね

971 :132人目の素数さん:2017/10/14(土) 18:54:48.20 ID:QshKtkmn.net
>>968
ん?すいません,よくわかりません。
素朴な「集まり」がどうやってラッセルのパラドックスを回避するのか具体的に教えてください。

公理系,|,-の定義は何ですか?

「メタレベルで証明の正当性を保証することができないこと」の証明をお願いします。

972 :132人目の素数さん:2017/10/14(土) 18:55:28.04 ID:B872vbOC.net
東京大学大学院数理科学研究科数理科学専攻博士課程修了という肩書きを手に入れたい。

973 :132人目の素数さん:2017/10/14(土) 18:58:22.36 ID:NeZ/jqZV.net
>>928
>τ|-φではないとします
これはシーケント計算による証明図が存在することであるとかの説明が必要だし
>{τ,¬φ}は無矛盾となります
無矛盾の定義とか
>もし矛盾しているならば、τ,¬φ|-すなわちτ|-φが証明可能となる
これを示して
全部で数行

974 :132人目の素数さん:2017/10/14(土) 18:59:04.09 ID:PXMDLbLi.net
劣等感ウンコ婆

975 :132人目の素数さん:2017/10/14(土) 18:59:15.87 ID:cr7AB90f.net
すみませんが、荒らしの人は遠慮して頂けないでしょうか?
ここは小学生・中学年・高校生・大学生・私の様なレベルの低い者が質問する場所です。

>>841はいかがでしょうか?
一週間考えても、分かりませんでした。
馬鹿な私でも分かるように、宜しくお願いします。

976 :132人目の素数さん:2017/10/14(土) 18:59:43.26 ID:PXMDLbLi.net
劣等感ウンコ婆

977 :132人目の素数さん:2017/10/14(土) 19:00:00.84 ID:qyKoN/eR.net
>>933
(mod.9)。

978 :132人目の素数さん:2017/10/14(土) 19:00:26.27 ID:Zd83HJfE.net
>>971
できません
集まり、は上で述べた定義内において用います
記号の集まりや論理式の集まりがラッセルのパラドックスを起こすことはあり得ません

公理系とは論理式の集まりのことです

|や---は証明論におけるメタレベルにおいての記号です

メタレベルのわれわれの思考の道筋を論理式に書き起こしたとしても、その書き起こすこと自体が正しいのかどうか、などということはメタレベルでの話になってしまうので、現実問題を形式論理で解決することはできないのです
その際にはメタレベルでの考察が必要になります

979 :132人目の素数さん:2017/10/14(土) 19:00:32.58 ID:PXMDLbLi.net
劣等感ウンコ婆

980 :132人目の素数さん:2017/10/14(土) 19:00:39.80 ID:NeZ/jqZV.net
誰かが言ったっていう「教科書を読め」ってのは
あなたが前提としていることを説明するのに結構かかるってことを意図してるんじゃないかな
私もかなり掛かると思ってる

981 :132人目の素数さん:2017/10/14(土) 19:01:04.79 ID:PXMDLbLi.net
劣等感ウンコ婆

982 :132人目の素数さん:2017/10/14(土) 19:01:39.44 ID:Zd83HJfE.net
>>973
一から用語を説明しろってことですか??
なんにもわからないってことじゃないですか(笑)

983 :132人目の素数さん:2017/10/14(土) 19:01:58.84 ID:PXMDLbLi.net
劣等感ウンコ婆

984 :132人目の素数さん:2017/10/14(土) 19:02:38.10 ID:PXMDLbLi.net
劣等感ウンコ婆

985 :132人目の素数さん:2017/10/14(土) 19:03:12.34 ID:PXMDLbLi.net
劣等感ウンコ婆

986 :132人目の素数さん:2017/10/14(土) 19:03:23.58 ID:Zd83HJfE.net
>>980
あなたはこの掲示板で質問に答える際、いちいち最初から定義をつらつら並べるんですか?
上の方でガンマ関数云々言ってますけど、ガンマ関数の定義、関数の定義、実数の定義、集合の定義、とか説明してないのはなぜですか?

987 :132人目の素数さん:2017/10/14(土) 19:03:52.98 ID:PXMDLbLi.net
劣等感ウンコ婆

988 :132人目の素数さん:2017/10/14(土) 19:04:05.85 ID:NeZ/jqZV.net
>>986
教科書を読め
じゃないかな
説明を求められたらするけど?

989 :132人目の素数さん:2017/10/14(土) 19:04:45.64 ID:QshKtkmn.net
>>978
記号や論理式の集まりが,ラッセルのパラドックスを起こさないことを示してください。

メタレベルで解決してよいことと,解決してはいけないことの基準は何でしょうか?

>>986
貴方のやっていることが高等すぎるので,皆さん説明を求めているのです。

990 :132人目の素数さん:2017/10/14(土) 19:04:46.76 ID:PXMDLbLi.net
劣等感ウンコ婆

991 :132人目の素数さん:2017/10/14(土) 19:05:04.74 ID:Zd83HJfE.net
>>988
あなたはそんなこと言ってないですよね

なぜですか?
説明不十分ですよね

992 :132人目の素数さん:2017/10/14(土) 19:05:31.65 ID:PXMDLbLi.net
劣等感ウンコ婆

993 :132人目の素数さん:2017/10/14(土) 19:07:12.94 ID:Zd83HJfE.net
>>989
ラッセルのパラドックスは、自分自身を含む集まり、を考えますね
記号や論理式の集まりを考える際には、そんなことは考えませんから、起こらないのです

われわれが行う思考は全てメタレベルでの話です

994 :132人目の素数さん:2017/10/14(土) 19:07:52.28 ID:NeZ/jqZV.net
何かを前提に説明することは多いよね
でもその説明を求められたらある程度はするでしょ
そして
長くなるんだったら「教科書を読め」になる
数行でできると書いたのを提示しないってのは
不誠実な人格だと思われて仕方ないと思うよ

995 :132人目の素数さん:2017/10/14(土) 19:08:48.16 ID:Zd83HJfE.net
>>994
だから数行で証明しましたよね?
何が不満なんですか?

996 :132人目の素数さん:2017/10/14(土) 19:09:09.12 ID:NeZ/jqZV.net
君は
何かに答えるということがどういうことで「あるべきか」を
もう一度考え直してみてはどうかな

997 :132人目の素数さん:2017/10/14(土) 19:09:36.52 ID:NeZ/jqZV.net
>>995
それにはまだ相当説明が必要だから

998 :132人目の素数さん:2017/10/14(土) 19:09:49.19 ID:Zd83HJfE.net
>>996
あなたがバカだから証明わかりませーん、ってことですよね??

999 :132人目の素数さん:2017/10/14(土) 19:10:01.84 ID:QshKtkmn.net
>>993
考えれば起きるということですか?
集合論におけるラッセルのパラドックスも,変なことを考えなければ起きませんが,解決を必要としました。
「集まり」におけるパラドックスも同様に解決されるべきですよね?

1000 :132人目の素数さん:2017/10/14(土) 19:10:04.20 ID:PXMDLbLi.net
劣等感ウンコ婆

1001 :2ch.net投稿限界:Over 1000 Thread
2ch.netからのレス数が1000に到達しました。

総レス数 1001
263 KB
掲示板に戻る 全部 前100 次100 最新50
read.cgi ver.24052200